PATHOPHYSIOLOGY FINAL

¡Supera tus tareas y exámenes ahora con Quizwiz!

A newborn has respiratory distress syndrome. A nurse monitors the newborn for atelectasis because of: A. A lack of surfactant B. Pulmonary edema C. Airway obstruction D. Pulmonary fibrosis

A. A lack of surfactant

A 50-year-old female was prescribed a drug that acts as a negative inotrope. Which of the following endogenous substances would be most similar? A. Acetylcholine B. Dopamine C. Epinephrine D. Thyroid hormone

A. Acetylcholine

A 60-year-old female with a 25-year history of smoking is diagnosed with emphysema. She has an increased anterior-posterior chest diameter. The nurse attributes this finding to: A. Air trapping B. Alveolar destruction C. Decreased inspiratory reserve volumes D. Increased flow rates

A. Air trapping

An 80-year-old male presents to his primary care provider reporting difficulty breathing. Pulmonary function tests reveal that he has increased residual volume. A nurse suspects the most likely cause of this disorder is _____ in lung compliance. A. An increase B. A decrease C. No change D. An absence

A. An increase

A 53-year-old male with a 20-year history of smoking is diagnosed with emphysema. When a staff member asks why the patient's airways are obstructed, how should the nurse respond? The airways are obstructed because of: A. Loss of elastic recoil B. Airway edema C. Infection and inflammation D. Excessive mucus production

A. Loss of elastic recoil

While planning care for a patient with superior vena cava syndrome (SVCS), which principle should the nurse remember? SVCS is a progressive _____ of the superior vena cava (SVC) that leads to venous distention of the upper extremities and head. A. Occlusion B. Inflammation C. Distention D. Sclerosis

A. Occlusion

A 65-year-old male is diagnosed with chronic pulmonary disease and elevated pulmonary vascular resistance. Which of the following heart failures should the nurse assess for in this patient? A. Right heart failure B. Low-output failure C. Left heart failure D. High-output failure

A. Right heart failure

When a patient has a massive pulmonary embolism (PE), what complications will the nurse monitor for? A. Shock and death B. Disseminated intravascular coagulation (DIC) C. Chronic obstructive pulmonary disease (COPD) D. Damage to the lung microcapillaries

A. Shock and death

Which is the most beneficial medication treatment for a patient experiencing detrusor sphincter dyssynergia? a. Alpha-blocker b. Beta-blocker c. Vasodilator d. Diuretic

ANS: A Because the bladder neck consists of circular smooth muscle with adrenergic innervation, detrusor sphincter dyssynergia may be managed by alpha-adrenergic blocking (antimuscarinic) medications. Treatment for detrusor sphincter dyssynergia is not associated with beta-blockers, vasodilators, or diuretics. REF: p. 751

The directional migration of leukocytes along a chemical gradient is termed: a. chemotaxis. b. endocytosis. c. margination. d. diapedesis.

ANS: A Chemotaxis is the process by which leukocytes undergo directed migration. Endocytosis is a form of engulfment and a part of phagocytosis. Margination occurs when leukocytes adhere to endothelial cells in the walls of vessels. Diapedesis is the emigration of the cells through cell junctions that have loosened in response to inflammatory mediators. REF: p. 147

Which assessment result would the nurse expect to find associated with a patient diagnosed with Graves disease? a. High levels of circulating thyroid-stimulating autoantibodies b. Ectopic secretion of thyroid-stimulating hormone (TSH) c. Low circulating levels of thyroid hormones d. Increased circulation of iodine

ANS: A Graves disease results from a form of type II hypersensitivity in which there is stimulation of the thyroid by autoantibodies directed against the TSH receptor. The thyroid-stimulating antibodies stimulate TSH receptors; it is not an ectopic secretion. Graves disease is manifested by elevated levels of thyroid hormones. Iodine deficiency leads to goiter but not Graves disease. REF: p. 467

A 54-year-old patient with pulmonary tuberculosis is evaluated for syndrome of inappropriate ADH secretion (SIADH). Which electrolyte imbalance would be expected in this patient? a. Hyponatremia b. Hyperkalemia c. Hypernatremia d. Hypokalemia

ANS: A Hyponatremia occurs due to increased water reabsorption by kidneys. Hyperkalemia does not occur due to increased water reabsorption. Sodium levels are lowered with hyponatremia; they are not elevated. Hypokalemia does not occur; SIADH is a problem of sodium. REF: pp. 461-462

A 30-year-old female presents to her primary care provider with fever, cardiac murmur, and petechial skin lesions and is diagnosed with infective endocarditis. The most likely cause of the disease is: a. bacteria. b. viruses. c. fungi. d. parasites.

ANS: A Infective endocarditis is due to a bacterial infection, not a viral, fungal, or parasitic infection. REF: p. 629

Which characteristic changes should the nurse keep in mind while caring for a patient with left heart failure? As left heart failure progresses: a. left ventricular preload increases. b. systemic vascular resistance decreases. c. left end-diastolic volume decreases. d. pulmonary vascular resistance decreases.

ANS: A Left ventricular preload increases in left heart failure because less blood is ejected from the left ventricle. Left heart failure does not lead to a decrease in systemic resistance; it leads to an increase in resistance. Left end-diastolic volume will increase. Pulmonary vascular resistance will increase. REF: p. 632

A nurse recalls the most common cardiac valve disease in the United States is: a. mitral valve prolapse. b. pulmonary stenosis. c. tricuspid valve prolapse. d. aortic stenosis.

ANS: A Mitral valve prolapse is the most common valve disorder in the United States. Neither pulmonary stenosis, tricuspid valve prolapse, nor aortic stenosis is the most common valve disorder in the United States. REF: p. 628

Most cases of combined systolic and diastolic hypertension have no known cause and are documented on the chart as _____ hypertension. a. primary b. secondary c. congenital d. acquired

ANS: A Most cases of hypertension are diagnosed as primary hypertension, not secondary, which is due to a known cause. Most cases of hypertension are not a result of congenital or acquired causes. REF: p. 600

Things that influence potassium levels

-aldosterone -insulin -catecholamines

active immunity

-exposure to antigen -immunization -long lived

cortisol

-gets released by ACTH -Stimulates gluconeogensis -ant-inflammatory and immunosuppressive agent -promotes secretion of gastric acid in stomach(ulcers)

volatile

-h2co3(eliminated as gas) -Regulated by the lungs

clonal diversity

-happens before birth -t and b cells produced

b-endorphins

-in brain pain relief -response to stress -inflammed tissue activate this

kg to lbs

1 kg = 2.2 lbs

Normal Serum Sodium Level

135-145 mEq/L

A 30-year-old white male recently suffered a cerebrovascular accident. Which of the following is the most likely factor that contributed to his stroke? Question options: 1) Age 2) Gender 3) Diabetes 4) Race

3) Diabetes

A 15-year-old female scrapes her knee while playing soccer and complains of sharp and well-localized pain. Which of the following should the nurse document to most accurately characterize her pain? Question options: 1) Chronic pain 2) Referred pain 3) Somatic pain 4) Visceral pain

3) Somatic pain

Patients with myasthenia gravis often have tumors or pathologic changes in the: Question options: 1) Brain 2) Pancreas 3) Thymus 4) Lungs

3) Thymus

Calcium levels

9-10.5 mg/dL

ANS: C The presence of HPV is a factor in cervical cancer. The presence of herpes virus, rubella virus, or hepatitis B virus is not a factor in cervical cancer. REF: p. 250

A 45-year-old female was recently diagnosed with cervical cancer. Which of the following is the most likely cause of her cancer? a. Herpes virus b. Rubella virus c. Human papillomavirus (HPV) d. Hepatitis B virus

ANS: A These anemias are the result of ineffective erythrocyte DNA synthesis; hemoglobin is normal. These anemias are not related to use of vitamin C or blocked protein synthesis. REF: p. 515

A 45-year-old is diagnosed with macrocytic, normochromic anemia. The nurse suspects the most likely cause of this condition is: a. defective DNA synthesis. b. abnormal synthesis of hemoglobin. c. defective use of vitamin C. d. blocked protein synthesis.

ANS: B Portal hypertension occurs secondarily to cirrhosis of the liver. Portal hypertension is not associated with thrombosis of the spleen, left ventricular failure, or renal stenosis. REF: p. 927

A 54-year-old reports vomiting blood. Tests reveal portal hypertension. Which of the following is the most likely cause of this condition? a. Thrombosis in the spleen b. Cirrhosis of the liver c. Left ventricular failure d. Renal stenosis

ANS: A Night terrors are characterized by sudden apparent arousals in which the child expresses intense fear or emotion. Parasomnia is unusual behaviors during sleep. Somnambulism is sleep-walking. Enuresis episodes are bed-wetting. REF: p. 346

A child suffers from sudden apparent arousals in which she expresses intense fear or other emotion. Her mother reports that she seems to wake screaming, but that she is difficult to waken completely. The child most likely suffers from: a. night terrors. b. parasomnia. c. somnambulism. d. enuresis episodes.

respiratory acidosis

A drop in blood pH due to hypoventilation (too little breathing) and a resulting accumulation of Co2.

ANS: B Individuals with a 10+ year history of ulcerative colitis have a 30-fold increase in developing colon cancer. There is no relationship between pneumonia and lung cancer; between prostatic hypertrophy and cancer of the prostate; and between hypercholesterolemia and leukemia. REF: pp. 248-249

A nurse is giving an example of inflammation as an etiology for cancer development. What is the best example the nurse should give? a. Pneumonia and lung cancer b. Ulcerative colitis and colon cancer c. Prostatic hypertrophy and prostate cancer d. Hypercholesterolemia and leukemia

ANS: D A concussion is the most common type of traumatic brain injury. A concussion is a much more common brain injury than penetrating trauma, diffuse axonal injury, or focal injury. REF: p. 394

A nurse is preparing to teach staff about the most common type of traumatic brain injury. Which type of traumatic brain injury should the nurse discuss? a. Penetrating trauma b. Diffuse axonal injury c. Focal brain injury d. Concussion

ANS: B Individuals with hypotonia tire easily (asthenia) or are weak. They may have difficulty rising from a sitting position, sitting down without using arm support, and walking up and down stairs, as well as an inability to stand on their toes. Individuals with Parkinson disease have rigidity and stiffness. Symptoms of Huntington disease include irregular, uncontrolled, and excessive movement. Paresis, or weakness, is partial paralysis with incomplete loss of muscle power. REF: pp. 376-377

A patient reports tiring easily, having difficulty rising from a sitting position, and the inability to stand on toes. The nurse would expect a diagnosis of: a. Parkinson disease. b. hypotonia. c. Huntington disease. d. paresis.

A 45-year-old male was previously diagnosed with Parkinson disease. He has impaired fine repetitive motor movements. Which of the following areas is most likely damaged? A) Basal ganglia B) Prefrontal area C) Hippocampus D) Temporal lobe

A) Basal ganglia

The predominant antibody of a typical secondary immune response is: A) IgG. B) IgM. C) IgE. D) IgA.

A) IgG.

Which of the following series of laboratory values reflects a state of uncompensated metabolic alkalosis? A) Increased pH, increased bicarbonate B) Increased pH, decreased bicarbonate C) Decreased pH, increased bicarbonate D) Decreased pH, decreased bicarbonate

A) Increased pH, increased bicarbonate

The nurse is teaching the staff about aldosterone. Which information should the nurse include? The main site of aldosterone synthesis is the:

Adrenal Cortex

Dr. McCarley'sCellular MetabolismTake Home Message

All cells need energy to maintain normal metabolism. ATP is the major source of energy transport. If oxygen is present aerobic metabolism occurs producing 34 ATP. If oxygen is not present anaerobic metabolism occurs producing only 2 ATP. During anaerobic metabolism lactic acid is produced as a byproduct.

ANS: A Lack of vitamin K, not D, E, or B12, interferes with clotting. REF: p. 544

An IV drug user was diagnosed with hepatitis C 5 years ago. The patient is now experiencing impaired blood clotting. The nurse suspects a decrease in which of the following vitamins? a. K b. D c. E d. B12

ANS: D Communicating hydrocephalus occurs because of defective reabsorption of the fluid. Hydrocephalus can occur because of overproduction of CSF, but in adults it occurs most often because of defective reabsorption of the fluid. Hydrocephalus is not due to either intercellular edema or elevated arterial blood pressure. REF: p. 376

An adult is diagnosed with communicating hydrocephalus. The form of hydrocephalus in adults is most often caused by: a. overproduction of CSF. b. intercellular edema. c. elevated arterial blood pressure. d. defective CSF reabsorption.

An aide asks the nurse what is the most common cause of elevated levels of antidiuretic hormone (ADH) secretion. How should the nurse respond? a. Autoimmune disease b. Cancer c. Pregnancy d. Heart failure

B

Which cells in the pancreas secrete insulin? A) Alpha B) Beta C) Delta D) Acinar

B) Beta

If a person is exposed to antigen X and is later exposed to antigen X again, which of the following immune responses will occur? A) Primary B) Secondary C) Determinant D) Immunosuppressive

B) Secondary

A tumor that has regional lymph node involvement only is classified as: A) Stage I. B) Stage II. C) Stage III. D) Stage IV.

B) Stage II.

Increased thyroid hormone levels in the blood result in: A) further stimulus to produce thyroid hormone. B) decreased release of thyroid-stimulating hormone from the anterior pituitary. C) increased secretion of thyrotropin-releasing hormone from the hypothalamus. D) inhibition of parathyroid hormone release from the parathyroid glands.

B) decreased release of thyroid-stimulating hormone from the anterior pituitary.

Cell surface markers are composed of: A) phospholipids. B) glycoproteins. C) lactose molecules. D) cholesterol.

B) glycoproteins.

Cytokines are: A) adhesion molecules. B) growth factors. C) cell junctions. D) sequences of DNA.

B) growth factors.

In response to an increased workload, such as that caused by high blood pressure (hypertension), myocardial cells in the left ventricle will adapt through the process of: A) atrophy. B) hypertrophy. C) hyperplasia. D) dsyplasia.

B) hypertrophy.

The first cell to react to tissue injury is the: A) macrophage. B) mast cell. C) fibroblast. D) neutrophil

B) mast cell.

Osmolarity measures: A) the number of dissolved molecules per kilogram of water. B) the concentration of solute per volume of solution. C) the partial pressure of the gases dissolved in a solution. D) the permeability of a cellular membrane to solutes.

B) the concentration of solute per volume of solution.

Which of the following are characteristic of idiopathic pulmonary arterial hypertension (IPAH)? (Select all that apply.) A. Weight gain B. Fatigue C. Male gender D. Dyspnea E. Jugular vein distention

B. Fatigue D. Dyspnea E. Jugular vein distention

A 28-year-old male reports to his primary care provider that he has had a cold for a week and is coughing up bloody secretions. When giving report, what term should the nurse use to describe this condition? A. Hematemesis B. Hemoptysis C. Rhinitis D. Cyanosis

B. Hemoptysis

A 60-year-old female was diagnosed with mitral stenosis. As a result, the nurse realizes the patient has incomplete emptying of the: A. Right atrium B. Left atrium C. Right ventricle D. Left ventricle

B. Left atrium

A 20-year-old male presents to his primary care provider reporting difficulty breathing when lying down. What term should the nurse use to document this condition? A. Tachypnea B. Orthopnea C. Apnea D. Dyspnea

B. Orthopnea

A 62-year-old male presents to his primary care provider reporting chest pain at rest and with exertion. He does not have a history of coronary artery disease and reports that the pain often occurs at night. He is most likely experiencing which type of angina? A. Silent B. Prinzmetal C. Unstable D. Stable

B. Prinzmetal

A middle-aged female presents with painful, tender joints. Laboratory testing reveals elevated IgM and IgG. Which of the following is the most likely diagnosis? A. Inflammatory joint disease. B. Rheumatoid arthritis. C. Caplan syndrome. D. Ankylosing spondylitis.

B. Rheumatoid arthritis.

A nurse recalls the pleural membranes are examples of _____ membranes. A. Mucous B. Serous C. Synovial D. Peritoneal

B. Serous

A nurse remembers a low ventilation-perfusion ratio results in: A. Alveolar collapse B. Shunting C. Bronchoconstriction D. Increased dead space

B. Shunting

A 56-year-old male is diagnosed with coronary artery disease. Which of the following modifiable risk factors would the nurse suggest the patient change? A. Drinking tomato juice B. Smoking Cigarettes C. Living arrangements D. Eating meat

B. Smoking cigarettes

A patient is diagnosed with type 1 diabetes and is unable to produce insulin. The pancreatic cells that are most likely damaged are the

Beta cells

ANS: A, B, C, D Radiation sources can be temporarily placed into body cavities through a delivery method termed brachytherapy. Brachytherapy is useful in the treatment of cervical, prostate, and head and neck cancers. It is not used in the treatment of lung cancer. REF: p. 260

Brachytherapy is being used to treat cancer in a patient. What types of cancers respond well to brachytherapy? (select all that apply) a. Prostate b. Cervix c. Head d. Neck e. Lung

What type of immunity is conferred when an individual is given a vaccine? A) Natural immunity B) Passive acquired immunity C) Active acquired immunity D) Alloimmunity

C) Active acquired immunity

Capillary oncotic pressure is primarily determined by which of the following molecules? A) Glucose B) Sodium C) Albumin D) Water

C) Albumin

Major histocompatibility class I (MHC I) antigens are found on which of the following cells? A) Red blood cells B) B lymphocytes and macrophages only C) All body cells except for red blood cells D) Liver, heart, and bone marrow cells only

C) All body cells except for red blood cells

Mental status changes in people with Addison disease are caused by: A) CNS ischemia. B) encephalopathy. C) hypoglycemia. D) insulin resistance

C) Hypoglycemia Mental status changes in people with Addison disease are caused by hypoglycemia from low cortisol levels.

Which of the following hormones stimulates sodium excretion by the kidneys? A) Cortisol B) Aldosterone C) Natriuretic hormones D) Antidiuretic hormone

C) Natriuretic hormones

Prior to engulfment of a bacterium during phagocytosis, which of the following events must occur? A) Release of lysosomal enzymes B) Fusion C) Recognition and adherence D) Formation of a phagolysosome

C) Recognition and adherence

What is the normal pH of the blood? A) Neutral B) Slightly acidic C) Slightly alkaline D) Very acidic

C) Slightly alkaline

The process of triggering new blood vessel formation in a tumor is called: A) erythropoiesis. B) atherosclerosis. C) angiogenesis. D) seeding.

C) angiogenesis.

Adrenocorticotropic hormone (ACTH) is synthesized and released in the: A) adrenal cortex. B) adrenal medulla. C) anterior pituitary. D) posterior pituitary.

C) anterior pituitary. ACTH is synthesized and released by the anterior pituitary gland in response to stimulation from the hypothalamus.

Humoral immunity is generated through the process of: A) direct cell lysis. B) stimulating an inflammatory response. C) producing antibodies. D) secreting toxic molecules.

C) producing antibodies.

The process of muscle hypertrophy involves an increase in: A) cell division. B) water accumulation. C) protein synthesis. D) plasma membrane thickness.

C) protein synthesis.

A man has a neurodegenerative disorder that causes ascending paralysis but reverses itself after a period of time, this is known as? A. Multiple sclerosis B. Amyotrophic lateral sclerosis C. Guillian-Barre syndrome D. Myasthenia gravis

C. Guillian-Barre syndrome

A 68-year-old female is admitted to the emergency department with a diagnosis of polycythemia vera (PV). A nurse realizes the patient's symptoms are mainly the result of: A. A decreased erythrocyte count B. Rapid blood flow to the major organs C. Increased blood viscosity D. Vessel injury

C. Increased blood viscosity

ANS: B Chronic gastritis is classified as type A (fundal) or type B (antral), depending on the pathogenesis and location of the lesions. Gastritis is not classified by severity, age, or symptoms, but by location. REF: p. 914

Chronic gastritis is classified according to the: a. severity. b. location of lesions. c. patient's age. d. signs and symptoms

Which of the following electrolytes is found in the highest concentrations in the intracellular fluid (ICF)? A) Sodium B) Calcium C) Magnesium D) Potassium

D) Potassium

Which of the following is an example of an endogenous antigen? A) Poison ivy B) Injected medications C) Pollen D) The body's own tissue

D) The body's own tissue

The primary cell of immunity is the: A) mast cell. B) macrophage. C) neutrophil. D) lymphocyte

D) lymphocyte

The primary physiologic effect of insulin is to: A) stimulate nutrient absorption from the digestive tract. B) raise blood glucose levels. C) decrease basal metabolic rate. D) promote glucose uptake into cells.

D) promote glucose uptake into cells.

A common cause of metabolic acidosis is: A) diuresis. B) dehydration. C) vomiting. D) renal failure.

D) renal failure.

While planning care for a patient with respiratory difficulty and retaining too much carbon dioxide, which principle should the nurse recall? _______ would be stimulated in an attempt to maintain a normal homeostatic state. A. Irritant receptors B. Stretch receptors C. Peripheral chemoreceptors D. Central chemoreceptors

D. Central chemoreceptors

A 56-year-old male presents to his primary care provider for a checkup. Physical exam reveals edema, hepatomegaly, and muffled heart sounds. Which of the following is of greatest concern to the nurse? A. Exudate B. Pulsus paradoxus C. Aneurysm D. Tamponade

D. Tamponade

Which description appropriately defines the term anemia?

Decreased erythrocytes in circulating blood

Membrane Transport (Cont.)

Diffusion ØMovement of solutes from area of greater concentration to area of lesser concentration ØRate of diffusion influenced by difference of electrical potential across the membrane •Also influenced by size of molecules and lipid solubility Filtration ØMovement of water and solutes through a membrane because of greater force on one side than on the other •Hydrostatic pressure •Blood pressure

Factors for distribution of K in ECF/ICF

Insulin Aldosterone epinephrine alkalosis

A 35 year old is diagnosed with vitamin B12 deficiency anemia (pernicious anemia). How should the nurse respond when the patient asks what causes pernicious anemia? A decrease in ______ is the most likely cause.

Intrinsic factor

Hyponatremia S/S

Nausea/vomiting Muscle cramps Confusion Muscular twitching, coma Seizures Headache

Risk factors for insulin resistance include

Obesity Inactivity Age Medications

ANS: A The individuals at highest risk for cerebrovascular accident are blacks over 65 years of age. Older adults are at greater risk than younger adults, and blacks are at greater risk than whites. REF: p. 402

Of the following groups, who are at highest risk for a cerebrovascular accident (CVA)? a. Blacks over 65 years of age b. Whites over 65 years of age c. Blacks under 65 years of age d. Whites under 65 years of age

purulent exudate

Pus: indicates a bacterial infection

hypersecretion of antidiuretic hormone (ADH)

SIADH

ANS: A, B, D, E The patient will experience little or no paralysis of voluntary movement. The patient will experience normal or slightly increased deep tendon reflexes. Babinski will be negative (absent). Tremor will be present. Rigidity of muscle tone occurs intermittently. REF: p. 386, Table 15-19

The nurse is explaining clinical manifestations of alterations in the extrapyramidal system. The nurse would correctly include: (select all that apply) a. little or no paralysis of voluntary movement. b. normal or slightly increased tendon reflexes. c. positive (present) Babinski. d. presence of tremor. e. rigidity in muscle tone.

ANS: C The choroid is the deeply pigmented middle layer that prevents light from scattering inside the eye. The iris is a part of the choroid and contains the pupil, which lets light into the eye. The retina is the innermost layer of the eye. REF: p. 347

The ophthalmologist is teaching about the structure of the eye that prevents light from scattering in the eye. What structure is the ophthalmologist describing? a. Iris b. Pupil c. Choroid d. Retina

metastasis

The spread of cancer cells beyond their original site

Ligands

There are several different types of Ligands. Remember that ligands are molecules that binds with receptors to make something happen (example-blood vessel constriction, bronchial constriction). ØHormones ØNeurotransmitters ØAntigens ØComplement components ØLipoproteins ØInfectious agents ØDrugs Ømetabolites

Palliative surgery

To relieve or reduce intensity of illness; is not curative

Isotonic alterations

Total body water change with proportional electrolyte and water change (no change in concentration)

ANS: B, D Both hepatitis B and D have incubation periods of up to 180 days. REF: p. 934, Table 36-8

Which of the following type(s) of hepatitis has an incubation period of up to 180 days? (select all that apply) a. A b. B c. C d. D e. E

ANS: C The most common route of metastasis is through the lymphatics, not lung tissue, body cavities, or connective tissues. REF: p. 253

Which statement indicates the patient has a correct understanding of metastasis? The most common route of metastasis is through the blood vessels and: a. lung tissue. b. body cavities. c. lymphatics. d. connective tissues.

renin-angiotensin-aldosterone system

a hormone (aldosterone) cascade pathway that helps regulate blood pressure and blood volume by regulating Na and K

A 67-year-old female is admitted to the emergency department with a diagnosis of polycythemia vera (PV). Which treatment should the nurse discuss with the patient? a. Therapeutic phlebotomy b. Restoration of blood volume by plasma expanders c. Administration of packed red blood cells d. Iron replacement therapy

a. Therapeutic phlebotomy

When a nurse is asked about the purpose of vasodilation and increased vascular permeability during inflammation, how should the nurse respond? a. To bring white blood cells to the area of injury b. To inactivate inflammatory chemicals at the area of injury c. To concentrate toxins d. To spread injury to other cells

a. To bring white blood cells to the area of injury

A 50-year-old male complains of frequently recurring abdominal pain, diarrhea, and bloody stools. A possible diagnosis would be: a. Ulcerative colitis b. Hiatal hernia c. Pyloric obstruction d. Achalasia

a. Ulcerative colitis

A 39 year old female with chronic intermittent pain in the epigastric area 2-3 hours after eating is diagnosed with a duodenal ulcer. which of the following behaviors may have contributed to the development of the ulcer a. cigarette smoking b. drinking caffeinated beverages c. consuming limited fiber d. antacid consumption

a. cigarette smoking

A newborn has respiratory distress syndrome. A nurse monitors the newborn for atelectasis because of a. lack of surfactant b. pulmonary edema c. airway obstruction d. pulmonary fibrosis

a. lack of surfactant

A 2 year old malnourished child has vitamin B12 and folate deficiencies. a blood smear suggests the deficiency is macrocytic and normochromic. the nurse would expect the hemoglobin to be a. normal b. sporadic c. low d. high

a. normal

When a nurse is teaching about urinary pathogens in men. Which information should the nurse include? Mechanisms for defense against urinary pathogens in men include (select all that apply) a. the long length of the urethra b. alkaline pH of urine c. secretion of mucus that traps bacteria d. antimicrobial secretions from the prostate e. implantation of the ureters in the bladder

a. the long length of the urethra d. antimicrobial secretions from the prostate

While planning care for a patient with renal calculi, the nurse remembers the most important factor in renal calculus formation is a. urine pH b. body temperature c. gender d. serum mineral concentrations

a. urine pH

The cardinal signs of small bowel obstruction are a. vomiting and distention b. diarrhea and excessive thirst c. dehydration and epigastric pain d. abdominal pain and rectal bleeding

a. vomiting and distention

Cerebrospinal fluid (CSF) can accumulate around the brain when there is injury to the sites of CSF reabsorption, which are called the: a.Arachnoid villi b.Epidural foramina c.Lateral apertures d.Choroid plexuses

a.Arachnoid villi

ischemia-reperfusion injury

additional injury that can be caused by restoration of blood flow and oxygen

Distribution and amount of TBW change with ____ and ___

age body fat %

renin causes the release of what

aldosterone

Human Papillomavirus (HPV)

almost all cervical cancers -once infected the cells gets introduced to the new DNA making cancer cells

During childhood, the thymus decreases in size, and this is referred to as _____ atrophy? a. physiologic b. pathologic c. disuse d. neurogenic

ans: a a normal decrease in cell size is physiologic atrophy. Atrophy can result from disease (pathologic), disuse, or nerve injury (neurogenic). REF: p. 74

what principle should the nurse remember when trying to distinguish aging from diseases? a. it is difficult to tell the difference because both processes are believed to result from cell injury b. it is easy to tell normal processes from abnormal processes. c. disease, unlike aging, has a genetic component d. aging is defined as exceeding life expectancy, but not maximal life span

ans: a it is difficult to differentiate between aging and disease because both occur secondary to cellular aging. it is not easy to differentiate normal processes from abnormal because aging appears as a normal process. disease and agng have a possible genetic component. aging is a time-dependent loss of structure and function that proceeds slowly and in such small increments that it appears to be the result of the accumulation of small, imperceptible injuries. it is not at ime period outside of life expectancy REF: p. 107

a newborn male is diagnosed with albinism based on skin, eye, and hair apperance. which finding will support this diagnosis? a. increased melanin b. increased hemoproteins c. inability to convert tyrosine to DAPA (3,4-dihydroxyphenylalanine) d. inability to convert bile to bilirubin

ans: c the person with albinism is unable to convert tyrosine to DOPA, an intermediate in melanin biosynthesis. An increase in melanin would cause skin to be darker. Hemoprotein accumulations in cells are caused by excessive storage of iron, which is transferred to the cells from the bloodstream. An inability to convert bile to bilirubin would not lead to albinism. REF: p.100

when a nurse observes muscle stiffening occurring within 6-14 hrs after death, the nurse should document this finding as the presence of: a. livor mortis b. gangrene c. algor mortis d. rigor mortis

ans: d rigor mortis occurs within 6 hrs after death and is evidenced by muscle stiffening. Livor motis is a purple discoloration. Gangrene refers to death of tissue and results from severe hypoxic injury and does ot lead to stiffening. Algor mortis is postmortem reduction of body temp. REF: p. 109-110

platelets

assist in clotting

cell mediated immunity

b cells and plasma cells produced antibodies

Which principle should the nurse remember while planning care for a cardiac patient? Pressure in the left ventricle must exceed pressure in the _____ before the left ventricle can eject blood. a. Coronary arteries b. Aorta c. Inferior vena cava d. Pulmonary veins

b. Aorta

A 35-year-old male with hyperthyroidism begins treatment to decrease thyroid activity. A nurse monitors for which of the following conditions that could result secondary to the treatment? a. Eosinophilia b. Basophilia c. Monocytosis d. Lymphocytosis

b. Basophilia

A 60-year-old female emphysema patient experiences a rapid and pounding heart, dizziness, and fatigue with exertion. Which of the following respiratory assessment findings indicate the respiratory system is compensating for the increased oxygen demand? a. Bronchoconstriction b. Increased rate and depth of breathing c. Dyspnea d. Activation of the renin-angiotensin response

b. Increased rate and depth of breathing

A 67-year-old female has chronic gastrointestinal bleeding. A nurse recalls the primary cause of her anemia is: a. Vitamin B12 deficiency b. Iron deficiency c. Folate deficiency d. Bone marrow failure

b. Iron deficiency

Cholecystitis is inflammation of the gallbladder wall usually caused by: a. Accumulation of bile in the hepatic duct b. Obstruction of the cystic duct by a gallstone c. Accumulation of fat in the wall of the gallbladder d. Viral infection of the gallbladder

b. Obstruction of the cystic duct by a gallstone

a 22 year old male is admitted to the intensive care unit with a closed head injury sustained in a motorcycle accident. the injury has caused severe damage to the posterior pituitary. which of the following complications should the nurse anticipate a. dilutional hyponatremia b. dehydration from polyuria c. cardiac arrest from hyperkalemia d. metabolic acidosis

b. dehydration from polyuria

The primary causes of duodenal ulcers include (select all that apply) a. consumption of spicy foods b. nonsteroidal anti-inflammatory drugs (NSAIDs) c. H. pylori infection d. trauma e. side effects of antibiotics

b. nonsteroidal anti-inflammatory drugs (NSAIDs) c. H. pylori infection

A 28 year old female presents with fever, chills, and flank pain. She is diagnosed with pyelonephritis. A nurse recalls the patient's infection is located in the a. bladder b. renal pelvis c. renal tubules d. glomerulus

b. renal pelvis

granuloma formation

bodys attempt to wall off and isolate organisms

direct tumor invasion in what

bone marrow leads to leukopenia and thrombocytopenia

A 30 year old male is demonstrating hematuria with red blood cell casts and proteinuria exceeding 3 to 5 g/day, with albumin being the major protein. The most probable diagnosis the nurse will see documented on the chart is a. cystitis b. chronic pyelonephritis c. acute glomerulonephritis d. renal calculi

c. acute glomerulonephritis

a nurse checks lab results as both Cushing syndrome and Addison disease can manifest with elevated levels of a. ADH b. cortisol c. adrenocorticotropic hormone level (ACTH) d. aldosterone

c. adrenocorticotropic hormone level (ACTH)

A 30 year old male prison inmate contracted tuberculosis during an outbreak. while planning interactions, the nurse realizes the patient can transmit this disease through a. skin contact b. fecal-oral contact c. airborne droplets d. blood transfusions

c. airborne droplets

Polyuria occurs with diabetes mellitus because of: a. the formation of ketones. b. chronic insulin resistance. c. an elevation in serum glucose. d. an increase in antidiuretic hormone

c. an elevation in serum glucose.

a nurse is caring for a patient with SIADH. what severe complication should the nurse assess for a. stroke b. diabetes insipidus c. neurologic damage d. renal failure

c. neurologic damage

While turning a patient with chronic renal failure, which principle should the nurse recall? Bone fractures are a risk factor in chronic renal failure because a. calcium is lost in the urine b. osteoblast activity is excessive c. the kidneys fail to activate vitamin D d. autoantibodies to calcium molecules develop

c. the kidneys fail to activate vitamin D

A 24 year old female is diagnosed with renal calculus that is causing obstruction. Which of the following symptoms would she most likely experience a. anuria b. hematuria c. pyuria d. flank pain

d. flank pain

Which of the following can bind to plasma membrane receptors? a.Oxygen b.Ribosomes c.Amphipathic lipids d.Ligands

d.Ligands

alkalosis

decrease in H+ or increase in bicarb

atrophy

decrease in cell size may cause an entire organ to shrink

Aging does what to TBW

decreases

Hypokalemia

deficient potassium in the blood (<3.5mEq/L)

plasmin

degrades fibrins the clots

stage 4

distant metastasis

hypervolemia treatment

diuretics

catecholamines

epinephrine and norepinephrine

Generalized edema

evenly distributed accumulation

most reported symptom of cancer

fatigue

Dependent edema

feet,legs,buttocks,back

alopecia

hair loss from chemotherapy affect on hair follicles

passive transport

high to low, no energy, osmosis, hydrostatic pressure and diffusion

extracellular matrix

holds the cells together and creates latticework where cells can interact together

hormonal communication

hormone is released from a gland into the bloodstream and travels to its target cell

basophils

in blood -chemotaxis and histamines

Oxidative phosphorylation

in mitochondria, ATP from fats,CHO and proteins

beta receptors are where

in the heart and lungs

baroreceptors located

in walls of carotid sinuses aortic sinuses R atrium

dysfunctional collage synthesis

keloid scar, hypertrophic scar

serum sodium level maintained by the ______

kidney

Paroxysmal nocturnal dyspnea generally occurs with:

left ventricular failure

most common cause of pure water deficit

loss of free water in the kidneys

hypovolemia

low blood volume

active transport

low to high, energy required, pumps, endocytosis, exocytosis

monocytes turn into

macrophages at the site of inflammation

Total body water(TBW)

make up 60% of adult weight

precipitation

making the soluble antigen into an insoluble preciptate

cachexia

malnutrition cancer eats up all of the nutrients to divide leading to this -80% of cancer patients at death

osmolality

measures the number of milliosmoles per kilogram of water, or the concentration of molecules per weight of water

lipid peroxidation leads to

membrane damage and increased permeability

Epstein-Barr virus (EBV)

mono,cause b cell lymphomas

prostate cancer and breast cancer

most commonly diagnosed

infection

most significant cause of person who is malignant

osmosis

movement of water down gradient

anions

negatively charged ions

stage 1

no metastasis

apoptosis

normal cell death

capillary oncotic pressure

osmotically attracts water from the interstitial space back into the capillary

mast cells

outside vascular system -chemotaxis and histamines

Increased Na+ retention commonly comes from

oversecretion of aldosterone

prostaglandins

pain and vasodilation

cations

positively charged ions

ADH secretion

posterior pituitary gland vasoconstriction hypertension decreased urine output

Hydrostatic pressure

pressure exerted by fluids

interferons

protect against viral infections

function of epithelial tissue

protection, secretion, absorption, excretion

stage 3

regional structures

neurotransmitter

released from neuron into neuromuscular junction

memory cells

remember antigens

Osmoreceptors

respond to the osmolarity of the blood (water homeostasis) cause thirst signal POSTERIOR PITUITARY gland to release ADH

isotonic

same solute concentration

Natriuretic peptides

secreted by atrial myocardium of the heart in response to high blood pressure which cause vasodilation promote urinary excretion of Na and h2o

Hyponatremia

serum levels <135 mEq/L

Hypernatremia

serum levels > 145 mEq/L intracellular dehydration (water from ICF to ECF)

induction

shrink tumor to improve symptoms not providing cure

atrophy usually occurs

skeletal muscle, heart, secondary sex organs and the brain

hypertrophy commonly seen in

skeletal muscles, heart, kidneys

contraction

skrinking the wound

treatment of water deficit

slow administration of isotonic solution that is low in Na+ D5W

hypertonic

solution has more concentration than the blood

pediatric immunity

sufficient IgM, deficient IgG

chronic stress

suppresses immunity

lymphedema

swelling of the tissues due to an abnormal accumulation of lymph fluid within the tissues

stress decreases

t cell and b cell function

hormonal hyperplasia

uterus and breast, normal response of these organs to release estrogen(uterus for pregnancy each month)

hypervolemia S/S

weight gain,increased blood pressure,distended neck veins,edema

hypovolemia S/S

weight loss,dry skin,decreased urine output,tachycardia,flat neck veins,low blood pressure

pitting edema

when press a finger onto swollen area leaves a pit

secondary intention

wounds that require a great deal more tissue replacement(open wound) -fills from the bottom up

Connective tissue

ØDense regular or irregular ØFibers ØLoose and dense connective tissue ØElastic and reticular connective ØCartilage, bone, vascular, and adipose

Muscle tissue

ØSmooth ØStriated (skeletal) ØCardiac

Epithelial tissues functions

•Protection •Absorption •Secretion •Excretion

Healing by secondary intention would occur in which of the following patients? A patient with a: a. sutured surgical wound. b. stage IV pressure ulcer. c. paper cut. d. sunburn.

ANS: B A patient with a stage IV pressure ulcer would heal by secondary intention. A surgical wound and a paper cut would heal by primary intention. A patient with a sunburn would heal without needing either primary or secondary intention. REF: p. 152

Which hypothyroid diagnosis is supported by low levels of TSH? a. Primary b. Secondary c. Autoimmune d. Atypical

ANS: B Causes of secondary hypothyroidism are related to either pituitary or hypothalamic failure, which would be evident by low levels of TSH. Primary and autoimmune hypothyroidism would be evident by elevated levels of TSH. Atypical hypothyroidism would be evident by normal or elevated TSH. REF: p. 469, Figure 19-8

When a patient is diagnosed with coronary artery disease, the nurse assesses for myocardial: a. hypertrophy. b. ischemia. c. necrosis. d. inflammation.

ANS: B Coronary artery disease leads to myocardial ischemia. Coronary artery disease would not lead to hypertrophy, but not to ischemia, necrosis, or inflammation. REF: p. 610

For a patient experiencing metabolic acidosis, the body will compensate by: a. excreting H+ through the kidneys. b. hyperventilating. c. retaining CO2 in the lungs. d. secreting aldosterone.

ANS: B It is the lungs hyperventilating that would compensate for metabolic acidosis by blowing off CO2, not any function associated with the kidneys. CO2 retention would increase the acidotic state. Aldosterone would conserve water but does not help compensate for acidosis. REF: p. 127

Individuals with Raynaud disease need to be counseled to avoid which of the following conditions to prevent severe symptoms? a. Allergic reactions b. Cold exposure c. Hot water immersion d. Tissue injury

ANS: B Raynaud disease demonstrates symptoms when extremities are exposed to cold. It is not an allergic reaction, and it is not due to hot water immersion or tissue injury. REF: p. 607

As stated by the Frank-Starling law, there is a direct relationship between the _____ of the blood in the heart at the end of diastole and the _____ of contraction during the next systole. a. pressure; duration b. volume; force c. viscosity; force d. viscosity; duration

ANS: B The Frank-Starling law of the heart describes the length-tension relationship of ventricular end-diastolic volume (VEDV) (preload) to myocardial contractility (as measured by stroke volume). It does not refer to duration, pressure, or viscosity. REF: p. 582

What represents the sum of all ventricular muscle cell depolarization? a. PR interval b. QRS complex c. QT interval d. P wave

ANS: B The QRS complex represents the sum of all ventricular muscle cell depolarizations. The PR interval represents the onset of atrial activation to the onset of ventricular activity. The QT interval represents "electrical systole" of the ventricles. The P wave represents atrial depolarization. REF: p. 577

Baroreceptors are located in the: a. renal artery. b. superior vena cava. c. carotid artery. d. circle of Willis.

ANS: C Baroreceptors are found in the carotid arteries. They are not found in the renal artery, the superior vena cava, or the circle of Willis. REF: p. 583

Which control mechanism will a patient's target cells implement in order to adapt to high hormone concentrations? a. Negative feedback b. Positive feedback c. Down-regulation d. Up-regulation

ANS: C High concentrations of hormone decrease the number of receptors; this is called down-regulation. Thus, the cell can adjust its sensitivity to the concentration of the signaling hormone. Negative and positive feedbacks regulate hormone release. Up-regulation is a response to low concentrations of hormone, thus increasing the number of receptors per cell. REF: p. 441

Airway hyperresponsiveness in asthma is related to: a. increased sympathetic nervous system response. b. the release of stress hormones. c. exposure to an allergen causing mast cell degranulation. d. hereditary decrease in IgE responsiveness.

ANS: C Hyperresponsiveness is due to mast cell degranulation. An increased sympathetic response would lead to bronchiolar dilation. Hyperresponsiveness is not due to release of stress hormones. Heredity is associated with asthma, but the problem is the mast cells. REF: p. 698

A nurse is reviewing the results of an arterial blood gas (ABG) and finds reduced oxygenation of arterial blood. What term should the nurse use to describe this condition? a. Ischemia b. Hypoxia c. Hypoxemia d. Hypocapnia

ANS: C Hypoxemia is a reduction of oxygen in arterial blood. Ischemia is a lack of blood supply to tissues. Hypoxia is reduced oxygen in tissues. Hypocapnia is decreased CO2. REF: p. 689

A 10-year-old male is stung by a bee while playing in the yard. He experiences a severe allergic reaction and has to go to the ER. The nurse providing care realizes this reaction is the result of: a. toxoids. b. IgA. c. IgE. d. IgM.

ANS: C IgE is normally at low concentrations in the circulation. It has very specialized functions as a mediator of many common allergic responses. Neither toxoids, IgA, nor IgM is the mediator of common allergic response. REF: p. 164

A 23-year-old pregnant female visits her primary care provider for her final prenatal checkup. The primary care provider determines that the fetus has developed an infection in utero. Which of the following would be increased in the fetus at birth? a. IgG b. IgA c. IgM d. IgD

ANS: C IgM is synthesized early in neonatal life, and its synthesis may be increased as a response to infection in utero. REF: p. 161

The nurse realizes the patient diagnosed with mitral stenosis has incomplete emptying of the: a. right atrium. b. right ventricle. c. left atrium. d. left ventricle.

ANS: C Mitral stenosis would result in incomplete emptying of the left atrium, as the mitral valve is located between the left atrium and left ventricle. REF: p. 627

Which of the following individuals would be at greatest risk for an opportunistic infection? a. 18-year-old with diabetes b. 70-year-old with congestive heart failure c. 24-year-old who is immunocompromised d. 30-year-old with pneumonia

ANS: C Opportunistic microorganisms can cause disease if the individual's defenses are compromised. Diabetes, congestive heart failure, and pneumonia are not associated with immunocompromised disorders. REF: p. 137

When a nurse checks the patient for orthostatic hypotension, what activity did the nurse have the patient engage in? a. Physical exertion b. Eating c. Standing up d. Lying down

ANS: C Orthostatic hypotension refers to a drop in blood pressure when standing up, not a drop with exertion, eating, or lying down. REF: p. 604

A 40-year-old patient undergoes surgery for a PTH-secreting tumor in which the parathyroid is removed. Which physiological alteration would the nurse expect following surgery? a. Increased serum calcium b. Decreased bone formation c. Decreased calcium reabsorption in the kidney d. Increased calcitonin

ANS: C PTH acts on the kidney to increase calcium reabsorption and to decrease phosphate reabsorption. None of the remaining options would be expected with such surgery. testbanks_and_xanax REF: p. 450

Which portion of the antibody is responsible for the biologic functions of antibodies? a. Heavy chain b. Variable region c. Fc portion d. Epitope

ANS: C The Fc portion is responsible for most of the functions of antibodies. None of the remaining options are responsible for most of the functions of antibodies. REF: p. 161

The PR interval viewed on a normal electrocardiogram represents: a. atrial depolarization. b. ventricular depolarization. c. onset of atrial activation to onset of ventricular activity. d. "electrical systole" of the ventricles.

ANS: C The PR interval represents the onset of atrial activation to the onset of ventricular activity. The P wave represents atrial depolarization. The QRS complex represents ventricular depolarization. The QT interval represents "electrical systole" of the ventricles. REF: p. 577

Which chamber of the heart generates the highest pressure? a. Right atrium b. Left atrium c. Left ventricle d. Right ventricle

ANS: C The left ventricle generates the highest pressure of all the heart's chambers. REF: p. 571

The primary cardiovascular control center is located in the: a. cerebral cortex. b. thalamus. c. medulla. d. hypothalamus.

ANS: C The primary cardiovascular control center is in the brainstem in the medulla. The cerebral cortex, the thalamus, and hypothalamus are secondary control sites. REF: p. 583

A nurse recalls that regulation of acid-base balance through removal or retention of volatile acids is accomplished by the: a. buffer systems. b. skin. c. lungs. d. liver

ANS: C The volatile acid is carbonic acid (H2CO3), which readily dissociates into carbon dioxide (CO2) and water (H2O). The CO2 is then eliminated by the lungs. Buffer systems are throughout the body and operate in the extracellular and intracellular systems. Neither the liver nor the skin regulates acid-base balance. REF: p. 125

The most common condition associated with the development of acute pyelonephritis is: a. cystitis. b. renal cancer. c. urinary tract obstruction. d. nephrotic syndrome.

ANS: C Urinary obstruction and reflux of urine from the bladder are the most common underlying risk factors. Risk factors do not include cystitis, renal cancer, or nephrotic syndrome. REF: p. 754

When a 42-year-old is diagnosed with chronic renal failure, which dietary restriction will the nurse discuss with the patient? a. Fats b. Complex carbohydrates c. Proteins d. Sugars

ANS: C testbanks_and_xanax Low-protein diets are recommended. Management of chronic renal failure is not associated with diets that limit fats, carbohydrates, or sugars. REF: p. 767

A 5-year-old male presents to the ER with delirium and sunken eyes. After diagnosing him with severe dehydration, the primary care provider orders fluid replacement. The nurse administers a hypertonic intravenous solution. Which of the following would be expected? a. Symptoms subside quickly b. Increased ICF volume c. Decreased ECF volume d. Intracellular dehydration

ANS: D A hypertonic solution would cause fluid to move into the extracellular space, leading to intracellular dehydration. With this solution, his symptoms will not subside quickly because his cells will lose fluid. His intracellular volume will decrease and his extracellular volume will increase. REF: p. 120

When giving report, what term should the nurse use to describe the coughing up of bloody secretions? a. Hematemesis b. Cyanosis c. Rhinitis d. Hemoptysis

ANS: D Hemoptysis is the coughing up of bloody secretions. Hematemesis is bloody vomiting. Cyanosis is a bluish color to the skin. Rhinitis is a runny nose. REF: p. 688

A 35-year-old female took corticosteroid therapy for several months. Which of the following would the nurse expect to find? a. Renal toxicity b. Episodes of hypoglycemia c. Hypotension d. Type 2 DM

ANS: D Overt DM develops in approximately 20% of individuals with hypercortisolism. Diabetes develops not renal toxicity, but hyperglycemia and hypertension may occur. REF: p. 482

A 20-year-old female is applying for nursing school and is required to be tested for immunity against several illnesses. Testing that looks at which of the following would be the best to determine immunity? a. Culture and sensitivity b. Agglutination c. Precipitation d. Titer

ANS: D The amount of antibody in a serum sample is referred to as the titer; a higher titer indicates more antibodies. Culture determines the type of organism that causes an infection, and sensitivity identifies the antibody it is sensitive to. The terms agglutination and precipitation are not used to identify a test to determine immunity. REF: p. 167

The internal lining of the cardiovascular system is formed by what tissue? a. Tunica adventitia b. Connective c. Mesothelium d. Endothelium

ANS: D The endothelium, not the tunica adventitia, is the lining of blood vessels. Connective tissues help make up arterial walls but are not the lining of blood vessels. The mesothelium is a part of the pericardial cavity. REF: p. 570

Which of the following alterations would the nurse expect to find in a patient with untreated Cushing disease or syndrome? a. Bradycardia b. Tachypnea c. Hyperkalemia d. Hypertension

ANS: D With elevated cortisol levels, vascular sensitivity to catecholamines increases significantly, leading to vasoconstriction and hypertension. Tachycardia is more likely than bradycardia due to increased sensitivity to catecholamines. Tachypnea does not occur; the patient experiences hypertension. Hyokalemia, not hyperkalemia, occurs. REF: p. 482

Cellular energy step 3: Citric acid cycle

ATP(via oxidative phosphorylation) is produced if oxygen is present 32. Waste products include: water, urea, CO2 and ammonia

Chloride

Primary ECF anion Provides electroneutrality

angiogensis

formation of new blood vessels

immunosuppresions

fosters cancer by making your cancer more resistant to chemotherapy and radiotherapy

hypovolemia causes

hemorrhage,severe wound drainage,excessive sweating,reduced fluid intake

interstitial oncotic pressure

- Osmotically attracts water from the capillary into the interstitial space - Created by large molecules such as plasma proteins that cannot penetrate the membrane

chronic hepatitis

-80% of liver cancers - after being infected w/ hepatitis B or C

Hypokalemia treatment

-oral potassium -IV potassium give slowly -dietary supplements

ANS: B Hypochromic erythrocytes have low concentrations of hemoglobin. Hyperchromic erythrocytes have high concentrations of hemoglobin. Macrocytic and microcytic refer to cell size. REF: p. 517

. When a nurse is reviewing lab results and notices that the erythrocytes contain an abnormally low concentration of hemoglobin, the nurse calls these erythrocytes: a. hyperchromic. b. hypochromic. c. macrocytic. d. microcytic.

Body Fluids (cont'd)

1 mEq of any cation can combine chemically with 1 mEq of any anion; one monovalent anion will combine with one monovalent cation To maintain electrochemical balance, one divalent ion will combine with two monovalent ions (e.g., Ca+++ 2Cl- = CaCl2) The combining activity affects movement of electrolytes within and between body fluid compartments

What term should the nurse use when talking about the outermost membrane surrounding the brain? 1) Dura mater 2) Arachnoid mater 3) Pia mater 4) Falx cerebri

1) Dura mater

hypokalemia treatment (3)

1. Oral potassium 2. IV potassium given slowly 3. Dietary supplements: bananas, fruits, vegetables, fish

hypovolemia treatment (1)

1. Replace fluid using isotonic IV fluid solutions

direct response (3)

1. neutralization 2. agglutination 3. precipitation

edema treatment (5)

1.treat underlying disease 2. Meds that reduce fluid volume 3. Compression stocking 4. Elevating the affected extremity 5. Restricting fluid and salt intake

extrcellular make up how much percent

1/3

divalent

2 charges

paracrine

2 different cell communicating that are close together.

If a patient's posterior pituitary is removed, which hormone would the nurse expect to decrease? 1) PRF 2) ADH 3) ACTH 4) GH

2) ADH

If a patient had a problem with the adrenal medulla, which of the following hormones should the nurse monitor? 1) Cortisol 2) Epinephrine 3) Androgens 4) Estrogens

2) Epinephrine

Hikers are attempting to cross the Arizona desert with a small supply of water. The temperatures cause them to sweat profusely and become dehydrated. The hikers are experiencing: Question options: 1) Heat cramping 2) Heat exhaustion 3) Heat stroke 4) Malignant hyperthermia

2) Heat exhaustion

A patient is admitted with amyotrophic lateral sclerosis (ALS). Which classic assessment findings will support the diagnosis of ALS? Question options: 1) Progressive dementia 2) Muscle weakness and atrophy 3) Severe paresthesias 4) Autonomic dysfunctions

2) Muscle weakness and atrophy

When a staff member asks the nurse which gland secretes ADH and oxytocin, how should the nurse respond? 1) Anterior pituitary 2) Posterior pituitary 3) Hypothalamus 4) Pineal gland

2) Posterior pituitary

intracellular make up how much percent

2/3

2-3 weeks after maturation scar tissue gains

2/3 of strenght

Ratio of bicarbonate to carbonic acid

20:1 at 7.4 pH

macrophages arrive

24 hours or later after neutrophils, multiply in tissues,start wound healing

What is the cause of the hyperpigmentation seen in people with Cushing syndrome? 1) Abnormal levels of cortisol 2) Permissive effects of aldosterone when cortisol levels are altered 3) Elevated levels of ACTH 4) Hypersensitivity of melanocytes with sun exposure

3) Elevated levels of ACTH

A 50-year-old male presents with low back pain. He denies trauma and says he just woke up and it was hurting. An MRI reveals that the vertebra at L5 slid forward relative to those above and below it. Which of the following conditions will be documented on the chart? Question options: 1) Degenerative disk disease 2) Spondylolysis 3) Spondylolisthesis 4) Spinal stenosis

3) Spondylolisthesis

Normal ECF K Concentration

35-45 mEq/L (3.5-5.0 mEq/L)

neutrophils

6-12 hours to arrive, engulf and destroy invading organisms

ph normal ranges

7.35-7.45

Newborn/infant TBW

75%-80%

pediatrics how much TBW

75-80% of body weight, more likely to be dehydrated

____% of calcium is in the ____

99% Bone

ANS: A There is also an increased incidence of leukemia in association with other hereditary abnormalities such as Down syndrome, not hemophilia, not hyperthyroidism, and not pheochromocytoma. REF: p. 526

A 10-year-old is diagnosed with leukemia. The nurse assesses for which other condition that could be associated with his disease? a. Down syndrome b. Hemophilia c. Hyperthyroidism d. Pheochromocytoma

ANS: D EBV is associated with almost all cases of Burkitt lymphoma. Burkitt lymphoma is not associated with CMV, adenovirus, or HPV. REF: p. 535

A 10-year-old presents with abdominal swelling, night sweats, fever, and weight loss. He is diagnosed with Burkitt lymphoma. Upon obtaining the history, which of the following is the most likely cause? a. Cytomegalovirus (CMV) b. Adenovirus c. Human papillomavirus (HPV) d. Epstein-Barr virus (EBV)

ANS: A The symptoms indicate encephalitis, and given the residence of the child and the symptoms, the diagnosis is Eastern equine encephalitis. Venezuelan occurs in Texas, Florida, and the South. St. Louis occurs in Canada and the Pacific coast. West Nile occurs throughout the United States but primarily affects the elderly. REF: p. 410, Table 16-8

A 15-month-old child from Pennsylvania was brought to the ER with symptomology that includes fever, seizure activity, cranial palsies, and paralysis. Which form of encephalitis is best supported by the available assessment data? a. Eastern equine encephalitis b. Venezuelan encephalitis c. St. Louis encephalitis d. West Nile encephalitis

. ANS: B Transmission of mononucleosis is usually through saliva from close personal contact. The virus also may be secreted in other mucosal secretions of the genital, rectal, and respiratory tract, as well as blood. A surgical mask is not necessary because the disorder is not airborne. The patient does not need to avoid all contact, but precautions should be taken. REF: p. 525

A 15-year-old male with infectious mononucleosis is being given instructions on how to prevent the spread of this infection to others. Which statement represents a correct instruction? a. Wear a surgical mask when others are in the room. b. Do not share drinking glasses or eating utensils. c. Avoid all contact with other people. d. No precautions are necessary.

ANS: B Paratonia is manifested by resistance to passive movement that varies in direct proportion to force applied. Spasticity is manifested by a gradual increase in tone causing increased resistance until tone suddenly reduces. Rigidity is manifested by muscle resistance to passive movement of a rigid limb that is uniform in both flexion and extension throughout the motion. Dystonia is manifested by sustained involuntary twisting movement. REF: p. 377, Table 15-16

A 16-year-old male fell off the bed of a pickup truck and hit his forehead on the road. He now has resistance to passive movement that varies proportionally with the force applied. He is most likely suffering from: a. spasticity. b. paratonia. c. rigidity. d. dystonia.

ANS: B Persons with metabolically induced coma generally retain ocular reflexes even when other signs of brainstem damage are present. Psychogenic arousal activation demonstrates a general psychiatric disorder. Structurally induced coma is manifested by asymmetric responses. Structural arousal alteration does not have drug use as its etiology. REF: p. 360, Table 15-2

A 16-year-old's level of arousal was altered after taking a recreational drug. Physical exam revealed a negative Babinski sign, equal and reactive pupils, and roving eye movements. Which of the following diagnoses will the nurse most likely see on the chart? a. Psychogenic arousal alteration b. Metabolically induced coma c. Structurally induced coma d. Structural arousal alteration

ANS: A The macrocytic (megaloblastic) anemias are characterized by unusually large stem cells (megaloblasts) in the marrow that mature into erythrocytes that are unusually large in size (macrocytic), thickness, and volume. The hemoglobin content is normal, thus allowing them to be classified as normochromic. REF: p. 515

A 2-year-old malnourished child is diagnosed with vitamin B12 and folate deficiencies. A blood smear suggests the deficiency is macrocytic and normochromic. The nurse would expect the hemoglobin to be: a. normal. b. sporadic. c. low. d. high.

ANS: D When the anemia is severe or acute in onset (e.g., hemorrhage), the initial compensatory mechanism is peripheral blood vessel constriction, diverting blood flow to essential vital organs. Fluid moves into the vascular space, not the cell. Blood volume increases; thus, cardiac output increases. There is an increase in hemoglobin release of oxygen. REF: p. 515

A 25-year-old female has a heavy menses during which she loses a profuse amount of blood. Which of the following adaptations should the nurse expect? a. Movement of fluid into the cell b. Decreased cardiac output c. Decreased oxygen release from hemoglobin d. Peripheral vasoconstriction

ANS: A Epstein-Barr virus is associated with B-cell lymphoma. Kaposi sarcoma is associated with HIV. Retroviruses are associated with leukemia. Lipomas are not associated with HIV. REF: p. 250

A 30-year-old male with HIV is diagnosed with Epstein-Barr virus. After 2 months, the virus is still active. Based upon the Epstein-Barr virus, which of the following cancers is most likely to develop in this patient? a. B-cell lymphoma b. Kaposi sarcoma c. T-cell leukemia d. T-cell lipoma

ANS: C The majority of cases of ITP are due to immune-driven destruction of platelets. It is not due to allergies, hypersplenism, or T-cell injury. REF: pp. 541-542

A 30-year-old presents with hematuria, menorrhagia, and bleeding gums, and is diagnosed with immune thrombocytic purpura (ITP). A nurse realizes the most likely cause is: a. allergy-induced platelet lysis. b. an immune response to hypersplenism. c. antibody destruction of platelets. d. T-cell injury to megakaryocytes.

ANS: A, B, C, E Narcotic medications may be needed to relieve pain. To decrease pancreatic secretions oral food and fluids may be withheld initially and continuous gastric suction instituted. Parenteral fluids are essential to restore blood volume and prevent hypotension and shock. Steroid therapy is not associated with this disorder. REF: p. 937

A 31-year-old female is diagnosed with acute pancreatitis. Which of the following will be part of the treatment plan? (select all that apply) a. Narcotic analgesics b. Restriction of food intake c. Nasogastric suctioning d. Steroid therapy e. IV fluids

ANS: 2000 REF: p. 523

A 35-year-old male was diagnosed with hepatitis B. Further tests revealed neutropenia characterized by a neutrophil count less than _____ per milliliter.

ANS: B Huntington disease is manifested by chorea, abnormal movement that begins in the face and arms and eventually affects the entire body. There is progressive dysfunction of intellectual and thought processes. Tardive dyskinesia is manifested by rapid, repetitive, and stereotypic movements. Most characteristic is continual chewing with intermittent protrusions of the tongue, lip smacking, and facial grimacing. Hypokinesia is a loss of voluntary movement despite preserved consciousness and normal peripheral nerve and muscle function. Alzheimer disease is manifested by cognitive deficits and not movement problems; motor impairments will occur in the later stages. REF: p. 378 | p. 380

A 40-year-old male complains of uncontrolled excessive movement and progressive dysfunction of intellectual and thought processes. He is experiencing movement problems that begin in the face and arms and eventually affect the entire body. The most likely diagnosis is: a. tardive dyskinesia. b. Huntington disease. c. hypokinesia. d. Alzheimer disease.

ANS: B Folate deficiency occurs more often in alcoholics and individuals who are malnourished because of fad diets or diets low in vegetables. Being alcoholic promotes the greatest risk, since this disorder is diet related. A diet high in vegetables would help prevent folate deficiency. Age is not a factor in folate deficiency. Gender is not a factor in folate deficiency. REF: p. 517

A 40-year-old male's history includes being a vegetarian and abusing alcohol. Which of the following factors put him at greatest risk for developing folate deficiency anemia? a. Being vegetarian b. Being alcoholic c. Age d. Gender

ANS: D Regurgitation, dysphagia, and epigastric discomfort after eating are common in individuals with hiatal hernia. Pyloric stenosis is manifested by projectile vomiting. Gastric cancer is not manifested by heartburn. Achalasia is a form of functional dysphagia caused by loss of esophageal innervation. REF: p. 912

A 45-year-old male complains of heartburn after eating and difficulty swallowing. These symptoms support which diagnosis? a. Pyloric stenosis b. Gastric cancer c. Achalasia d. Hiatal hernia

ANS: A, B, D Chronic pancreatitis is associated with risk factors that include alcoholism, obstructive biliary tract disease (gallstones), and smoking. Neither trauma nor bulimia is associated with chronic pancreatitis. REF: p. 937

A 52-year-old female presents with continuous abdominal pain that intensifies after eating. She is diagnosed with chronic pancreatitis. Contributing factors include: (select all that apply) a. alcohol abuse. b. history of gallstones. c. trauma. d. smoking. e. bulimia

ANS: B OSAS generally results from upper airway obstruction recurring during sleep with excessive snoring and multiple apneic episodes that last 10 seconds or longer. Insomnia is the inability to fall or stay asleep and may be mild, moderate, or severe. It does not involve snoring. Somnambulism is sleep-walking. Jet-lag syndrome is a disorder of waking and sleeping and does not involve snoring. REF: p. 346

A 52-year-old male enters a sleep study to gather information about his sleep disturbances. He reports that his wife will not let him sleep in the bed with her until he stops snoring so loudly. He also reports feeling tired a lot through the day. When the nurse checks the chart, what is the most likely diagnosis? a. Insomnia b. Obstructive sleep apnea syndrome (OSAS) c. Somnambulism d. Jet-lag syndrome

ANS: A Liver and germ cell tumors secrete a protein known as AFP, not catecholamines. Prostate tumors secrete prostate-specific antigen. Homovanillic acid is a catecholamine marker. REF: p. 259, Table 10-7

A 52-year-old male with hepatitis C recently developed hepatic cancer. Which of the following markers should be increased? a. Alpha-fetoprotein (AFP) b. Catecholamines c. Prostate-specific antigen d. Homovanillic acid

ANS: A Adenocarcinomas are more prevalent in males and are associated with cigarette smoking, obesity, and gastroesophageal reflux disease (GERD). Intestinal parasites, ingestion of salty foods, or the use of antacids does not lead to adenocarcinoma of the esophagus. REF: p. 937

A 60-year-old male is diagnosed with adenocarcinoma of the esophagus. Which of the following factors most likely contributed to his disease? a. Reflux esophagitis b. Intestinal parasites c. Ingestion of salty foods d. Frequent use of antacids

ANS: B Agnosia is the failure to recognize the form and nature of objects. Hypomimesis is a disorder of communication. Dysphasia is an impairment of comprehension of language. Echolalia is the ability to repeat. REF: p. 367

A 65-year-old patient who recently suffered a cerebral vascular accident is now unable to recognize and identify objects by touch because of injury to the sensory cortex. How should the nurse document this finding? a. Hypomimesis b. Agnosia c. Dysphasia d. Echolalia

ANS: A When there is force applied to the skull, an injury may occur to the corresponding location on the brain. The injury may be coup (injury at site of impact) or contrecoup (injury from brain rebounding and hitting opposite side of skull). REF: pp. 390-391

A coup injury resulting from a blow to the frontal portion of the skull would occur in which region of the brain? a. Frontal b. Temporal c. Parietal d. Occipital

ANS: C Over 20-30 years, atheromatous plaques (stenotic lesions) form at branchings and curves in the cerebral circulation, primarily the arteries, not in veins, near the sphincters, or in the venous sinuses. REF: p. 404

A major contributing process in CVAs is the development of atheromatous plaques in cerebral circulation. Where do these plaques most commonly form? a. In the larger veins b. Near capillary sphincters c. In cerebral arteries d. In the venous sinuses

ANS: C Arousal is mediated by the reticular activating system, which regulates aspects of attention and information processing and maintains consciousness. The cerebral cortex affects movement. The medulla oblongata controls things such as hiccups and vomiting. The cingulate gyrus plays other roles in response. REF: p. 359

A neurologist explains that arousal is mediated by the: a. cerebral cortex. b. medulla oblongata. c. reticular activating system. d. cingulate gyrus.

ANS: B A lack of the intrinsic factor leads to pernicious anemia. Iron deficiency anemia is not related to the intrinsic factor. Pernicious anemia is due to the lack of the intrinsic factor; sideroblastic anemia is not related to the intrinsic factor. Hemolytic anemia results from destruction of cells. REF: p. 515

A newborn is diagnosed with congenital intrinsic factor deficiency. Which of the following types of anemia will the nurse see documented on the chart? a. Iron deficiency b. Pernicious c. Sideroblastic d. Hemolytic

ANS: A An individual who walks with the leg extended and held stiff, causing a scraping over the floor surface, is experiencing a spastic gait. A cerebellar gait is wide based with the feet apart and often turned outward or inward for greater stability. A basal ganglion gait occurs when the person walks with small steps and a decreased arm swing. A scissors gait is associated with bilateral injury and spasticity. The legs are abducted so they touch each other. REF: p. 385

A nurse notes that a patient walks with the leg extended and held stiff, causing a scraping over the floor surface. What type of gait is the patient experiencing? a. Spastic gait b. Cerebellar gait c. Basal ganglion gait d. Scissors gait

ANS: A, B, C The neural systems that are essential to cognitive function are: (i) attentional systems that provide arousal and maintenance of attention over time; (ii) memory and language systems by which information is communicated; and (iii) affective or emotive systems that mediate mood, emotion, and intention. The sensory, motor, and somatic systems are not involved. The tactile system is not involved in cognitive functioning. REF: p. 359

A nurse recalls that neural systems basic to cognitive functions include _____ systems. (select all that apply) a. attentional b. memory and language c. affective d. sensory and motor e. tactile

ANS: D REM sleep occurs every 90 minutes. REF: p. 345

A patient asks the nurse how often REM sleep occurs. The nurse responds, "About every _____ minutes." a. 15 b. 30 c. 60 d. 90

ANS: C Adjuvant chemotherapy is given after surgical excision of a cancer with the goal of eliminating micrometastases. Adjuvant chemotherapy is not given as the primary treatment or before radiation therapy. Adjuvant chemotherapy is indicated in the treatment of individuals with metastasis. REF: p. 261

A patient asks when adjuvant chemotherapy is used. How should the nurse respond? Adjuvant chemotherapy treatment is used: a. as the primary treatment. b. before radiation therapy. c. after surgical removal of a tumor. d. in cancer with little risk of metastasis.

ANS: C DBI is not associated with intracranial hypertension immediately after injury; however, acute brain swelling caused by increased intravascular blood flow within the brain, vasodilation, and increased cerebral blood volume is seen often and can result in death. Individuals who experience diffuse brain injury may experience visual impairments but do not experience loss of vision. A diffuse brain injury is not associated with arrhythmias. Individuals experiencing basilar skull fractures are at increased risk for the development of meningitis. REF: p. 394

A patient diagnosed with a diffuse brain injury (DBI) is at increased risk for which complication? a. Complete loss of vision b. Arrhythmia c. Acute brain swelling d. Meningitis infection

ANS: B Multiple myeloma is a B-cell cancer characterized by the proliferation of malignant plasma cells that infiltrate the bone marrow and aggregate into tumor masses throughout the skeletal system; thus, the pain is related to bone destruction, not neuropathic infiltrations, tissue hypoxia, or accumulation of toxic proteins. REF: pp. 536-537

A patient diagnosed with multiple myeloma reports severe pain. This pain can be attributed to: a. neuropathic infiltrations. b. destruction of bone tissue. c. tissue hypoxia. d. accumulation of toxic proteins.

ANS: A Some tumors initially described as benign can progress to cancer and then are referred to as malignant tumors. Lipomas are benign growths, while a meningioma is a benign tumor. Hypertrophy refers to tissue overgrowth, but not cancer. REF: p. 234

A patient has a tissue growth that was diagnosed as cancer. Which of the following terms best describes this growth? a. Malignant tumor b. Lipoma c. Meningioma d. Hypertrophy

ANS: A Normal ICP is 1-15 mm Hg; at 24 mm Hg, the patient's ICP is higher than normal. REF: p. 374, Box, 15-4

A patient is admitted to the neurological critical care unit with a severe closed head injury. When an intraventricular catheter is inserted, the intracranial pressure (ICP) is recorded at 24 mm Hg. How should the nurse interpret this reading? a. Higher than normal b. Lower than normal c. Normal d. Borderline

ANS: B The description is of a patient in decerebrate posturing. The description provided is not associated with decorticate, dystonic, or basal ganglion posturing. REF: p. 385

A patient is admitted to the neurological critical care unit with a severe closed head injury. All four extremities are in rigid extension, the forearms are hyperpronated, and the legs are in plantar extension. How should the nurse chart this condition? a. Decorticate posturing b. Decerebrate posturing c. Dystonic posturing d. Basal ganglion posturing

ANS: B The patient experiences disturbed thermal control because the sympathetic nervous system is damaged. The hypothalamus cannot regulate body heat through vasoconstriction and increased metabolism; therefore, the individual assumes the temperature of the air. In this situation there is no evidence to support the presence of pneumonia, brain injury, or septicemia. REF: p. 396

A patient is brought to the ER for treatment of injuries received in a motor vehicle accident. An MRI reveals spinal cord injury, and his body temperature fluctuates markedly. The most accurate explanation of this phenomenon is that: a. he developed pneumonia. b. his sympathetic nervous system has been damaged. c. he has a brain injury. d. he has septicemia from an unknown source.

ANS: C Vomiting is due to disruptions in the medulla oblongata. Skull fractures can result in vomiting but would not be related to the mass. The thalamus controls other things such as temperature. The frontal lobe deals with emotions. REF: p. 363

A patient presents to the emergency room (ER) reporting excessive vomiting. A CT scan of the brain reveals a mass in the: a. skull fractures. b. thalamus. c. medulla oblongata. d. frontal lobe.

ANS: B A continuous loss of blood is one of the most common causes of iron deficiency anemia (IDA). Gastrointestinal bleeding does not lead to vitamin B12 deficiency, folate deficiency, or bone marrow failure. REF: p. 517

A patient who demonstrates chronic gastrointestinal bleeding is diagnosed with anemia. What is the primary cause of the patient's anemia? a. Vitamin B12 deficiency b. Iron deficiency c. Folate deficiency d. Bone marrow failure

ANS: C Poikilocytosis means the erythrocytes are able to assume various shapes; it does not refer to color, size, and life span. REF: p. 513

A patient's anemia is described as having erythrocytes that demonstrate poikilocytosis. The nurse would recognize the erythrocytes would be: a. pale in color. b. present in various sizes. c. able to assume various shapes. d. live only a few days.

ANS: B Strangulation of the hernia is a major complication. Neither hemorrhage, peritonitis, nor ascites is associated with paraesophageal hiatal hernia complications. REF: p. 912

A serious complication of paraesophageal hiatal hernia is: a. hemorrhage. b. strangulation. c. peritonitis. d. ascites

ANS: D Stupor is a condition of deep sleep or unresponsiveness from which a person may be aroused or caused to open his or her eyes only by vigorous and repeated stimulation. Confusion is the loss of the ability to think rapidly and clearly and is characterized by impaired judgment and decision making. Coma is a condition in which there is no verbal response to the external environment or to any stimuli; noxious stimuli such as deep pain or suctioning do not yield motor movement. Obtundation is a mild-to-moderate reduction in arousal (awakeness) with limited response to the environment. REF: p. 361, Table 15-3

A teenager sustains a severe closed head injury following an all-terrain vehicle (ATV) accident and is in a state of deep sleep that requires vigorous stimulation to elicit eye opening. How should the nurse document this in the chart? a. Confusion b. Coma c. Obtundation d. Stupor

ANS: A The deviation of the eye inward is entropia, while deviation of the eye outward is extropia. Diplopia is double vision. Nystagmus is an involuntary unilateral or bilateral rhythmic movement of the eyes. REF: p. 348

A young child presents to the ophthalmologist for visual difficulties secondary to eye deviation. One of the child's eyes deviates inward, thereby decreasing the visual field. Which of the following diagnoses is most likely? a. Entropia b. Extropia c. Diplopia d. Nystagmus

Which of the following cell junctions allows small ions and other molecules to pass directly from the inside of one cell to the inside of another? A) Gap junctions B) Tight junctions C) Desmosomes D) Gating channels

A) Gap junctions

How do natural killer (NK) cells differ from cytotoxic T (Tc) cells? A) NK cells lack antigen-specificity and can target any infected or malignant cell. B) NK cells are phagocytic. C) NK cells can opsonize bacteria and viruses. D) NK cells release toxins that kill the target cell.

A) NK cells lack antigen-specificity and can target any infected or malignant cell.

Under normal conditions, which cell accounts for the greatest percentage of the white blood cellcount differential? A) Neutrophils (PMNs) B) Monocytes C) Bands D) Lymphocytes

A) Neutrophils (PMNs)

Which of the following is a serious complication of SIADH? A) Permanent neurological damage B) Renal failure C) Myocardial infarction D) Panhypopituitarism

A) Permanent neurological damage Hyponatremia in SIADH can lead to severe, irreversible neurological damage In SIADH the renal tubules are responsive to ADH and are active in reabsorbing large volumes of water. Increased secretion of ADH from the posterior pituitary does not affect other pituitary hormones. Myocardial infarction is not a complication of SIADH, although excessive fluid overload can lead to heart failure.

In addition to phagocytosis, which of the following functions are also performed by macrophages? A) Release of growth factors and stimulating new blood vessel growth B) Destroying circulating antibodies C) Inhibiting inflammation D) Promotion of blood clotting

A) Release of growth factors and stimulating new blood vessel growth

Metaplasia involves the replacement of normal cells by: A) another type of cell. B) abnormal cells of the same tissue type. C) scar tissue. D) cancer cells.

A) another type of cell.

Growth hormone is secreted into the bloodstream by the: A) anterior pituitary. B) posterior pituitary. C) hypothalamus. D) thymus.

A) anterior pituitary. Growth hormone from the anterior pituitary regulates metabolic processes related to cellular growth. The posterior pituitary releases antidiuretic hormone and oxytocin. The hypothalamus releases hormones that control the release of anterior pituitary hormones, including growth hormone. The thymus is not an endocrine gland

Sarcomas are cancers that arise from: A) connective tissues. B) glandular tissues. C) epithelial tissues. D) germ cells.

A) connective tissues.

If a person has innate resistance to a disease, the person has _____ immunity. A) natural B) native C) active acquired D) passive

A) natural

Opsonization promotes the process of: A) phagocytosis. B) vasodilation. C) increased vascular permeability. D) clotting.

A) phagocytosis.

Causes of myxedema coma include: A) untreated hypothyroidism. B) subclinical hyperthyroidism. C) thyroid storm. D) a reaction to abnormally high levels of thyroid autoantibodies.

A) untreated hypothyroidism.

Encephalitis is usually caused by: A) viral infection. B) bacterial infection. C) parasitic infection. D) fungal infection.

A) viral infection.

The primary effect of antidiuretic hormone (ADH) in the kidneys is to stimulate: A) water reabsorption. B) sodium reabsorption. C) sodium and water excretion. D) acid secretion.

A) water reabsorption.

The primary effect of antidiuretic hormone (ADH) in the kidneys is to stimulate: A) water reabsorption. B) sodium reabsorption. C) sodium and water excretion. D) acid secretion.

A) water reabsorption. The presence of ADH makes the renal tubules more permeable to water, which enhances water reabsorption. ADH does not affect sodium reabsorption in the kidneys. The presence of ADH makes the renal tubules more permeable to water, which enhances water reabsorption and decreases water excretion. The presence of ADH makes the renal tubules more permeable to water, which enhances water reabsorption. The presence of aldosterone will stimulate hydrogen ion (acid) secretion.

When a patient asks the nurse what is the most common cause of myocardial ischemia, which statement is the correct response? The most common cause of myocardial ischemia is: A. Atherosclerosis B. Venous emboli C. Arterial emboli from heart valve D. Idiopathic vasospasm

A. Atherosclerosis

A 42-year-old male was involved in a motor vehicle accident during which he suffered a severe head injury. He died shortly after the accident from loss of respiration. The nurse suspects the area of the brain most likely involved is the: A. Brainstem B. Basal ganglia C. Thalamus D. Cerebral cortex

A. Brainstem

A 50-year-old male is diagnosed with pulmonary embolism (PE). Which of the following symptoms most likely occurred before treatment? A. Chest pain and shortness of breath B. Shallow respirations and wheezing C. Dry cough and inspiratory crackles D. Kussmaul respirations and back pain

A. Chest pain and shortness of breath

A group of mountain climbers experience confusion, tachycardia, edema, and decreased renal output after climbing Mount Rainier. A nurse recalls this condition is caused by: A. Decreased inspired oxygen B. Diffusion abnormalities C. Hypoventilation D. Bronchoconstriction

A. Decreased inspired oxygen

A 28-year-old female presents with severe chest pain and shortness of breath. She is diagnosed with pulmonary embolism, which most likely originated from the: A. Deep veins of the leg B. Left ventricle C. Systemic arteries D. Superficial veins of the arm

A. Deep veins of the leg

A varsity tennis player states, "I have tennis elbow." Which condition best describes this injury? A. Lateral epicondylitis. B. Medial epicondylitis. C. Chronic bursitis. D. Tendinopathy.

A. Lateral epicondylitis.

When a patient wants to know about the renin-angiotensin-aldosterone system, what is the nurse's best response? Conversion of angiotensin I to angiotensin II happens in the: A. Lungs B. Liver C. Kidneys D. Heart

A. Lungs

What is the appropriate term for the failure of the bone ends to grow together? A. Nonunion. B. Delayed union. C. Malunion. D. Dislocation.

A. Nonunion.

A patient has a disease in which there is inadequate and delayed mineralization of osteoid in mature compact bone. This description describes which disease? A. Osteomalacia. B. Osteoporosis. C. Paget disease. D. Osteomyelitis.

A. Osteomalacia.

The nurse is describing the movement of blood into and out of the capillary beds of the lungs to the body organs and tissues. What term should the nurse use to describe this process? A. Perfusion B. Ventilation C. Diffusion D. Circulation

A. Perfusion

A 47-year-old male is diagnosed with pulmonary edema. Which assessment findings will the nurse observe? A. Pink, frothy sputum B. Wheezing C. Thick mucus secretions D. Hypocapnia

A. Pink, frothy sputum

A 65-year-old male recently had a cerebrovascular accident that resulted in dysphagia. He now has aspiration of gastric contents. The nurse assesses the patient for which complication? A. Pneumonia B. Bronchiectasis C. Pneumothorax D. Emphysema

A. Pneumonia

An 80-year-old female is in the hospital for a bone fracture. While there she develops a large, nonlethal pulmonary embolus. Which of the following is a direct result of the obstruction to pulmonary blood flow? A. Pulmonary hypertension B. Risk of cerebral emboli C. Pulmonary edema D. Systemic hypertension

A. Pulmonary hypertension

Many valvular stenosis and regurgitation disorders in adults have a common etiology. Which of the following conditions should alert the nurse that the patient may have both types of valve dysfunction? A. Rheumatic fever or heart disease B. Connective tissue disorders C. Syphilis infection D. Heart failure

A. Rheumatic fever or heart disease

While reviewing the results of the pulmonary functions test, the nurse is aware that the maximum amount of gas that can be displaced (expired) from the lung is called: A. Vital capacity (VC) B. Total lung capacity C. Functional capacity D. Residual volume

A. Vital capacity (VC)

Aldosterone's main effect increases ________ reabsorption while ADH's main effect increases _______ reabsorption. A. sodium; water B. water; potassium C. potassium; sodium D. glucose; potassium

A. sodium; water

A patient experiences nausea, vomiting, loss of body hair, fatigue, weakness and hypoglycemia. The hormone deficiency the patient is mostly likely experiencing is that of

ACTH

A function of the pericardium is to: a. provide a barrier against extracardial infections. b. improve blood flow through the heart. c. play a role in cardiac conduction. d. assist in cardiac contraction.

ANS: A A function of the pericardium is to provide a barrier against extracardial infections. The pericardium does not improve blood flow through the heart as it is on the outside. The inner portions of the heart control cardiac conduction. The muscular layers assist with cardiac contraction. REF: p. 570

What is the most common cause of Addison disease? a. An autoimmune reaction b. Dietary deficiency of sodium and potassium c. Cancer d. Viral infection of the pituitary gland

ANS: A Addison disease is caused by autoimmune mechanisms that destroy adrenal cortical cells and is more common in women. Addison disease is not due to dietary deficiency, to cancer, or to a viral infection. REF: p. 484

Within a normal physiologic range, an increase in left ventricular end-diastolic volume would lead the nurse to monitor for: a. an increased force of contraction. b. a decrease in cardiac output. c. an increase in heart rate. d. heart failure.

ANS: A An increase in end-diastolic volume leads to an increased force of contraction because fibers are stretched to handle increased volume. Cardiac output would increase. Although volume increases, heart rate does not respond accordingly. An increase in end-diastolic volume would not lead to heart failure since failure results in output decrease. REF: p. 582

One consequence of switching from aerobic to anaerobic cellular metabolism during shock states is: a. decreased adenosine triphosphate (ATP) production. b. cellular dehydration. c. cellular alkalosis. d. free radical formation.

ANS: A Anaerobic metabolism leads to decreased ATP production, not cellular dehydration, not cellular alkalosis, and not free radical formation. REF: p. 637

A 6-year-old female is diagnosed with a bacterial infection of the respiratory system. Which of the following will most likely try to fight the antigen? a. Antibodies b. Cytotoxic T cells c. Self-antigens d. Helper T cells

ANS: A Antibodies are produced by plasma cells that mature from lymphocytes, called B lymphocytes (B cells), in response to an antigen. Cytotoxic T cells do not respond to antigens. Self-antigens do not respond to antigens. Helper T cells do not respond to antigens. REF: p. 164, Figure 7-7

Aspiration of oral secretions increases a patient's risk for which complication? a. Pneumonia b. Bronchiectasis c. Pneumothorax d. Emphysema

ANS: A Aspiration could lead to pneumonia; bronchiectasis is related to dilation and is not associated with aspiration. Neither pneumothorax nor emphysema is associated with aspiration. REF: p. 704

When phagocytes begin to stick avidly to capillary walls, which process is occurring? a. Margination b. Exudation c. Integration d. Emigration

ANS: A Both leukocytes and endothelial cells secrete substances that increase adhesion, or stickiness, causing the leukocytes to adhere more avidly to the endothelial cells in the walls of the capillaries and venules in a process called margination. Exudation is the process of pus formation and does not result in stickiness. Integration occurs in cells but is not a major function and does not lead to stickiness. Emigration is similar to diapedesis and is not associated with increased stickiness. REF: p. 147

A patient presents with breast discharge, dysmenorrhea, and excessive excitability. Tests reveal that all pituitary hormones are elevated. What does the nurse suspect as the most likely cause for these assessment findings? a. A pituitary adenoma b. Hypothalamic hyposecretion c. Hypothalamic inflammation d. Pheochromocytoma

ANS: A Hormonal effects of pituitary adenomas include hypersecretion from the adenoma itself and hyposecretion from surrounding pituitary cells; in this case, prolactin would be elevated with the manifestation of menstrual irregularities and secretion from the breast. These symptoms are not indicative of hypothalamic inflammation, which would lead to hyposecretion. Pheochromocytoma is a tumor of the adrenal gland and would be manifested by elevated blood pressure. REF: p. 464

An immunology nurse is caring for a patient. While planning care, which principle will the nurse remember? The primary role of IgA1 is to prevent infections in the: a. blood. b. kidneys. c. lungs. d. mucous membranes.

ANS: A IgA1 is found predominantly in the blood. REF: p. 161

Which statement indicates a correct understanding of antibodies? The most abundant class of antibody in the serum is: a. IgG. b. IgM. c. IgA. d. IgE

ANS: A IgG is the most abundant class of immunoglobulins, constituting 80-85% of the immunoglobulins in the blood. REF: p. 161

A patient's arterial blood gas reveals decreased carbon dioxide (CO2) levels. What is the most likely cause of this situation? a. Hyperventilation b. Hypoventilation c. Apnea d. Cyanosis

ANS: A Individuals with hyperventilation blow off CO2 while individuals with hypoventilation retain CO2.Apnea is cessation of breathing.Cyanosis is a blue color to the skin. REF: p. 688

A 3-year-old is making play cakes in a sandbox and is eating the play cakes. The sand was also being used by cats as a litter box and was contaminated with toxoplasmosis. Which of the following would most likely also be present? a. Granuloma formation b. Degranulation c. Blood clots d. Exudate production

ANS: A Infections caused by bacteria such as toxoplasmosis can result in granuloma formation. Degranulation is a part of mast cell destruction. Blood clots are not expected with chronic inflammation. Exudate production is pus formation. REF: p. 150

A patient presents with polyuria and extreme thirst and is given exogenous ADH. For which of the following conditions would this treatment be effective? a. Neurogenic diabetes insipidus b. Psychogenic diabetes insipidus c. Nephrogenic diabetes insipidus d. SIADH

ANS: A Neurogenic diabetes insipidus is caused by the insufficient secretion of ADH; thus, exogenous ADH would be useful in the treatment of this disorder. Psychogenic diabetes insipidus is due to increased intake of water and would not respond to exogenous ADH. ADH is high in nephrogenic diabetes insipidus; thus, exogenous ADH would be contraindicated. SIADH is manifested by high levels of ADH; thus, exogenous administration of ADH would be contraindicated. REF: p. 462

A 72-year-old female has a history of right heart failure caused by a right ventricular myocardial infarction. Which of the following symptoms are specifically related to her right heart failure? a. Significant edema to both lower legs and feet b. Hypertension c. Decreased urine output d. Dyspnea upon exertion

ANS: A Right-sided failure allows blood to back up into the systemic circulation, leading to peripheral edema. Since it is backed up into the venous system, hypertension is less likely. Right-sided failure leads to edema and a greater venous blood volume, which would lead to increased urinary output. Dyspnea upon exertion is more indicating of left-sided failure. REF: p. 636

A 65-year-old male is diagnosed with chronic pulmonary disease and elevated pulmonary vascular resistance. Which of the following heart failures should the nurse assess for in this patient? a. Right heart b. Left heart c. Low-output d. High-output

ANS: A Right-sided failure occurs when the patient experiences chronic pulmonary disease and elevated pulmonary vascular resistance because the blood has difficulty overcoming the pressure and blood builds up in the right side of the heart. Pulmonary congestion leads to right-sided failure, not left, not low-output failure, and not high-output failure. REF: p. 636

A 28-year-old presents to the ER reporting severe chest pain that worsens with respirations or lying down. Other signs include a fever, tachycardia, and a friction rub. Assessment findings support which medical diagnosis? a. Acute pericarditis b. Myocardial infarction (MI) c. Stable angina d. Pericardial effusion

ANS: A Severe chest pain that worsens with respirations or lying down in a patient with fever, tachycardia, and a friction rub is characteristic of acute pericarditis. MI pain does not worsen with respirations. Stable angina does not worsen with respiration or lying down. Pericardial effusion is not manifested by these symptoms. REF: p. 622

What is the nurse monitoring when observing the QRS complex on the electrocardiogram? a. Ventricular activity b. Pulmonary artery closure c. Mitral valve opening d. Aortic valve closing

ANS: A The QRS complex represents the sum of all ventricular muscle cell depolarizations, not the closure of the pulmonary artery. The QRS complex does not reflect activity of either the mitral or aortic valves. REF: p. 577

Which structures act as anchors for the atrioventricular valves? a. Chordae tendineae b. Great vessels c. Coronary ostia d. Trabeculae carneae

ANS: A The atrioventricular valve openings are attached to the papillary muscles by the chordae tendineae. The great vessels are the vessels that bring blood to and out of the heart and are not attached to the chordae tendineae. The coronary ostia are openings in the aorta for the coronary arteries. The trabeculae carneae are a portion of the myocardium. REF: p. 572

A 50-year-old was prescribed a drug that acts as a negative inotrope. Which endogenous substances would be most similar? a. Acetylcholine b. Dopamine c. Epinephrine d. Thyroid hormone

ANS: A The most important negative inotropic agent is acetylcholine. Dopamine, epinephrine, and thyroid hormone are positive inotropes. REF: p. 583

A 60-year-old male with a 30-year history of smoking is diagnosed with a hormone-secreting lung tumor. Further testing indicates that the tumor secretes ADH. Which of the following assessment findings should the nurse expect? (select all that apply) a. Confusion b. Weakness c. Nausea d. Muscle twitching e. Increased reflexes

ANS: A, B, C, D Secretion of ADH leads to water intoxication with symptoms of cerebral edema, including confusion, convulsions, weakness, nausea, and muscle twitching. Depressed reflexes are associated with water intoxication. REF: p. 119 | p. 121

The nurse is assessing a patient with a diagnosis of inflammation. The nurse would expect to find which of the following signs and symptoms consistent with acute inflammation? (select all that apply) a. Heat b. Erythema c. Pain d. Swelling e. Paleness f. Loss of function

ANS: A, B, C, D, F The classic symptoms of acute inflammation include redness (erythema), heat, swelling, pain, and loss of function. Paleness would indicate poor circulation, not inflammation. REF: p. 137

A nurse is assessing a patient with hypoparathyroidism. Clinical manifestations of hypoparathyroidism include: (select all that apply) a. tetany. b. Chvostek sign. c. Trousseau sign. d. oily skin. e. hair loss.

ANS: A, B, C, E Symptoms of hypoparathyroidism include tetany, Chvostek and Trousseau signs, dry (not oily) skin, and loss of body and scalp hair. REF: p. 471

When a patient asks about secondary lymph organs, how should the nurse respond? Secondary lymph organs include: (select all that apply) a. the spleen. b. Peyer patches. c. adenoids. d. the liver. e. bone marrow. f. the appendix.

ANS: A, B, C, F The secondary lymphoid organs include the spleen, lymph nodes, adenoids, tonsils, Peyer patches (intestines), and appendix. The liver and bone marrow are not secondary lymph organs. REF: p. 161, Figure 7-3

Elevated levels of glucocorticoids result in which of the following assessment findings? (select all that apply) a. Polycythemia b. Increased appetite c. Weight loss d. Decreased calcium e. Increased height

ANS: A, B, D Increased glucocorticoid secretion leads to polycythemia, increased appetite, fat deposition in the face and cervical areas, decreased serum calcium levels, and interference with the action of growth hormone so that somatic growth is inhibited. Weight gain and loss of height are expected. REF: p. 453

A 67-year-old was previously diagnosed with rheumatic heart disease. Tests now reveal lipoprotein deposition with chronic inflammation that impairs blood flow from the left ventricle into the aorta. Which diagnosis does this history support? a. Aortic regurgitation b. Aortic stenosis c. Mitral regurgitation d. Mitral stenosis

ANS: B Aortic stenosis would impair blood flow from the left ventricle to the aorta. Aortic regurgitation would allow blood to flow back into the left ventricle. Mitral regurgitation would allow blood to flow from the left ventricle to the left atrium. Mitral stenosis would impair blood flow from the left atrium to the left ventricle. REF: p. 626

A 19-year-old male presents to his primary care provider reporting restlessness, muscle cramping, and diarrhea. Lab tests reveal that he is hyperkalemic. Which of the following could have caused his condition? a. Primary hyperaldosteronism b. Acidosis c. Insulin secretion d. Diuretic use

ANS: B During acute acidosis, hydrogen ions accumulate in the ICF and potassium shifts out of the cell to the ECF, causing hyperkalemia. Primary hyperaldosteronism is associated with hypokalemia, not hyperkalemia. Insulin secretion helps reduce potassium levels in the cell; it does not cause hyperkalemia. Diuretics would cause hypokalemia, not hyperkalemia. REF: p. 122

A patient is diagnosed with a hormone-secreting tumor of the pancreas alpha cells. Which of the following would the nurse expect to be most likely increased in this patient? a. Amylin b. Glucagon c. Insulin d. Somatostatin

ANS: B Glucagon is produced by the alpha cells of the pancreas. Amylin and insulin are secreted by the beta cells. Somatostatin is produced by the delta cells. REF: p. 452

What is the purpose of the glycosylated hemoglobin (hemoglobin A1c) test? a. Measuring fasting glucose levels. b. Monitoring long-term serum glucose control. c. Detecting acute complications of diabetes. d. Checking for hyperlipidemia.

ANS: B Glycosylated hemoglobin refers to the permanent attachment of glucose to hemoglobin molecules and reflects the average plasma glucose exposure over the life of a red blood cell (approximately 120 days). Glycosylated hemoglobin does not measure fasting, but rather glucose control over time. Glycosylated hemoglobin does not identify complications but could provide data if the patient is at risk. Glycosylated hemoglobin does not check for hyperlipidemia. REF: p. 471

A low ventilation-perfusion ratio of the lung will result in: a. increased dead space. b. shunting. c. alveolar collapse. d. bronchoconstriction.

ANS: B Hypoxemia caused by inadequate ventilation of well-perfused areas of the lung is a form of mismatching called shunting, not bronchoconstriction. Inadequate ventilation of well-perfused areas of the lung is not referred to as dead space. Alveolar collapse is called atelectasis. REF: p. 690

The predominant antibody of a typical primary immune response is: a. IgG. b. IgM. c. IgA. d. IgE.

ANS: B IgM is the largest immunoglobulin and is the first antibody produced during the initial, or primary, response to antigen. REF: p. 161

An 82-year-old female was admitted to the hospital with confusion and severe hypotension. Her body's compensatory mechanisms are increased heart rate, vasoconstriction, and movement of large volumes of interstitial fluid to the vascular compartment. What kind of shock does the nurse suspect the patient is experiencing? a. Anaphylactic b. Hypovolemic c. Neurogenic d. Septic

ANS: B In hypovolemic shock, heart rate and SVR increase, boosting both cardiac output and tissue perfusion pressures. Interstitial fluid moves into the vascular compartment. In anaphylactic shock, bronchoconstriction occurs with hypotension. In neurogenic shock, hypotension occurs, but fluid does not shift. In septic shock, interstitial fluid shift does not occur. REF: p. 642

Which of the following lab values would the nurse expect in a patient who has sustained trauma to the lungs and chest wall and is experiencing respiratory failure? a. Electrolyte imbalances b. Elevated PaCO2 c. Low hematocrit d. Elevated pH

ANS: B In respiratory failure, inadequate gas exchange occurs such that PaO2 ≤ 60 mm Hg or PaCO2 ≥ 50 mm Hg with pH ≤ 7.25. Electrolyte imbalances do not occur, but changes in blood gas values do. Hematocrit may be unaffected. pH will be decreased. REF: p. 690

Insulin is primarily regulated by: a. metabolic rate. b. serum glucose levels. c. prostaglandins. d. enzyme activation.

ANS: B Insulin secretion is promoted when blood levels of glucose rise. Insulin secretion is not only based on metabolic rate, prostaglandins, or enzyme activation but also on blood levels of glucose. REF: p. 451

A patient has been searching on the Internet about natriuretic hormones. When the patient asks the nurse what these hormones do, how should the nurse respond? Natriuretic hormones affect the balance of: a. calcium. b. sodium. c. magnesium. d. potassium.

ANS: B Natriuretic hormones are sometimes called a "third factor" in sodium regulation but have no influence on calcium, magnesium, or potassium balance. REF: p. 117

Which organ contains prolactin-inhibiting factor's target tissue? a. Hypothalamus b. Anterior pituitary c. Mammary glands d. Posterior pituitary

ANS: B Prolactin-inhibiting factor (PIF) inhibits prolactin secretion by the anterior pituitary, not the hypothalamus, mammary glands, or posterior pituitary. REF: p. 444, Table18- 4

A 25-year-old male is in a car accident and sustains a fracture to his left femur with extensive soft tissue injury. The pain associated with the injury is related to: a. histamine. b. prostaglandins. c. vasoconstriction. d. immune complex formation.

ANS: B Prostaglandins cause increased vascular permeability, neutrophil chemotaxis, and pain by direct effects on nerves. Histamine promotes vasodilation. Prostaglandins are not associated with vasoconstriction or the immune complex formation. REF: p. 145

When considering pyelonephritis, where is the site of the infection? a. Bladder b. Renal pelvis c. Renal tubules d. Glomerulus

ANS: B Pyelonephritis is an infection of one or both upper urinary tracts (ureter, renal pelvis, and interstitium). Pyelonephritis is not associated with the bladder, renal tubules, or glomerulus. REF: p. 754

A 75-year-old reports loss of urine with cough, sneezing, or laughing. These symptoms support which diagnosis? a. Urge incontinence b. Stress incontinence c. Overflow incontinence d. Functional incontinence

ANS: B Reduced resistance is associated with the symptom of stress incontinence, which is incontinence with coughing or sneezing. Urge incontinence is the inability to hold the urine when the urge is felt. There is coordination between the contracting bladder and the external sphincter, but the detrusor is too weak to empty the bladder, resulting in urinary retention with overflow or stress incontinence. Functional incontinence is similar to overflow and is not associated with coughing. REF: p. 750, Table 30-1

A 30-year-old diagnosed with Graves disease is admitted to a hospital unit for the surgical removal of the thyroid gland. During the postoperative period, the nurse notes that the patient's serum calcium is low. The nurse should observe the patient for which of the following signs/symptoms? a. Muscle weakness and constipation b. Laryngeal spasms and hyperreflexia c. Abdominal pain and fever d. Anorexia, nausea, and vomiting

ANS: B Symptoms of low calcium are associated with tetany, a condition characterized by muscle spasms, hyperreflexia, tonic-clonic convulsions, and laryngeal spasms. Symptoms of low calcium are not associated with muscle weakness, constipation, abdominal pain, anorexia, nausea, or vomiting. testbanks_and_xanax REF: p. 471

While checking the lab results for a patient diagnosed with Graves disease, the nurse would expect the T3 level to be abnormally: a. low. b. high. c. variable. d. absent.

ANS: B T3 levels are elevated in Graves disease. REF: p. 467

A 50-year-old received trauma to the chest that caused severe impairment of the primary pacemaker cells of the heart. Which of the following areas received the greatest damage? a. Atrioventricular (AV) node b. Sinoatrial (SA) node c. Bundle of His d. Ventricles

ANS: B The SA node, not the AV node, is considered the pacemaker of the heart. Neither the bundle of His nor the ventricles are involved. REF: p. 576

If a patient's posterior pituitary is removed, which hormone would the nurse expect to decrease? a. PRF b. ADH c. ACTH d. Growth hormone (GH)

ANS: B The hormones ADH and oxytocin are released from the posterior pituitary gland. PRF is released by the hypothalamus. ACTH is released by the anterior pituitary. GH is released by the hypothalamus. REF: p. 448

A 10-year-old male presents with fever, lymphadenopathy, arthralgia, and nosebleeds and is diagnosed with rheumatic heart disease. While planning care, which characteristic changes should the nurse remember? a. Blood-borne organisms that adhere to the valvular surface b. Antigens that bind to the valvular lining, triggering an autoimmune response c. High fevers that damage collagen in valve leaflets d. Rheumatoid factor in the blood, stimulating valvular degeneration

ANS: B The immune response cross-reacts with molecularly similar self-antigens in heart, muscle, joints, and the brain, causing an autoimmune response resulting in diffuse, proliferative, and exudative inflammatory lesions in these tissues. It is not due to blood-borne organisms, high fevers, or rheumatoid factors. REF: p. 628

A 45-year-old presents with hypertension, anorexia, nausea and vomiting, and anemia and is diagnosed with chronic renal failure. What is the cause of this patient's anemia? a. Red blood cells being lost in the urine b. Inadequate production of erythropoietin c. Inadequate iron absorption in the gut d. Red blood cells being injured as they pass through the glomerulus

ANS: B The kidneys are inadequate in their production of erythropoietin; red cells may be lost in the urine, but it is the lack of erythropoietin that leads to anemia. The anemia is not associated with inadequate iron absorption or red cell destruction. REF: p. 764, Table 30-13

When considering hypothyroidism, the basal metabolic rate is unusually: a. high. testbanks_and_xanax b. low. c. steady. d. variable.

ANS: B The metabolic rate with hypothyroidism is low. REF: p. 469

What is the most common cause of elevated levels of antidiuretic hormone (ADH) secretion? a. Autoimmune disease b. Cancer c. Pregnancy d. Heart failure

ANS: B The most common cause of elevated levels of ADH is cancer, not autoimmune disorders, pregnancy, or heart failure. REF: p. 461

A 50-year-old is diagnosed with lung cancer. The health history includes a 30-year history of smoking, exposure to air pollution, asbestos, and radiation. What had the greatest impact on the development of his cancer? a. Radiation b. Cigarette smoke c. Asbestos d. Air pollution

ANS: B The most common cause of lung cancer is tobacco smoking. While considered risk factors, neither exposure to radiation, asbestos, nor air pollution is the most common of the known triggers. REF: p. 709

The most common cause of pulmonary edema is: a. right heart failure. b. left heart failure. c. asthma. d. lung cancer.

ANS: B The most common cause of pulmonary edema is left-sided heart disease, not right-sided. Pulmonary edema is not commonly caused by asthma or lung cancer. REF: p. 695

Which organism does the nurse suspect is the most likely cause of empyema? a. Virus b. Staphylococcus aureus c. Fungus d. Moraxella catarrhalis

ANS: B The most likely cause of empyema is Staphylococcus aureus; it is not due to a virus. Empyema is not due to Moraxella catarrhalis or to a fungal infection. REF: p. 692

The onset of anaphylactic shock is usually: a. mild. b. sudden and life threatening. c. delayed by several hours. d. delayed by 24 hours.

ANS: B The onset of anaphylactic shock is usually sudden and life threatening, not mild or delayed. REF: p. 644

A 20-year-old underwent an echocardiogram to assess chest pain. Results revealed a congenital defect in the papillary muscles. Which of the following would the nurse expect to occur? a. Closure of the semilunar valve b. Backward expulsion of the atrioventricular valves c. Closure of the atrioventricular valve d. Backward expulsion of the semilunar valves

ANS: B The papillary muscles are extensions of the myocardium that pull the cusps together and downward at the onset of ventricular contraction, thus preventing their backward expulsion into the atria. Defects in the papillary muscles would not affect either the semilunar or atrioventricular valves. REF: p. 572

What causes the chronic microvascular and macrovascular complications of DM? a. Pancreatic changes b. Hyperglycemia c. Ketone toxicity d. Hyperinsulinemia

ANS: B The underlying cause of the microvascular and macrovascular diseases is related to hyperglycemia, not pancreatic changes, ketone toxicity, or hyperinsulinemia. REF: p. 478

A 50-year-old presents with hypotension, hypoxemia, and tracheal deviation to the left. Tests reveal that the air pressure in the pleural cavity exceeds barometric pressure in the atmosphere. Based upon these assessment findings, what does the nurse suspect the patient is experiencing? a. Pleural effusion b. Tension pneumothorax c. Open pneumothorax d. Transudative pneumothorax

ANS: B Tracheal deviation suggests tension pneumothorax, not pleural effusion. With an open pneumothorax, a sucking sound would be heard without tracheal deviation. Tracheal deviation suggests a tension, not transudative pneumothorax. REF: p. 692

A 35-year-old presents with pulmonary hypertension and is diagnosed as being in right heart failure. Which is the most likely cause of this condition? a. Aortic stenosis b. Tricuspid regurgitation c. Aortic regurgitation d. Mitral regurgitation

ANS: B Tricuspid regurgitation leads to volume overload in the right atrium and ventricle, increased systemic venous blood pressure, and right heart failure. Aortic stenosis is manifested by narrowed pulse pressure. Aortic regurgitation is manifested by widened pulse pressure resulting from increased stroke volume and diastolic backflow. Mitral regurgitation is manifested by heart failure but not pulmonary hypertension. REF: p. 627

A 55-year-old presents reporting urinary retention. Tests reveal a lower urinary tract obstruction. Which of the following is of most concern to the nurse? a. Vesicoureteral reflux and pyelonephritis b. Formation of renal calculi c. Glomerulonephritis d. Increased bladder compliance

ANS: B Urine stasis occurs with urinary tract obstruction and can lead to the formation of renal calculi. Reflux and pyelonephritis would not lead to calculi as much as stasis would. Neither glomerulonephritis nor increased bladder compliance causes the stasis that would lead to calculus formation. REF: p. 748

Visual disturbances are a common occurrence in patients with untreated Graves disease. The endocrinologist explains to the patient that the main cause of these complications is: a. decreased blood flow to the eye. b. orbital edema and protrusion of the eyeball. c. TSH neurotoxicity to retinal cells. d. local lactic acidosis.

ANS: B Visual disturbances with Graves disease include orbital fat accumulation, inflammation, and edema of the orbital contents resulting in exophthalmos (protrusion of the eyeball), periorbital edema, and extraocular muscle weakness leading to diplopia (double vision). Blood flow to the eye is not an effect, but visual changes occur. Functional abilities of the eye result from hyperactivity of the sympathetic system. Lactic acid is not involved with visual changes in the eye. REF: p. 468

Which is a characteristic of idiopathic pulmonary arterial hypertension (IPAH)? (select all that apply) a. Male gender b. Fatigue c. Dyspnea d. Jugular vein distention e. Weight gain

ANS: B, C, D IPAH is a rare condition that usually occurs in women between the ages of 20 and 40. Manifestations of fatigue, chest discomfort, tachypnea, and dyspnea (particularly with exercise) are common. Examination may reveal peripheral edema, jugular venous distention, a precordial heave, and accentuation of the pulmonary component of the second heart sound. REF: p. 708

A 25-year-old female experiences a headache and takes aspirin for relief. A nurse recalls aspirin relieves the headache by: a. decreasing leukotriene production. b. increasing histamine release. c. decreasing prostaglandin production. d. increasing platelet-activating factor.

ANS: C Aspirin is a prostaglandin inhibitor. Aspirin is not associated with leukotriene production, histamine release, or platelet-activating factor. REF: p. 145

An older adult is diagnosed with cerebral aneurysm. Where does the nurse suspect the cerebral aneurysm is located? a. Vertebral arteries b. Basilar artery c. Circle of Willis d. Carotid arteries

ANS: C Cerebral aneurysms often occur in the circle of Willis. Such an aneurysm is not associated with the vertebral arteries, the basilar artery, or the carotid arteries. REF: p. 605

What is the most common cause of hypoparathyroidism? a. Pituitary hyposecretion b. Parathyroid adenoma c. Parathyroid gland injury d. Hypothalamic inactivity

ANS: C Hypoparathyroidism is most commonly caused by damage to the parathyroid glands, not pituitary hyposecretion, parathyroid adenoma, or inactivity of the hypothalamus. REF: p. 470

Many valvular stenosis and regurgitation disorders in adults have a common etiology. Which of the following conditions should alert the nurse that the patient may have both types of valve dysfunctions? a. Heart failure b. Connective tissue disorders c. Rheumatic fever or heart disease d. Syphilis infection

ANS: C Valvular dysfunction is often related to rheumatic fever. Heart failure decreases cardiac output but does not affect valvular function. It is not due to connective tissue disorders. Syphilis infection could affect valves, but it is not the most common cause. REF: p. 625

What type of effect occurs when insulin binds to its receptors on muscle cells, resulting in an increase in glucose uptake by those muscle cells? a. Pharmacologic b. Permissive c. Biphasic d. Direct

ANS: D Direct effects are the obvious changes in cell function that result specifically from stimulation by a particular hormone as is true with insulin. Pharmacologic effects are the result of high doses of a drug. Permissive effects are less obvious hormone-induced changes that facilitate the maximal response or functioning of a cell. Biphasic effects are twofold effects. REF: p. 441

Which factor will help the nurse differentiate leukotrienes from histamine? a. Site of production b. Vascular effect c. Chemotactic ability d. Time of release

ANS: D Leukotrienes are released slower and longer than histamine. Both leukotrienes and histamine are produced by mast cells, and they have similar vascular effects and chemotactic abilities. REF: p. 145

Which statement indicates the nurse understands coronary ostia? The coronary ostia (the openings to the coronary arteries) are found in the: a. left ventricle. b. inferior vena cava. c. coronary sinus. d. aorta.

ANS: D The ostia are found on the aorta, not the ventricle, the vena cava, or the coronary sinus. REF: p. 573

A 60-year-old female has survived a myocardial infarction. The nurse is providing care for impaired ventricular function because: a. there is a temporary alteration in electrolyte balance. b. there is too much stress on the heart. c. the cells become hypertrophic. d. the resulting ischemia leads to hypoxic injury and myocardial cell death.

ANS: D The patient has impaired ventricular functioning because a portion of the myocardium has died due to ischemia. Impaired ventricular function is due to damage to the myocardium; it is not due to electrolyte imbalance. There was stress on the heart, but the impaired functioning is due to myocardial damage secondary to ischemia. The impaired ventricular dysfunction is due to myocardial cell death, not hypertrophy. REF: p. 620

A 27-year-old male is admitted to a neurologic unit with a complete C-5 spinal cord transection. On initial assessment, he is bradycardic, hypotensive, and hyperventilating. He appears to be going into shock. The most likely mechanism of his shock is: a. hypovolemia caused by blood loss. b. hypovolemia caused by evaporative fluid losses. c. vasodilation caused by gram-negative bacterial infection. d. vasodilation caused by a decrease in sympathetic stimulation.

ANS: D The patient is experiencing neurogenic shock in which blood volume has not changed, but SVR decreases drastically so that the amount of space containing the blood has increased, leading to hypotension. In this type of shock, blood loss has not occurred. In this type of shock, fluid loss has not occurred. Vasodilation due to infection would be septic shock; the type of shock described in the patient is due to loss of sympathetic stimulation. REF: pp. 642-643

An older male presents with flank pain and polyuria. Tests reveal that he has an enlarged prostate. Which type of renal failure is this patient at risk for? a. Prerenal b. Intrarenal c. Extrarenal d. Postrenal

ANS: D The patient will experience postrenal renal failure due to obstruction by the prostate. REF: p. 762

A 35-year-old male weighs 70 kg. Approximately how much of this weight is considered the total volume of body water? a. 5 L b. 10 L c. 28 L d. 42 L

ANS: D The total volume of body water for a 70-kg person is about 42 L or two thirds of 70 kg. REF: p. 114

A 28-year-old with seizure disorder has a vagus nerve stimulator implanted to help control seizure activity. Which result will the nurse expect to occur? a. Increased speed of cardiac cycle b. Increased cardiac contractility c. Decreased vasodilation d. Decreased cardiac conduction

ANS: D The vagus nerve releases acetylcholine. Acetylcholine causes decreased heart rate and slows conduction through the AV node, thus decreasing the speed of the cardiac cycle. This reaction also decreases contractility but does not cause decreased vasodilation. REF: p. 576 | p. 578

A patient presents to the emergency department reporting difficulty swallowing and shortness of breath. A CT scan would most likely reveal an aneurysm in the: a. cerebral vessels. b. renal arteries. c. inferior vena cava. d. thoracic aorta.

ANS: D Thoracic aortic aneurysms can cause dysphagia (difficulty swallowing) and dyspnea (breathlessness). Aneurysms in cerebral vessels will produce a headache. Aneurysms in the renal arteries will produce flank pain. Aneurysms in the inferior vena cava may produce chest pain. REF: p. 605

leukopenia

Abnormally low white blood cell count

hypersecretion of growth hormone (GH) in adults

Acromegaly

hyposecretion of adrenal cortex hormones

Addison's disease

ANS: D The majority of gallstones are composed of cholesterol. The other options are not found in high quantities. REF: p. 935

An analysis of most gallstones would reveal a high concentration of: a. phosphate. b. bilirubin. c. urate. d. cholesterol.

ANS: B Excessive hemolysis (breakdown) of red blood cells can cause hemolytic jaundice (prehepatic jaundice). Red blood cell breakdown is not associated with the other forms of jaundice. REF: p. 930

An increase in the rate of red blood cell breakdown causes which form of jaundice? a. Obstructive b. Hemolytic c. Hepatocellular d. Metabolic

ANS: A Visceral pain arises from a stimulus (distention, inflammation, and ischemia) acting on an abdominal organ. Somatic is a form of parietal pain. Parietal pain, from the parietal peritoneum, is more localized and intense than visceral pain, which arises from the organs themselves. Referred pain is visceral pain felt at some distance from a diseased or affected organ. REF: p. 909

Assuming that midline epigastric pain is caused by a stimulus acting on an abdominal organ, the pain felt is classified as: a. visceral. b. somatic. c. parietal. d. referred.

Which of the following processes describes the mechanism underlying the sodium-potassium pump? A) Passive diffusion B) Active transport C) Facilitated diffusion D) Osmosis

B) Active transport

Which of the following is a typical characteristic of benign tumors? A) Cells in the tumor are undifferentiated. B) Benign tumors are surrounded by a capsule. C) Benign tumors have a tendency to invade surrounding tissue. D) Cells in the tumor occasionally metastasize to lymph nodes.

B) Benign tumors are surrounded by a capsule.

Which cells in the pancreas secrete insulin? A) Alpha B) Beta C) Delta D) Acinar

B) Beta Alpha cells secrete glucagon. Beta cells secrete insulin. Delta cells secrete somatostatin. Acinar cells secrete pancreatic digestive enzymes.

A 10-year-old male is diagnosed with a large tapeworm. Which of the following cells would be produced in response to this worm? A) Monocytes B) Eosinophils C) Neutrophils D) Natural killer cells

B) Eosinophils

Which hormone functions as an antagonist to insulin? A) Aldosterone B) Glucagon C) Somatostatin D) Amylin

B) Glucagon Aldosterone does not have any physiologic effects that are antagonistic to insulin. Glucagon increases blood sugar levels by stimulating glycogenolysis and gluconeogenesis. Although somatostatin can inhibit the secretion of insulin, the hormone that has the opposite effect of insulin is glucagon. Amylin is co-secreted with insulin to suppress glucagon secretion after a meal. Glucagon is insulin's antagonist.

Which of the following cancers arise from connective tissue? A) Lymphoma B) Sarcoma C) Glioma D) Adenoma

B) Sarcoma

Acromegaly is caused by increased secretion of: A) prolactin. B) growth hormone. C) insulin. D) glucocorticoids.

B) growth hormone. Acromegaly from increased growth hormone and insulin-like growth factor secretion results in gigantism as well as other structural and physiological problems.

Cells defend against viral invasion through the production and secretion of: A) histamine. B) interferon. C) growth factors. D) prostaglandins.

B) interferon.

Immune cells distinguish "self" from "nonself" by recognizing: A) cellular RNA. B) major histocompatibility antigens. C) different types of phospholipids in a cell's membrane. D) "nonself" enzymes secreted by foreign cells.

B) major histocompatibility antigens.

A patient has a temporary but total displacement of a bone from its normal position. Which of the following terms is appropriate to describe this injury? A. Fracture. B. Dislocation. C. Subluxation. D. Nonunion.

B. Dislocation.

A patient is brought to the hospital after twisting an ankle while running around the bases in a baseball game. On examination, it appears that the ligaments have sustained tears. Which is the best term to describe this injury? A. Strain. B. Sprain. C. Avulsion. D. Reduction.

B. Sprain.

A 51-year-old male is at the health clinic for an annual physical exam. After walking from the car to the clinic, he developed substernal pain. He also reported discomfort in his left shoulder and his jaw, lasting 2 to 3 minutes and then subsiding with rest. He indicates that this has occurred frequently over the past few months with similar exertion. The nurse suspects he is most likely experiencing: A. Unstable angina B. Stable angina C. Prinzmetal angina D. Myocardial infarction (MI)

B. Stable angina

What factors make a patient prone to neurogenic shock? Neurogenic shock can be caused by any factor that inhibits the: A. Parasympathetic nervous system B. Sympathetic nervous system C. Somatic nervous system D. Thalamus

B. Sympathetic nervous system

Which information by the nurse indicates a good understanding of depolarization? Depolarization of a cardiac muscle cell occurs as the result of: A. A decrease in the permeability of the cell membrane to ions B. The rapid movement of ions across the cell membrane C. A blockade by calcium ions D. Stimuli instigated during the refractory period

B. The rapid movement of ions across the cell membrane

A newborn is in respiratory distress and requires ventilation. Tests reveal that he does not produce surfactant due to the absence of: A. Mucus-producing cells B. Type II alveolar cells C. Alveolar macrophages D. Goblet cells

B. Type II alveolar cells

Which principle should the nurse remember when planning nursing care for a patient with heart problems? As stated by the Frank-Starling law, there is a direct relationship between the _____ of the blood in the heart at the end of diastole and the _____ of contraction during the next systole. A. Pressure, duration B. Volume, force C. Viscosity, force D. Viscosity, duration

B. Volume, force

A 13-year-old female fell on her knee while skating. The area became inflamed and sore to touch. Which of the following would result from activation of the plasma system secondary to her injury? A) Mast cell degranulation B) Release of cellular products C) Cellular infiltration D) Pathogenic invasion

C) Cellular infiltration

Which of the following is a steroid hormone? A) Insulin B) Thyroxine (T4) C) Cortisol D) Growth hormone

C) Cortisol

Which of the following risk factors is associated with the development of lung, bladder, pancreatic, renal, laryngeal, pharyngeal, and esophageal cancer? A) Ultraviolet (UV) radiation B) Radon C) Tobacco use D) Asbestos

C) Tobacco use

A group of mountain climbers experience confusion, tachycardia, edema, and decreased renal output after climbing Mount Rainier. A nurse recalls this condition is caused by: A. Bronchoconstriction B. Hypoventilation C. Decreased inspired oxygen D. Diffusion abnormalities

C. Decreased inspired oxygen

An aide asks a nurse how surfactant works. How should the nurse respond? Surfactant facilitates alveolar distention and ventilation by: A. Decreasing thoracic compliance B. Attracting water to the alveolar surface C. Decreasing surface tension in alveoli D. Increasing diffusion in alveoli

C. Decreasing surface tension in alveoli

Anemia accompanies chronic renal failure because of: A. Blood loss via the urine B. Renal insensitivity to vitamin D C. Inadequate production of erythropoietin D. Inadequate retention of serum iron

C. Inadequate production of erythropoietin

When the nurse is asked what causes asthma, how should the nurse respond? Asthma is thought to be caused by: A. An autosomal recessive trait B. Excessive use of antibiotics as a young child C. Interactions between genetic and environmental factors D. Autoimmunity

C. Interactions between genetic and environmental factors

The nurse is reviewing a normal electrocardiogram. The nurse assesses the PR interval because it represents: A. Atrial depolarization B. Ventricular depolarization C. Onset of atrial activation to onset of ventricular activity D. "Electrical systole" of the ventricles

C. Onset of atrial activation to onset of ventricular activity

The nurse is describing the receptors in the lung that decrease ventilatory rate and volume when stimulated. Which receptors is the nurse discussing? A. Carbon dioxide receptors B. Baroreceptors C. Stretch receptors D. Chemoreceptors

C. Stretch receptors

Membrane Transport

Cellular intake and output ØCells continually take in nutrients, fluids, and chemical messengers from the extracellular environment and expel metabolites, or the products of metabolism, and end products of lysosomal digestion

___ is inversely related to ___

Chloride Bicarbonate

Osmotic demyelination syndrome

Complication of aggressive correction of HYPOnatremia. Axonal demyelination of pontine white matter. "From low to high, your pons will die."

scar tissue

Composed primarily of collagen to restore the tensile strength of the tissue but cant function the same

Water excess

Compulsive water drinking Decreased urine formation Syndrome of inappropriate ADH (SIADH)

ANS: D These deficiencies lead to defective erythrocytes that die prematurely, which decreases their numbers in the circulation, causing anemia. These deficiencies do not cause the RBCs to lack differentiation, contain malformed hemoglobin, or experience diminished O2-carrying capacity. REF: p. 515

Considering anemia, what effect do vitamin B12 and folate deficiencies have on red blood cells? a. They are unable to differentiate into erythrocytes. b. They contain malformed hemoglobin molecules. c. O2-carrying capacity is decreased. d. Their life span is shorter.

Which of the following cancers arise from epithelial tissue? A) Sarcoma B) Leukemia C) Glioma D) Carcinoma

D) Carcinoma

Which of the following would increase a patient's risk for thrombotic stroke? A. Hyperthyroidism B. Hypertension C. Anemia D. Dehydration

D. Dehydration

hyposecretion of antidiuretic hormone (ADH)

Diabetes insipidus

ANS: C Loss of temperature control occurs during REM sleep. Loss of temperature control is not associated with non-REM sleep, light sleep, or delta wave sleep. REF: p. 345

During the sleep cycle, when does loss of temperature control occur? a. Non-REM sleep b. Light sleep c. REM sleep d. Delta wave sleep

hemorrhagic exudate

Exudate contains blood: indicates bleeding

ANS: C Hypersplenism results in sequestering, not proliferation, activation, or infection, of the blood cells. REF: pp. 539-540

For a patient experiencing hypersplenism, the nurse expects the erythrocytes to be: a. proliferated. b. activated. c. sequestered. d. infected.

Adenosine Triphosphate

Fuel for cell survival Created from the chemical energy contained within organic molecules Used in synthesis of organic molecules, muscle contraction, and active transport of molecules across cell membranes Stores and transfers energy

hypersecretion of thyroid hormone (TH)

Graves disease

Cellular energy step 2: Glycolysis

Happens in the cell, splitting of glucose, oxygen isnt needed, 2 ATP is produced.

Cellular energy step 1: Digestion

Happens in the gut, breakdown of proteins to amino acids, fat to fatty acids and polysaccarides to monosaccharide.

ANS: B Convection occurs by transfer of heat through currents of gases or liquids, exchanging warmer air at the body's surface with cooler air in surrounding spaces. Convection does not involve electromagnetic waves, bringing blood to skin surfaces, or molecule-to-molecule transfer. REF: p. 343, Table 14-5

Heat loss from the body via convection occurs by: a. evaporation of electromagnetic waves. b. transfer of heat through currents of liquids or gas. c. dilation of blood vessels bringing blood to skin surfaces. d. direct heat loss from molecule-to-molecule transfer.

A nurse is asked about the role of plasmin in the inflammatory response cycle. What is the nurse's best reply?

It controls clotting by breaking down fibrin. Plasmin is an enzyme that will degrade fibrin during clot dissolution or at the resolution of inflammation.

ANS: D Individuals with sideroblastic anemia may show signs of iron overload (hemosiderosis), including mild to moderate enlargement of the liver (hepatomegaly) and spleen (splenomegaly). High levels of iron indicate sideroblastic anemia, not pernicious anemia, folate deficiency, or iron deficiency anemia. REF: p. 519

Lab results showing high iron, bilirubin, and transferrin and low hemoglobin and hematocrit would support a diagnosis of which form of anemia? a. Pernicious b. Folate deficiency c. Iron deficiency d. Sideroblastic

ANS: B Leukocytosis is present when the count is higher than normal. It is not another term for leukopenia. REF: p. 523

Leukocytosis can be defined as: a. a normal leukocyte count. b. a high leukocyte count. c. a low leukocyte count. d. another term for leukopenia.

Cellular Receptors

Ligands ØBind with cellular receptors to activate or inhibits the receptor's associated signaling or biochemical pathway Plasma membrane receptors ØDetermine response to binding

Cellular Metabolism

Metabolism ØChemical tasks of maintaining essential cellular functions ØAnabolism •Energy using (to build more complex structures) ØCatabolism •Energy releasing (by destruction of complex substances to simple complexes (Co2 and water are released))

ANS: C Dreams occur during REM sleep. Dreams are not associated with non-REM, alpha wave, or delta wave sleep. REF: p. 345

Most memorable dreams occur during which sleep phase? a. Non-REM b. Alpha wave c. REM d. Delta wave

Reabsorption

Movement of fluid out of the interstitial space into the capillary bed

Filtration

Movement of water and solutes because of a greater force on one side than another(hydrostatic pressure and blood pressure)

______ is a major cation in ECF.

Na

Hypophosphatemia S/S

OSTEOMALACIA (SOFT BONES) muscle weakness anemia platelet impairment leukocyte alterations antacids bind phosphate

ANS: D Myasthenia gravis is a disorder resulting from autoimmune injury at the neuromuscular junction, not from a viral infection. It is not due to motor neuron atrophy, but a lack of acetylcholine. Multiple sclerosis (MS) is due to demyelination of skeletal motor neurons. REF: p. 413

Patient teaching is considered successful regarding myasthenia gravis when the patient identifies its cause as being: a. viral infection of skeletal muscle. b. atrophy of motor neurons in the spinal cord. c. demyelination of skeletal motor neurons. d. autoimmune injury at the neuromuscular junction.

ANS: C Patients diagnosed with myasthenia gravis experience tumors in the thymus, not the brain, the pancreas, or the lungs. REF: p. 413

Patients diagnosed with myasthenia gravis often have tumors or pathologic changes in the: a. brain. b. pancreas. c. thymus. d. lungs.

Cellular Communication

Plasma membrane-bound receptors Intracellular receptors Gap junctions (contact signaling) Chemical signaling ØParacrine ØAutocrine ØHormonal ØNeurohormonal Neurotransmitters

Sodium

Primary ECF cation, Regulated osmotic forces(water)

If a person comes into a hospital with hyperkalamia, what could be going wrong inside the body?

Renal failure

A low ventilation-perfusion ratio results in:

Shunting

ANS: B Spinal shock is the complete cessation of spinal cord function below the lesion. The reflexes are not preserved in spinal shock. Spinal shock is the complete cessation of spinal cord function below the lesion, not at the lesion only. REF: p. 382

Spinal shock is characterized by: a. loss of voluntary motor function with preservation of reflexes. b. cessation of spinal cord function below the lesion. c. loss of spinal cord function at the level of the lesion only. d. temporary loss of spinal cord function above the lesion.

Cellular Energy

Step 1-Digestion (happens in the gut) ØExtracellular breakdown of proteins, fats, polysaccarides to subunits Step 2- Glycolysis (happens in the cell) ØSplitting of glucose ØIntracellular breakdown of subunits to pyruvate, then to acetyl CoA ØOxygen is not needed (Anaerobic) ØOnly 2 ATP produced Step 3 - Citric acid cycle ØAlso called Krebs cycle or the tricarboxylic acid cycle (TCA) ØMuch ATP produced via oxidative phosphorylation if oxygen present ØWaste products excreted (water, urea, and CO2)

Nonvolatile

Sulfuric, phosphoric, and other organic acids Eliminated by the renal tubules with the regulation of HCO3-

ANS: C The pylorus is the opening between the esophagus and the duodenum; the obstruction is pyloric. Ileocecal obstruction is in the small intestine. Hiatal hernia is related to the esophagus. Hiatal obstruction is related to the esophagus. REF: p. 912

Tests reveal narrowing of the opening between the stomach and the duodenum. This condition is referred to as: a. ileocecal obstruction. b. hiatal hernia. c. pyloric obstruction. d. hiatal obstruction.

fibrinous exudate

Thick, clotted exudate: indicates more advanced inflammation

ANS: C The condition may also be either congenital or acquired and may be either primary or secondary to other conditions. It is not referred to as transient or consistent, normal or abnormal, or active or inactive. REF: p. 541

Thrombocytopenia may be: a. transient or consistent. b. normal or abnormal. c. congenital or acquired. d. active or inactive.

ANS: B Saccular aneurysms (berry aneurysms) occur frequently (in approximately 2% of the population) and likely result from congenital abnormalities in the media of the arterial wall and degenerative changes. Fusiform aneurysms (giant aneurysms) occur as a result of diffuse arteriosclerotic changes and are found most commonly in the basilar arteries or terminal portions of the internal carotid arteries. Arteriovenous malformation (AVM) is a tangled mass of dilated blood vessels creating abnormal channels between the arterial and venous systems. Thrombotic stroke would show signs of necrotic tissue, not degenerative changes. REF: p. 404

Upon autopsy of a 25-year-old, abnormalities in the media of the arterial wall and degenerative changes were detected. Which of the following would most likely accompany this finding? a. Fusiform aneurysm b. Saccular aneurysm c. Arteriovenous malformation d. Thrombotic stroke

Movement of Body Fluids and Electrolytes

Water and electrolytes move across cell membranes via multiple passive (no energy required) and active (energy required) mechanisms

serous exudate

Watery exudate: indicates early inflammation

ANS: C Astrocytomas are the most common primary tumors in the CNS, accounting for over 50%. Meningioma tumors usually originate from the arachnoidal (meningeal) cap cells in the dura mater and account for 30% of tumors. Oligodendrogliomas account for about 2% of tumors. Ependymomas are more common in children. REF: p. 416

What are the most common primary central nervous system (CNS) tumors in adults? a. Meningiomas b. Oligodendrogliomas c. Astrocytomas d. Ependymomas

ANS: B Peptic ulcer disease is caused by breaks in the mucosa and the presence of corrosive substances. High gastrin occurs, but the disease is due to breaks in the mucosa. Vagal activity increases. Gastric erosions occur, but they are not due to high ammonia. REF: pp. 915-916

What is the cause of peptic ulcer disease? a. Hereditary hormonal imbalances with high gastrin levels b. Breaks in the mucosa and presence of corrosive secretions c. Decreased vagal activity and vascular engorgement d. Gastric erosions related to high ammonia levels and bile reflux

ANS: A The presence of telomere caps gives cancer cells the ability to divide over and over again, thus cancer cells have unlimited mitosis. Telomere caps do not give cells clonal distinction. Mutation capability is a characteristic of cancer cells, but this property is not related to telomeres. REF: p. 245

What is the effect of telomere caps on cancer cells? a. Repeated divisions b. Clonal distinction c. Limited mitosis d. Mutation abilities

ANS: A An artery is the source of bleeding in 85% of extradural hematomas. The bleeding associated with an extradural hematoma is not a result of damage to a vein, a capillary, or a sinus. REF: p. 391

What is the main source of bleeding in extradural (epidural) hematomas? a. Arterial b. Venous c. Capillary d. Sinus

ANS: D Early manifestations include low-grade fever, headache (most common symptom), nausea and vomiting, neck pain and stiffness, confusion, drowsiness, sensory deficits, and communication deficits. REF: p. 409

What is the most common early symptom of a brain abscess? a. Neck rigidity b. Vomiting c. Drowsiness d. Headache

ANS: A Parasitic invasions and allergic disorders are associated with eosinophilia. Viral infections do not lead to eosinophilia nor do stress and anxiety reactions or fungal infections. REF: p. 523

What is the most likely cause of the eosinophilia? a. Parasitic invasion and allergic reactions b. Viral and bacterial infections c. Stress and anxiety reactions d. Fungal infections and delayed hypersensitivity

ANS: B Hypothermia (marked cooling of core temperature) produces depression of the central nervous and respiratory systems, vasoconstriction, alterations in microcirculation, coagulation, and ischemic tissue damage. Hypothermia does not lead to increased respirations, CNS excitation, or increased cellular metabolism. REF: p. 344

What is the physiological response when the body's core temperature is altered due to prolonged exposure to a cold environment? a. Increased respirations b. Ischemic tissue damage c. CNS excitation d. Increased cellular metabolism

ANS: B The primary clinical manifestation of stress-related mucosal disease is bleeding, which is uncommon, but occurs more readily with the presence of coagulopathy and more than 48 hours of mechanical ventilation. None of the other options is associated with stress ulcers. REF: p. 918

What is the primary clinical manifestation of a stress ulcer? a. Bowel obstruction b. Bleeding c. Pulmonary embolism d. Hepatomegaly

ANS: B The pain threshold in children is lower than or the same as that of adults. REF: p. 338, Table 14-2

When planning care for a child in pain, which principle should the nurse remember? The pain threshold in children is _____ that of adults. a. higher than b. more variable c. the same as d. not related to

ANS: A Color blindness, present most often in males, affects 8% of the male population and 0.5% of the female population. Neither the elderly nor children are most prone to color blindness. REF: p. 349

Which group of people is most prone to color blindness? a. Males b. Females c. Elderly persons d. Children

ANS: C With removal of the stomach, the intrinsic factor is also removed, leading to an inability to absorb B12; thus, the person with removal of the stomach is at greatest risk. Being a fussy eater, being pregnant, and having a history of duodenal ulcers are not risk factors for vitamin B12 deficiency anemia. REF: p. 516

Which individual should the nurse assess initially for a vitamin B12 deficiency anemia? a. A 3-year-old female who is a fussy eater b. A 26-year-old female in the second trimester of her first pregnancy c. A 47-year-old male who had a gastrectomy procedure d. A 64-year-old male with a history of duodenal ulcers

ANS: C Malignant tumors have cells that vary in both size and shape, and they grow rapidly. They are poorly differentiated and not encapsulated. REF: p. 234

Which information indicates a nurse understands characteristics of malignant tumors? a. Grows slowly b. Has a well-defined capsule c. Cells vary in size and shape d. Is well differentiated

ANS: B Aneurysms often are asymptomatic. A headache can occur but is not the most common symptom. Nosebleeds do not occur. Subarachnoid hemorrhage is the first indication. REF: p. 405

Which information is basic to the assessment findings associated with a patient diagnosed with an aneurysm? a. A headache is the most common symptom. b. The majority are asymptomatic. c. Nosebleeds are an early symptom. d. Epidural hemorrhage occurs in over 80% of patients.

ANS: B Leptin, a product of the obesity gene (Ob gene), acts on the hypothalamus to suppress appetite and functions to regulate body weight within a fairly narrow range. Leptin levels increase as the number of adipocytes increases; however, for unknown reasons, high leptin levels are ineffective at decreasing appetite and energy expenditure, a condition known as leptin resistance. Leptin resistance fails to inhibit oriexigenic hypothalamic satiety signaling and promotes overeating and excessive weight gain. Insulin becomes resistant, not present in excess. Leptin resistance, not adipocyte failure, leads to obesity. Malabsorption does not lead to obesity, but primarily to weight loss. REF: p. 925

Which of the following conditions is thought to contribute to the development of obesity? a. Insulin excess b. Leptin resistance c. Adipocyte failure d. Malabsorption

ANS: A In its normal, nonmutant state, an oncogene is referred to as a proto-oncogene. A proto-oncogene is not an altered gene, an inactive gene, or a tumor-suppressor gene. REF: p. 240

Which of the following indicates a nurse understands a proto-oncogene? A proto-oncogene is best defined as a(n) _____ gene. a. normal b. altered c. inactive d. tumor-suppressor

ANS: B Bacterial infections lead to secretory diarrhea. A nonabsorbable substance in the intestine leads to osmotic diarrhea. Hypotonic diarrhea is not a form of diarrhea. Food is not mixed properly, digestion and absorption are impaired, and motility is increased, leading to motility diarrhea. REF: p. 908

Which of the following types of diarrhea would most likely occur with a bacterial GI infection? a. Osmotic b. Secretory c. Hypotonic d. Motility

ANS: D Dehydration is a risk factor because it increases blood viscosity and decreases cerebral perfusion. Hyperthyroidism would lead to increased blood pressure but does not place the patient at risk for thrombotic stroke. Hypotension, not hypertension, is a risk factor for thrombotic stroke. Anemia would decrease a person's risk for thrombotic stroke. REF: p. 403

Which of the following would increase a patient's risk for thrombotic stroke? a. Hyperthyroidism b. Hypertension c. Anemia d. Dehydration

Outbreaks of hepatitis _____ often occur in young children attending day care centers and can be attributed to poor hand washing a. A b. B c. C d. D

a. A

A cardiologist is teaching about the period that follows depolarization of the myocardium and represents a period during which no new cardiac potential can be propagated. What is the cardiologist describing? a. Absolute refractory b. Hyperpolarization c. Resting d. Threshold

a. Absolute refractory

Which of the following compounds has been shown to increase the risk of cancer when used in combination with smoking? a. Alcohol b. Steroids c. Antihistamines d. Hypnotics

a. Alcohol

ADH is important in: a. The body's water balance and urine concentration b. Maintaining electrolyte levels and concentration c. Follicular maturation d. Regulation of metabolic processes

a. The body's water balance and urine concentration

A 65-year-old male is transported to the ER for chest pain. An electrocardiogram reveals a prolonged QRS interval. What is the nurse monitoring when the nurse observes the QRS complex on the electrocardiogram? The QRS complex reflects: a. Ventricular activity b. Pulmonary artery closure c. Mitral valve opening d. Aortic valve closing

a. Ventricular activity

The complement, clotting, and kinin systems share which of the following characteristics a. activation of a series of proenzymes b. phagocytosis initiation c. granulocyte production d. activated by interferon

a. activation of a series of proenzymes

A 29 year old female presents with cloudy urine, flank pain, hematuria, and fever. which of the following does the nurse suspect the patient is most likely experiencing? a. acute cystitis b. renal calculi c. chronic renal failure d. postrenal renal failure

a. acute cystitis

the nurse is preparing to teach a patient about Addison disease. Which information should the nurse include? The most common cause of Addison disease is a. an autoimmune reaction b. dietary deficiency of sodium and potassium c. cancer d. viral infection of the pituitary gland

a. an autoimmune reaction

A patient is taking an antihistamine. Which assessment finding would indicate the medication is having a therapeutic effect? a. inflammation decreases b. swelling increases c. coughing increases d. free radicals decrease

a. inflammation decreases

A 46 year old female is diagnosed with gastric ulcers. which of the following characterizes the disorder (select all that apply) a. pain occurs immediately after eating b. weight gain occurs c. bile is regurgitated d. occurrence is typically a single event e. duration of treatment is extended

a. pain occurs immediately after eating e. duration of treatment is extended

A 15 year old female presents with flank pain, irritability, malaise, and fever. Tests reveal glomerulonephritis. When the parents ask what could have caused this, how should the nurse respond a. poststreptococcal infection b. nephropathy c. nephrotic syndrome d. potter syndrome

a. poststreptococcal infection

A 45 year old male presents with oliguria. He is diagnosed with chronic glomerulonephritis. The nurse knows oliguria is related to a. thickening of the glomerular membrane and decreased renal blood flow b. increased glomerular capillary oncotic pressure and tubular obstruction c. activation of renin-angiotensin from decreased blood volume d. vasoconstriction fo the efferent arterioles

a. thickening of the glomerular membrane and decreased renal blood flow

Which form of cell communication is used to relate to other cells in direct physical contact? a.Cell junction b.Gap junction c.Desmosomes d.Tight junctions

a.Cell junction

A nurse is discussing the movement of fluid across the arterial end of capillary membranes into the interstitial fluid surrounding the capillary. Which process of fluid movement is the nurse describing? a.Hydrostatic pressure b.Osmosis c.Diffusion d.Active transport

a.Hydrostatic pressure

hypocalemia

abnormally low blood calcium levels (<8.5mg/dL)

Hyperphosphatemia

above-normal serum concentration of inorganic phosphorus due to acute or chronic renal failure

edema

accumulation of fluid in the interstitial spaces

pericardial effusion

accumulation of fluid in the pericardial cavity

Edema

accumulation of fluid w/i the interstitial spaces

compensatory hyperplasia

adaptive response that allows organs to regenerate(callus)

Hyperchloremia

an excess of chloride in the blood plasma often accompanies hypernatremia

When planning care for a cardiac patient, the nurse knows that in response to an increased workload, cardiac myocardial cells will experience hypertrophy which is an: a. increase in size b. decrease in length c. increase in excitability d. decrease in number

ans: a Hypertrophy is a compensatory increase in the size of cells in response to mechanical stimuli (also called mechanical load or stress, such as from stretching, repetitive, chronic, pressure, or volume overload) and consequently increases the size of the affected organ. The cells of the heart and kidneys are particularly prne to enlargement. A decrease in length is not associated with hypertrophy. A deficiency of electrolytes or minerals could lead to an increase in excitability; it is not due to increased workload or related to hypertrophy. A decrease in cell numbers is referred to as hypoplasia. REF: p. 75

Sodium and water accumulation in an injured cell are a direct result of: a. decreased ATP production b. karyorrhexis c. ribosome detachment d. dehydration

ans: a a reduction in ATP levels causes the plasma membrane's sodium-potassium (Na+ - K+) pump and sodium-calcium exchange to fail. Sodium and water can then enter the cell freely. Karyorrhexis mens fragmentation of the nucleus into smaller particles or "nuclear dust." Ribosome detachment reduces protein synthesis. Dehydration leads to loss of sodium and water. REF: p. 80

An 86 year old female patient has the wasting syndrome of aging, making her vulnerable to falls, functional decline, disease, and death. The nurse knows this patient is experiencing a. frailty b. sarcopenia c. somatic death d. cellular aging

ans: a frailty leaves the individual vulnerable to falls, disease, and death. sarcopenia is loss of muscle mass. it is associated with aging but is not as severe as frailty. somatic death is death of the whole person. cellular aging occurs at the cellular level and is characteristic of aging, including cell atrophy, decreased function, and loss of cells. REF: p.109

a 35-year-old male is diagnosed with a hormone-secreting tumor of the adrenal medulla. he experiences elevated blood pressure and increased anxiety. which of the following hormones is the predominant one released by the tumor? a. antidiuretic hormone b. acetylcholine c. norepinephrine d. cortisol

ans: c norepinephrine regulates blood pressure and promotes arousal, increased vigilance, increased anxiety, and other protective emotional responses. antidiuretic hormone regulates urine output. acetylcholine will not result in an increase in blood pressure. cortisol regulates glucocorticoids. REF: p. 221

a 40-year-old female's pap smear indicates abnormal changes in the shape and organization of cervical cells. which term would be used to identify this type of change? a. metaplasia b. atrophy c. hypertrophy d. dysplasia

ans: d wehn cervical cells undergo dysplasia, there is a change in their size, shape, and organization. Metaplasia is the reversible replacement of one mature cell type. The cells have not decreased in size; therefore, atrophy is incorrect. The cells have not increased in size in reponse to stimuli; therefore, they have not hypertrophied. REF: p. 77

NSAIDS

anti-inflammatory

allergens

antigens that produce an allergic response

Volume and Barorecptors found:

atria,chest,aorta,pulmonary arteries and carotid sinus. Also cause release of ADH

interleukins(IL)

attraction of leukocytes to inflammation area

wounds that heal by scar tissue are

avascular

A 54-year-old male is diagnosed with peptic ulcer disease. This condition is most likely caused by: a. Hereditary hormonal imbalances with high gastrin levels b. Breaks in the mucosa and presence of corrosive secretions c. Decreased vagal activity and vascular engorgement d. Gastric erosions related to high ammonia levels and bile reflux

b. Breaks in the mucosa and presence of corrosive secretions

A 5-year-old male was diagnosed with normocytic-normochromic anemia. Which of the following anemias does the nurse suspect the patient has? a. Sideroblastic anemia b. Hemolytic anemia c. Pernicious anemia d. Iron deficiency anemia

b. Hemolytic anemia

Which information by the nurse indicates a good understanding of depolarization? Depolarization of a cardiac muscle cell occurs as the result of: a. A decrease in the permeability of the cell membrane to ions b. The rapid movement of ions across the cell membrane c. A blockade by calcium ions d. Stimuli instigated during the refractory period

b. The rapid movement of ions across the cell membrane

When a nurse observes post-streptococcal glomerulonephritis as a diagnosis on a patient, which principle will the nurse remember? Acute post-streptococcal glomerulonephritis is primarily caused by a. swelling of mesangial cells in the Bowman space in response to the presence of bacterial b. antigen-antibody complex deposition in the glomerular capillaries and inflammatory damage c. inflammatory factors that stimulate cellular proliferation of epithelial cells d. accumulation of anti-glomerular basement membrane antibodies

b. antigen-antibody complex deposition in the glomerular capillaries and inflammatory damage

A 15 year old male is diagnosed with infectious mononucleosis. when the patient asks how he got this disease, how should the nurse respond? the most likely cause is a. adenovirus b. epstein-barr virus (EBV) c. cytomegalovirus (CMV) d. toxoplasma gondii

b. epstein-barr virus (EBV)

When a nurse is reviewing lab results and notices that the erythrocytes contain an abnormally low concentration of hemoglobin, the nurse calls these erythrocytes a. hyperchromic b. hypochromic c. macrocytic d. microcytic

b. hypochromic

A 45 year old female presents with hypertension, anorexia, nausea and vomiting, and anemia. She is diagnosed with chronic renal failure. When the patient asks what caused this anemia, how should the nurse respond? Your anemia is caused by a. red blood cells being lost in the urine b. inadequate production of erythropoietin c. inadequate iron absorption in the gut d. red blood cells being injured as they pass through the glomerulus

b. inadequate production of erythropoietin

When a nurse is asked how cytokines are classified, which of the following is the best response? Cytokines are classified as: a. leukocytes or monocytes b. interleukins or interferons c. platelets or macrophages d. bacteria or microorganisms

b. interleukins or interferons The majority of important cytokines are classified as interleukins or interferons.

The icteric phase of hepatitis is characterized by which of the following clinical manifestations a. fatigue, malaise, vomiting b. jaundice, dark urine, enlarged liver c. resolution of jaundice, liver function returns to normal d. fulminant liver failure, hepatorenal syndrome

b. jaundice, dark urine, enlarged liver

When caring for a patient with tissue injury, which principle should the nurse remember? a. macrophage b. mast cell c. fibroblast d. neutrophil

b. mast cell An inflammatory response to tissue injury is typically initiated by mast cells.

A serious complication of paraesophageal hiatal hernia is a. hemorrhage b. strangulation c. peritonitis d. ascites

b. strangulation

A 75 year old male reports to his primary care provider loss of urine with cough, sneezing, or laughing. Which of the following is the most likely diagnosis the nurse will observe on the chart a. urge incontinence b. stress incontinence c. overflow incontinence d. functional incontinence

b. stress incontinence

when a nurse is assessing the physical features of individuals with Cushing syndrome, these findings will include a. weight loss and muscle wasting b. truncal obesity and moon face c. pallor and swollen tongue d. depigmented skin and eyelid lag

b. truncal obesity and moon face

The faculty member asked the student to identify the appropriate term for the movement of small, electrically uncharged molecules through a semipermeable barrier. Which answer indicates the nursing student understood the teaching? a.Osmosis b.Diffusion c.Hydrostatic pressure d.Active transport

b.Diffusion

A 30-year-old male is involved in a motor vehicle accident and sustains trauma to the lungs and chest wall. He experiences respiratory failure. Which of the following lab values would the nurse expect? a.Electrolyte imbalances b.Elevated PaCO2 c.Low hematocrit d.Elevated pH

b.Elevated PaCO2

Hypophosphatemia

below-normal serum concentration of inorganic phosphorus caused by intestinal malabsorption and increased renal excretion of phosphate

epithelial cells line

blood vessels, air sacs, respiratory tract, bladder, GI tract, ducts,skin

A 30-year-old female is diagnosed with cancer. Testing reveals that the cancer cells have spread to local lymph nodes. A nurse realizes this cancer would be documented as stage: a. 1 b. 2 c. 3 d. 4

c. 3

A nurse checks lab results as both Cushing syndrome and Addison disease can manifest with elevated levels of: a. ADH b. Cortisol c. Adrenocorticotropic hormone (ACTH) d. Aldosterone

c. Adrenocorticotropic hormone (ACTH)

A 72-year-old male demonstrates left-sided weakness of upper and lower extremities. The symptoms lasted 4 hours and resolved with no evidence of infarction. The patient most likely experienced a(n): a. Stroke in evolution b. Arteriovenous malformation c. Transient ischemic attack d. Cerebral hemorrhage

c. Transient ischemic attack

The cardinal sign of pyloric stenosis caused by ulceration or tumors is: a. Constipation b. Diarrhea c. Vomiting d. Heartburn

c. Vomiting

a 53 year old male with a 20 year history of smoking is diagnosed with emphysema. when the nurse is asked what causes this, what is the nurse's best response? changes in his lungs are caused by a. viral infections b. destruction of alveolar macrophages c. alpha-1-antitrypsin deficiency d. fibrotic lung disease

c. alpha-1-antitrypsin deficiency

A 30 year old obese female underwent gastric resection in an attempt to lose weight. which of the following complications could the surgery cause a. constipation b. acid reflux gastritis c. anemia d. hiccups

c. anemia

A 16 year old female presents with abdominal pain in the right lower quadrant. Physical examination reveals rebound tenderness and low grade fever. A possible diagnosis would be a. colon cancer b. pancreatitis c. appendicitis d. hepatitis

c. appendicitis

While discussing the complement system, which information should the nurse include? The alternative pathway of the complement system is activated by: a. histamine b. antigen antibody complexes c. bacteria d. bleeding

c. bacteria

Acute pancreatitis often manifests with pain to which of the following regions a. right lower quadrant b. right upper quadrant c. epigastric d. suprapubic

c. epigastric

The most common clinical manifestation of portal hypertension is ____ bleeding a. rectal b. duodenal c. esophageal d. intestinal

c. esophageal

The most common disorder associated with upper GI bleeding is a. diverticulosis b. hemorrhoids c. esophageal varices d. cancer

c. esophageal varices

a nurse is preparing to teach the staff about asthma. which information should the nurse include? Airway hyper-responsiveness in asthma is related to a. increased sympathetic nervous system response b. the release of stress hormones c. exposure to an allergen causing mast cell degranulation d. hereditary decrease in IgE responsiveness

c. exposure to an allergen causing mast cell degranulation

A 52 year old presents with bleeding from the rectum. this condition is referred to as a. melena b. occulut bleeding c. hematochezia d. hematemesis

c. hematochezia

You are working in the emergency department, and a 35-year-old female motor vehicle accident has experienced blood loss and hypovolemia. Her kidneys were not perfused well following the accident. All of the following would be true EXCEPT a. the patient would have significant vasoconstriction b. sodium and water retention would occur at the level of the kidney c. potassium retention would occur d. renin would be released from the juxtaglomerular cells

c. potassium retention would occur

A 42 year old female is diagnosed with chronic renal failure and the nurse is discussing dietary treatment. Which information indicates the nurse understands dietary regimen? Treatment includes restricting a. fats b. complex carbohydrates c. proteins d. sugars

c. proteins

While planning care for a patient who has acute pyelonephritis, a nurse recalls the most common condition associated with the development of acute pyelonephritis is a. cystitis b. renal cancer c. urinary tract obstruction d. nephrotic syndrome

c. urinary tract obstruction

A 13 year old female is diagnosed with asthma. which of the following should the nurse teach the patient to recognize as part of an asthmatic attack a. headache b. chest pain c. wheezing d. low heart rate

c. wheezing

When a student asks which type of nerves transmit nerve impulses at the fastest rate, what is the best response by the nurse? a.Large, non-myelinated b.Small, myelinated c.Large, myelinated d.Small, non-myelinated

c.Large, myelinated

Forces opposing filtration

capillary oncotic pressure and interstitial hydrostatic pressure

epinephrine and noepinephrine

cause vasoconstriction, increased heart rate, bronchodilation in response to stress -bind to alpha and beta cells

beta receptors cause what to the heart and lungs

causes harder contractions in the heart and increased heart rate. The lungs bronchodilate.

beta blockers

causes heart to contract slower and decreased heart rate. Lungs bronchoconstrict.

isotonic fluid excess

causes hypervolemia

Aldosterone does what to na and k

causes na to be reabsorbed and k to be excreted

alterations of DNA

causing mutations of cells

Necrosis

cell death from a bad stimulus, not planned(bad thing)

Cell trauma causes

cell to die and release its content(including K+) into stream

Reprofusion injury

cells die and leave trash in tissues, occlusion is removed all the trash gets picked up by the vessel and causes further damage(free radicals)

cancer

cells divide w/o control and invade other tissues(requires multiple mutations)

humoral(immunoglobulins(antibodies))

circulating in the blood bind to the antigen. Can inactivate the microorganism or activate inflammatory response that destroy the pathogen. -produced by the b cells

debridement

cleaning the dissolved clots,microorganisms,erythrocytes, and dead tissue cells

agglutionation

clumping the antigens together so they can not affect the cell

autocrine

communication that involves targeting itself. Cancer cells use this to proliferate.

__________, ___________, and __________ are associated with excessive salt and water retention, which causes ______, _______, __________

congestive heart failure, renal failure, cirrhosis of the liver plasma volume overload, increased capillary hydrostatic pressure, and edema

impaired contraction

contracture (burns)

T helper (Th)

control both cell mediated and humoral immune reponse and active other cells such as macrophages

adrenal glands secrete

cortisol and catecholamines

Adenosine Triphosphate (ATP)

created from chemical energy, Stores and transfers energy.

pattern formation

creation of tissue and organs from cells that are similar in function and structure

The body's inability to conserve water and sodium when affected by Addison disease is explained by which of the following conditions? a. Elevated levels of cortisol b. Decreased levels of ACTH c. Hypersecretion of ADH d. Aldosterone deficiency

d. Aldosterone deficiency

A 39-year-old female presents with abdominal pain and jaundice. She is diagnosed with gallstones and undergoes cholecystectomy. An analysis of her gallstones would most likely reveal a high concentration of: a. Phosphate b. Bilirubin c. Urate d. Cholesterol

d. Cholesterol

Which statement indicates the patient has a good understanding of cancer risk factors? The most important environmental risk factor for cancer is exposure to: a. Ultraviolet (UV) radiation b. Radon c. Estrogen d. Cigarette smoke

d. Cigarette smoke

A nurse recalls physical activity was shown to reduce the risk of which of the following types of cancer? a. Prostate b. Lung c. Bone d. Colon

d. Colon

When an oncologist is discussing the degree to which an organism's development is contingent on its environment, which of the following is the oncologist explaining? a. Transgenerational inheritance b. Epigenetics c. Histone modification d. Developmental plasticity

d. Developmental plasticity

A nurse is caring for a patient who cannot clot. Which end product of the clotting cascade is this patient unable to make? a. Collagen b. Fibrinogen c. Thrombin d. Fibrin

d. Fibrin

A 34-year-old male presents in the emergency room with extreme fatigue and shortness of breath. His skin and sclera appear to have a yellowish discoloration. These assessment findings are consistent with which type of anemia? a. Posthemorrhagic anemia b. Iron deficiency anemia c. Aplastic anemia d. Hemolytic anemia

d. Hemolytic anemia

A 3 month old female develops colicky pain, abdominal distention, and diarrhea after drinking cow's milk. the best explanation for her symptoms is a. deficiency of bile that stimulates digestive secretions and bowel motility b. excess of amylase, which increases the breakdown of starch and causes and osmotic diarrhea c. overgrowth of bacteria from undigested fat molecules, which leads to gas formation and decreased bowel motility d. excess of undigested lactose in her digestive tract, resulting in increased fluid movement into the digestive lumen and increased bowel motility

d. excess of undigested lactose in her digestive tract, resulting in increased fluid movement into the digestive lumen and increased bowel motility

A 15 year old male was diagnosed with pharyngitis. Eight days later he developed acute glomerulonephritis. While reviewing the culture results, which of the following is the most likely cause of this disease a. Kelbsiella b. human immunodeficiency virus (HIV) c. genital herpes virus d. group A B-hemolytic streptococcus

d. group A B-hemolytic streptococcus

A 45 year old male complains of heartburn after eating and difficult swallowing. he probably has a. pyloric stenosis b. gastric cancer c. achalasia d. hiatal hernia

d. hiatal hernia

T cytotoxic (Tc)

destroys cells directly

plasma membrane receptors

determine response to binding

Hypertonic fluid alterations

develop when the osmolality of the ECF is elevated above normal (>294mOsm)

increased loss of potassium is caused by:

diuretics which get rid of the potassium with urine

free radicals

electrically charged atom that has an unpaired electron

electrolytes

electrically charged make up 95% of solutes

metabolic alkalosis

elevation of HCO3- usually caused by an excessive loss of metabolic acids

Catabolism

energy is released by destruction of complex substances to simple complexes

Anabolism

energy using to build up

acute stress

enhances the immunity

hypervolemia

excess blood volume

hypernatremia

excess sodium in the blood or just an excess water loss

hypervolemia causes

excessive fluid,hyper secretion of ADH,drugs that increase absorption of NA+ and water

Hyperkalemia

excessive potassium in the blood (>5.5 mEq/L)

interstitial hydrostatic pressure

facilitates the inward movement of water from the interstitial space into the capillary

capillary hydrostatic pressure

facilitates the outward movement of water from the capillary to the interstitial space

healing

filling in the wound

hypervolemia

fluid excess

Net Filtration

forces favoring - forces opposing

net filtration (also referred to as ______ hypothesis

forces favoring filtration - forces opposing filtration Starling

hypersecretion of growth hormone (GH) in children

gigantism

adjuvant

given after surgical removal of the tumor to eliminate any micro metastases that may be present

neoadjuvant

given before surgery to shrink the tumor to that not as much surrounding tissues will be compromised

brain tumors cause

headaches,seizures,neurologic defects

A 28-year-old male complains to his physician that he has had a cold for a week and is coughing up bloody secretions. This condition is referred to as:

hemoptysis

neurohormonal

hormones are released into blood by neurosecretory neurons which then goes to its target cell.

ligand types

hormones, neurotransmitters, antigens, complement components, lipoproteins, infectious agents, drugs and metabolites.

A patient develops severe dehydration and hyperglycemia in the absence of ketosis. Which of the following conditions does this patient have?

hyperosmolar hyperglycemic nonketoic syndrome

neutralization

inactivating or blocking the receptors sites so antigen cannot bind to the cell

proenzymes

inactive enzymes for inflammation

geriatric risk of dehydration due to

increase adipose tissue decreased muscle mass renal decline diminished thirst perception

acidosis is

increase in H+ or decrease in bicarb

hypertrophy

increase in cell size cause an entire organ to enlarge

hyperplasia

increase in number of cells from increase in cellular division

hyperkalemia causes

increased intake, shift of K+ from ICF to ECF(acidosis), decreased of renal excretion, insulin deficiency or cell trauma

kidney hypertrophy

increased workload because of removal of a kidney

alcohol consumption

increases risk of cancer

incidence of cancer is

increasing of cancer

stress increases

inflammation

increased entry of potassium to cells caused by:

insulin administration

2/3s of all cancer

is environmental factor interacting w/ genes

It is correct to assume that primary hypothyroidism

is the result of absent thyroid tissue

isotonic fluid loss

isotonic dehydration; causes contraction of the ECF volume with weight loss, dryness of skin and mucous membranes, decreased urine output, and symptoms of hypovolemia--rapid heart rate, flattened neck veins, and normal or decreased blood pressure

hypernatermia treatment

isotonic fluids low in sodium such as D5W

pH regulated by

kidneys, lungs, and buffers

atrophy normally happens because

lack of use, blood supply, nutrition, hormonal stimulation, nervous system simulation

ICF (intracellular fluid) contains

large amounts of K small amounts of Na and Cl high concentration of Phosphate and Mg

ECF (extracellular fluid) contains

large amounts of Na and Cl small amounts of K high concentration of Ca

acute inflammation

lasts 8-10 days self limiting

cancer is

leading cause of death in developed world

alterations of proteins lead to

leads to protein lost

Aldosterone

leads to sodium and water reabsorption back into circulation and potassium and hydrogen to be lost in urine

Leukotrienes

like histamines but takes longer

Localized edema

limited swelling to site of trauma ex sprained finger, injury, cerebral edema, pulmonary edema, pleural effusion

stage 2

locally invasive

anaplasia

loss of cellular differentiation(dont look like normal cells)

hepatitis causes

majority of liver cancer

osmolarity

measures the number of milliosmoles per liter of solution, or the concentration of molecules per volume of solution.

normal flora

microorganisms that are normally present in a specific site protect against disease

antigens

molecular targets of the lymphocytes and antibodies

Diffusion

movement of solutes from high to low, influenced by electrical potential difference, size and lipid solubility

Filtration

movement out of the capillary bed into interstitial space

Hypercalcemia

muscle weakness constipation kidney stones (renal calculi) cardiac arrest emergency!

inducible

must recognize the pathogen as foreign or non-self. Something that does not belong there.

physiologic adaptation

normal cell stimulation by hormones or other mediators(uterine enlargement during pregnancy)

prolonged use of antibiotics cause

normal flora to die and lead to infections

hypovolemia

not enough blood volume

hypokalemia

not enough potassium in the ICF

hyponatremia

not enough sodium below 135

hyponatremia

not enough sodium in the blood or too much water that its dilutes the blood

hypotonic fluid imbalances

occur when the osmolality of the ECF is less than 280 mOsm

pathogenic adaptation

occurs as a response to some pathological state, usually not helpful in the long run

monovalent

one charge

to maintain electrochemical balance

one divalent ion will combine with two monovalent ions

Narcotics

opium related drugs

Anaerobic glycolysis

oxygen not present CHO is converted to pyruvic acid in cytoplasm with production of 2 ATP(not enough) pyruvate then converts to lactic acid.

Calcium and phosphate concentrations controlled by

parathyroid hormone (PTH) Vitamin D calcitonin

stress

perceived threat that disrupts a person well being

sodium deficits cause

plasma hypo-osmolality and cellular swelling

hyperkalemia

potassium levels above 5.5 mEq/L -rare because of efficient renal excretion

hypokalemia

potassium levels are lower then 3.5 mEq/L

oncotic pressure

pressure exerted by solutes(mostly plasma proteins/albumin)

oncotic pressure

pressure in the blood vessels by plasma proteins

cellular receptors

protein molecules that are either on the plasma membrane, cytoplasm or nucleus(bind to ligands)

hypokalemia causes

reduced intake of potassium, increased entry of potassium into cells, and increased loss of potassium

compensation

renal and respiratory adjustments to pH

Evevated K+ levels causes release of what hormone

renin

hypovolemia treatment

replace fluids using isotonic solutions and/or blood products

cytokines

requires ligand to release it and assist immunee

Baroreceptors reflexes

respond to changes in blood pressure/volume

skeletal muscles hypertrophy

response to heavy workload

heart hypertrophy

response to high blood pressure(afterload)

Resolution

returning injured tissue to the original structure and function

epithelializational

sealing the wound

hypothalamus stress response

secretes CRH

pain in cancer

significant pain in the late stages

haptens

small antigens that bind to larger molecules that carry them

Hypernatremia

sodium greater than145,sodium gain or water loss, water moves from the ICF to ECF

hepatic cancers

stretch the liver causes feelings of bloating

Potassium levels in the body are regulated by

the kidney, aldosterone and insulin secretion, and by changes in pH

potassium major influence for balance is

the kidneys

hypotonic

the solution has less concentration than the blood

Total Body Water (TBW)

the sum of fluids within all body compartments; about 60% of body weight; expressed as a % of body weight in kilograms

t cells formed

thymus

tumor refers

to new growth

leukocytosis

too many circulating leukocytes

hyperkalemia

too much potassium in the ICF

xenobiotics

toxic, mutagenic, and carcinogenic chemicals in food

histamine

vasodilation, increase permeability to capillar

reduced intake of potassium caused by:

vomiting, diarrhea and renal disorders

primary intention

wounds that heal under conditions of minimal tissue loss

Starling forces

- hydrostatic and oncotic pressures - both hydrostatic and oncotic are found in capillary and interstitial -balance of these forces maintains proper fluid volumes & solute concentrations inside & outside the vasculature -Imbalance of these pressures results in too little or too much fluid in tissues

Hyponatremia manifestations

-Confusion, irritability, headache, seizures, and coma -if leads to loss of ECF and hypovolemia see: hypotension, tachycardia, decreased urine output -dilutional from excess water see: weight gain, edema, ascites and jugular vein distention

coagulation system

-Forms a fibrinous meshwork at an injured or inflamed site -framework for healing -forms clot at the site

acidosis

-H+ accumulates in ICF -K+ shifts out to maintain the balance of cations across membrane -leads to hyperkalemia

tumor cell markers used for what

-screen and identify individuals w/ high risk for cancer -diagnose types of cancer -observe course of cancer -problems: false + and -

4 Causes of Edema

-Increase in capillary hydrostatic pressure -decrease in plasma oncotic pressure -Increase in capillary permeability -Lymph obstruction

pathologic hyperplasia

-abnormal proliferation of normal cells -usually occurs in endometrial tissues because of imbalance of progesterone and estrogen

h2 receptors(histamine)

-anti-inflammation -parietal cells of stomach mucosa -induces secretion of gastric acid

immunogens

-antigens that are able to trigger an immune response. -depends on foreignness, molecular size, quanity

kinin system

-assist inflammation -primary kinin is bradykinin -causes:dilation leads to smooth muscle contraction, increase permeability and acts with prostaglandins to produce pain

inflammatory response: vascular response

-blood vessel dilate -increased permeability -WBC adhere to the walls and migrate toward infection

chemokines

-call for chemotaxis -promotes phagocytosis -wound healing

anoxia cellular response

-decrease in ATP, failure of na/k pumps and na and calcium exchange -cellular swelling -vacuolation

elevated levels of cortisol are shown to

-decrease innate immunity -increase autoimmune response

complement system

-destroy pathogens directly via proteins -activates every other component of inflammatory responce

passive immunity

-doesnt involve host immune system at all -antibodies or t cells is transferred from donor to a recipient -ex: mother to baby during pregnancy -short lived as long as donor t cells live

ionizing radiation goal

-eradicate cancer w/o excessive toxicity -avoid damage to normal structures

Aging TBW 3 effects

-increased adipose tissues and decreased muscle mass -Renal decline -diminished thirst perception

tumor necrosis factor

-induces fever -increases synthesis of inflammatory serum proteins -causes muscle wasting(cachexia)

primary response

-initial response -5 to 7 days an IgM is detected -igG response equal or slightly less follows the IgM response

IgM

-largest immunoglobulins -1st antibody produced during the primary response to an antigen -synthesized during fetal life

free radicals damage:

-lipid peroxidation; destruction of polyunsaturated lipids -alterations of proteins -alterations of DNA

Potassium

-major intracellular cation -regulated intracellular electrical neutrality in relation to H+ and Na+

IgE

-mediator of common allergic responses -defender against parasites

Hyperkalemia treatments

-meds given in GI tract -Insulin/Glucose drip -Sodium bicarbonate given to correct acidosis -Dialysis

Secondary response

-more rapid -large amounts of antibody is produced -IgM similar quantities to primary, IgG produced in greater numbers

IgG

-most abundant (80-85%) -accounts for most protective activity against infections -transported across the placenta

hypoxia

-most common cause of cell injury -lack of oxygen to the cell

Ischemia

-most common hypoxia -reduction of blood supply to the tissues

chemotherapy

-non selectively drugs that target cells -schedule must be strict so cancer doesnt get immune

dysplasia

-not true adaptive change -abnormal changes in size,shape and organization -could lead to development of cancer but not all the time

ischemia-reperfusion injury mechanisms

-oxidative stress -increased intracellular calcium - inflammation -complement activation

tumor cell markers found where

-plasma cell membranes -blood -CSF -urine

h1 receptors(histamine)

-pro inflammation -smooth muscle of bronchi

growth hormone

-produced by anterior pituitary gland -enhances immune function -chronic stress decreases growth hormone

cellular

-produced by the t cells -kill target directly

cortisol is show to

-promote resolution and repair -induce t cell apoptosis

hyponatremia causes

-pure sodium loss -low intake -dilutional hyponatremia(excessive water)

fever

-pyrogens -acts directly on hypothalamus

hypoxia results from

-reduced amount of oxygen in the air -loss of hemoglobin/decreased efficacy of hemoglobin -decreased production of red blood cells -diseases of respiratory or cardiovascular system -poisoning of oxidative enzymes w/i the cell

repair

-replacement of destroyed tissue with scar tissue

metaplasia

-replacement of one cell type for another -reversible -ex: smoker lung lining is different than non smoker(mucus to non mucus producing)

anoxia

-total lack of oxygen to tissue -usually result of acute obstruction of blood vessel

Generalized edema treatment

-treat underlying disease -meds that reduce fluid volume(diuretics) -Compression stocking -Elevating the affected extremity -restricting fluid and volume intake

clonal selection

-turn t and b cells into more mature form and allow them to produce antibodies

Generalized edema S/S

-weight gain -swelling and puffiness -tight fitting clothes and shoes

ANS: 150,000 REF: p. 541

. A nurse realizes a patient has thrombocytopenia when the platelet count is below _____ platelets per cubic millimeter.

ANS: D Thrombocythemia is characterized by a platelet count more than 400,000 platelets per cubic millimeter of blood and is symptomatic when the count exceeds 1,000,000 platelets per cubic millimeter, at which time the risk for intravascular clotting (thrombosis) is high. Thrombocythemia is characterized by a high platelet count, not a defective one, a fragmented one, or a consumed one. REF: p. 543

. A patient is diagnosed with primary thrombocythemia. A nurse would expect the blood smear to reveal _____ platelets. a. defective b. fragmented c. consumed d. overproduced

Hypocalcemia treatment

1. Vitamin D (Helps utilize Ca) 2. Phosphate binders (Renagel, PhosLo, OsCal) 3. IV Ca (GIVE SLOWLY) and always make sure client is on a heart monitor

carcinoma in situ(CIS)

1. remain stable for long 2. can progress to invasive and metastatic cancer 3. can regress and disappear

hyponatremia treatment (3)

1. small amounts of hypertonic sodium chloride 2. Restriction of water 3.ADH anatagonists

dysfunctional wound healing (4)

1. Ischemia 2. Excessive bleeding 3. Excessive fibrin deposition 4. predisposing factors

hyperkalemia treatment (4)

1. Meds given in GI tract - High in Na exchanged for K which is excreted in the feces 2. Insulin/Glucose drip 3. Sodium bicarbonate IV to correct acidosis 4. Dialysis

controlling pain

1. NSAIDS 2. Narcotics 3. Palliative surgery 4. Radiation therapy

hypervolemia treatment (2)

1. Restrict sodium intake 2. Diuretics

ANS: C The clinical manifestations of alcoholic steatohepatitis include jaundice, hepatomegaly, and testicular atrophy. These symptoms are not a result of a bacterial or viral infection, or a drug overdose. REF: p. 932

1. The autopsy of a 55-year-old revealed an enlarged liver, testicular atrophy, and mild jaundice secondary to cirrhosis. What is the most likely cause of this condition? a. Bacterial infection b. Viral infection c. Alcoholic steatohepatitis d. Drug overdose

anemia mechanisms (4)

1. chronic bleeding 2. malnutrition 3. medical therapies 4. malignancy in blood forming organs

Characterstics of cancer cells

1. decreased need for growth hormone to grow 2. lack contact inhibition 3. anchorage independence 4. immortality

adaptive immunity purposes (2)

1. destruction of infectious 2. long term protection against future exposure

indirect (3)

1. inflammation 2. Phagocytosis 3. Complement

hypernatremia treatment (1)

1. isotonic fluids low in sodium such as D5W

inflammation goals

1. limit infection 2. initiate adaptive immune response 3. initiate healingg

chronic inflammation (4)

1. peptic ulcer disease 2. stomach carcinoma 3. lymphoid tissues lymphomas 4. chronic viral hepatitis

pain mechanisms in cancer (6)

1. pressure 2. obstruction 3. invasion of sensitive areas 4. stretching of tissues 5. tissue destruction 6. inflammation

cancer surgery purpose (5)

1. prevent cancer 2. biopsy for diagnosis and staging 3. lymph node sampling 4. debulking surgery 5. Palliative surgery

ANS: C Appendicitis is manifested originally with periumbilical pain that then migrates to the right lower quadrant pain with rebound tenderness. A low-grade fever is common. Colon cancer may be asymptomatic, followed by bleeding. Pancreatitis is manifested by vomiting. Hepatitis would be manifested by upper abdominal pain, not lower. REF: p. 923

19-year-old presents with abdominal pain in the right lower quadrant. Physical examination reveals rebound tenderness and a low-grade fever. A possible diagnosis would be: a. colon cancer. b. pancreatitis. c. appendicitis. d. hepatitis.

An infant is diagnosed with noncommunicating hydrocephalus. What is an immediate priority concern for this patient? Question options: 1) Metabolic edema 2) Interstitial edema 3) Vasogenic edema 4) Ischemic edema

2) Interstitial edema

If a patient had a problem with the hypothalamus, which of the following hormones would be affected? 1) ACTH 2) Oxytocin 3) ADH 4) TSH

2) Oxytocin

proliferation phase

3-4 days after injury, last as long as 2 weeks

Potassium concentration

3.5-5 mg/dl

When the endocrinologist asks the staff how the releasing hormones that are made in the hypothalamus travel to the anterior pituitary, how should the staff reply? Via the: 1) Vessels of the zona fasciculata 2) Chromophils 3) Median eminence 4) Hypophysial portal system

4) Hypophysial portal system

A nurse is teaching staff about protein hormones. Which information should the nurse include? One of the protein hormones is: 1) Thyroxine (T4) 2) Aldosterone 3) Testosterone 4) Insulin

4) Insulin

ANS: C Infectious mononucleosis (IM) is usually self-limiting, and recovery occurs in a few weeks; severe clinical complications are rare (5%). Fatigue may last for 1-2 months after resolution of other symptoms. REF: p. 525

A 15-year-old female presents with splenomegaly, hepatomegaly, and lymph node enlargement. She is diagnosed with infectious mononucleosis. What should the nurse tell the patient about the recovery time? a. 72 hours b. 3-5 days c. A few weeks d. 6 months

ANS: D The most common causes of acute infections are bacterial microorganisms including Pseudomonas, Escherichia coli, and Staphylococcus aureus. REF: p. 352

A 15-year-old is diagnosed with an outer ear infection. Which of the following is most likely to cause this infection? a. Haemophilus b. Streptococcus pneumonia c. Moraxella catarrhalis d. Escherichia coli

ANS: B Infectious mononucleosis (IM) is an acute infection of B lymphocytes (B cells) with EBV. IM is not associated with adenovirus, cytomegalovirus, or Toxoplasma gondii. REF: p. 525

A 15-year-old male is diagnosed with infectious mononucleosis (IM). When the patient asks how he got this disease, how should the nurse respond? The most likely cause is: a. adenovirus b. Epstein-Barr virus (EBV) c. cytomegalovirus (CMV) d. Toxoplasma gondii

ANS: A, B, D, E At the time of diagnosis, the individual commonly presents with the classic group of symptoms: fever, sore throat, cervical lymph node enlargement, and fatigue; generalized lymph node enlargement also may develop, as well as enlargement of the spleen and liver. REF: p. 525

A 15-year-old male is diagnosed with infectious mononucleosis. Which of the following assessment findings would he most likely demonstrate? (select all that apply) a. Lymph node enlargement b. Fever and sore throat c. Rash on the trunk and extremities d. Fatigue e. Enlargement of liver and spleen

ANS: B When a person loses cerebral function, the reticular activating system and brainstem can maintain a crude waking state known as a VS. Cognitive cerebral functions, however, cannot occur without a functioning reticular activating system. A VS is not associated with the cerebral cortex, spinal cord, or cerebellum. REF: p. 364

A 20-year-old experiences a severe closed head injury as a result of a motor vehicle accident. Which of the following structures is most likely keeping the patient in a vegetative state (VS) 1 month after the accident? a. Cerebral cortex b. Brainstem c. Spinal cord d. Cerebellum

ANS: A A nonabsorbable substance in the intestine leads to osmotic diarrhea. Infections lead to secretory diarrhea. Hypotonic diarrhea is not a form of diarrhea. Food is not mixed properly, digestion and absorption are impaired, and motility is increased leading to motility diarrhea. REF: p. 908

A 20-year-old recently diagnosed with lactose intolerance eats an ice cream cone and develops diarrhea. This diarrhea can be classified as _____ diarrhea. a. osmotic b. secretory c. hypotonic d. motility

ANS: B Iron deficiency anemia is manifested by fingernails that become brittle and spoon shaped or concave. It does not involve hyperactivity, gait problems, or petechiae. REF: pp. 517-518

A 21-year-old female was recently diagnosed with iron deficiency anemia. In addition to fatigue and weakness, which of the following clinical signs and symptoms would she most likely exhibit? a. Hyperactivity b. Spoon-shaped nails c. Gait problems d. Petechiae

ANS: A Iron replacement therapy is required and very effective. Initial doses are 150-200 milligrams per day and are continued until the serum ferritin level reaches 50 milligrams per liter. Neither a splenectomy nor a bone marrow transplant is indicated. REF: p. 518

A 21-year-old woman was recently diagnosed with iron deficiency anemia. Her hematocrit is 32%. Which of the following treatments would the nurse expect to be prescribed for her? a. Iron replacement b. Splenectomy c. A bone marrow transplant d. No treatment is necessary

ANS: B A Cushing ulcer is a stress ulcer associated with severe head trauma or brain surgery that results from decreased mucosal blood flow and hypersecretion of acid caused by overstimulation of the vagal nerve. Cushing ulcers are not associated with infections or H. pylori. Curling ulcers develop secondary to burns. REF: p. 918

A 22-year-old underwent brain surgery to remove a tumor. Following surgery, the patient experienced a peptic ulcer. This ulcer is referred to as a(n) _____ ulcer. a. infectious b. Cushing c. H. pylori d. Curling

ANS: A The presence of Helicobacter pylori is associated with gastric cancer, not leukemia, lung cancer, or colon cancer. REF: p. 249

A 45-year-old male presents with persistent, severe stomach pain. Testing reveals a peptic ulcer. Further laboratory tests reveal the presence of Helicobacter pylori. Which of the following is of concern for this patient? a. Gastric cancer b. Leukemia c. Lung cancer d. Adenocarcinoma of the colon

ANS: B Tumors that arise from or form ductal or glandular structures are named adenocarcinomas. Cancers arising in epithelial tissue are called carcinomas; mesenchymal tissue (including connective tissue, muscle, and bone) usually have the suffix sarcoma; lymphatic tissue are called lymphomas. REF: p. 234

A 25-year-old male develops a tumor of the breast glandular tissue. What type of tumor will be documented on the chart? a. Carcinoma b. Adenocarcinoma c. Sarcoma d. Lymphoma

Phosphate acts as a

buffer in acid-base regulation and provides energy for muscle contraction.

ANS: C Cancer that has spread to regional structures, such as lymph nodes, is stage 3. Cancer confined to the organ of origin is stage 1. Cancer that is locally invasive is stage 2. Cancer that has spread to distant sites, such as a liver cancer spreading to lung or a prostate cancer spreading to bone, is stage 4. REF: p. 259, Figure 10-22

A 30-year-old female is diagnosed with cancer. Testing reveals that the cancer cells have spread to local lymph nodes. A nurse realizes this cancer would be documented as stage: a. 1. b. 2. c. 3. d. 4.

ANS: C The underlying alteration in pernicious anemia (PA) is the absence of intrinsic factor (IF), an enzyme required for gastric absorption of dietary vitamin B12, a vitamin essential for nuclear maturation, and DNA synthesis in red blood cells. PA is not due to a decrease in ferritin, gastric enzymes, or erythropoietin but to a lack of intrinsic factor. REF: p. 515

A 35-year-old female is diagnosed with vitamin B12 deficiency anemia (pernicious anemia). The most likely cause is a decrease in: a. ferritin. b. gastric enzymes. c. intrinsic factor. d. erythropoietin.

ANS: C The most likely contributing factor to the patient's stroke is that he has diabetes with a fourfold increase in stroke incidence and an eightfold increase in stroke mortality. Age greater than 65 years is contributing factor. Men are affected, but for the 30-year-old, his type 2 diabetes mellitus contributes to a fourfold increase in stroke incidence and an eightfold increase in stroke mortality. Blacks are affected more than whites, and it is this patient's diabetes that places him at risk. REF: p. 402

A 30-year-old white male recently suffered a cerebrovascular accident. Which of the following is the most likely factor that contributed to his stroke? a. Age b. Gender c. Diabetes d. Race

ANS: C Hodgkin lymphoma is manifested by Reed-Sternberg cells, not Merkel cells, not Schwann cells, and not Kupffer cells. REF: p. 533

A 35-year-old male has enlarged lymph nodes in the neck and a mediastinal mass. He was diagnosed with Hodgkin lymphoma. Which of the following abnormal cells would the nurse expect to find with this disease? a. Merkel cell b. Schwann cell c. Reed-Sternberg cell d. Kupffer cell

ANS: B Basophilia results from antithyroid therapy. Antithroid therapy is not associated with eosinophilia, monocytosis, or lymphocytosis. REF: p. 524, Table 21-4

A 35-year-old male with hyperthyroidism begins treatment to decrease thyroid activity. A nurse monitors for which of the following conditions that could result secondary to the treatment? a. Eosinophilia b. Basophilia c. Monocytosis d. Lymphocytosis

ANS: D Lymphadenopathy is evidenced by enlarged lymph nodes, not small, hard, disordered, or nonpalpable nodes. REF: p. 531

A 35-year-old female is diagnosed with lymphadenopathy. Which assessment finding will help confirm this diagnosis? a. Small, hard lymph nodes b. Disordered lymph nodes c. Nonpalpable, nontender lymph nodes d. Enlarged lymph nodes

ANS: A Duodenal ulcers occur with greater frequency than other types of peptic ulcers and are commonly caused by H. pylori infection and NSAID use. Neither antacids nor caffeinated beverages contribute to ulcer formation. Fiber is important, but consuming limited fiber will not contribute to ulcer formation. REF: p. 916

A 39-year-old is diagnosed with a duodenal ulcer. Which of the following behaviors may have contributed to the development of the ulcer? a. Regular NSAID use b. Drinking caffeinated beverages c. Consuming limited fiber d. Antacid consumption`

ANS: C Sepsis is one of the most common conditions associated with DIC. While snake venom, blood transfusions, and ITP may cause DIC, sepsis is the most likely cause.

A 40-year-old develops disseminated intravascular coagulation (DIC). Upon obtaining the history, which finding is the most likely cause of this condition? a. Snakebite b. Blood transfusion c. Sepsis d. Immune thrombocytopenic purpura (ITP)

ANS: A Sideroblastic anemia is due to ineffective iron uptake and hemoglobin production. Erythrocytes are neither misshapen, premature, nor destroyed in sideroblastic anemia. Sideroblastic anemia is not associated with decreased levels of tissue iron with megaloblastic erthrocytes. REF: pp. 518-519

A 45-year-old male is diagnosed with sideroblastic anemia. When he asks what the most likely cause of this disease is, what is the nurse's best response? a. Ineffective iron uptake and abnormal hemoglobin production b. Misshapen erythrocytes with low hemoglobin c. Decreased levels of tissue iron with megaloblastic erythrocytes d. Premature erythrocyte destruction and erythropoietin deficiency

ANS: C Thyroxine acts on the adrenal medulla, causing the release of epinephrine into the bloodstream. Epinephrine causes vasoconstriction that increases metabolic rates, thus increasing heat production. Heat production does not involve the pancreas, the liver, or the heart. REF: p. 342

A nurse wants to teach about one of the primary organs responsible for heat production. Which organ should the nurse include? a. Pancreas b. Liver c. Adrenal medulla d. Heart

ANS: B With subarachnoid hemorrhage, meningeal irritation occurs, leading to nuchal rigidity. Nuchal rigidity is not associated with a diffuse brain injury, an epidural hematoma, or a classic concussion. REF: pp. 405-406

A 48-year-old patient presents at the ER reporting an acute severe headache, nausea, photophobia, and nuchal rigidity. Which medical diagnosis is supported by these signs and symptoms? a. Diffuse brain injury b. Subarachnoid hemorrhage c. Epidural hematoma d. Classic concussion

ANS: C Chemotherapy, used in various combinations, is the treatment of choice for leukemia. Neither bone marrow transplant, immunotherapy, nor localized radiation therapy is considered the treatment of choice for leukemia. REF: p. 530

A 5-year-old is diagnosed with acute leukemia. The patient will mostly likely be treated with: a. bone marrow transplant. b. immunotherapy. c. chemotherapy. d. localized radiation therapy.

ANS: B Hemolytic anemia is an example of normocytic-normochromic anemia. Sideroblastic anemia is an example of microcytic hypochromic anemia. Pernicious anemia is an example of a macrocytic anemia. Iron deficiency anemia is an example of microcytic hypochromic anemia. REF: p. 514, Table 21-1

A 5-year-old was diagnosed with normocytic-normochromic anemia. Which type of anemia does the nurse suspect the patient has? a. Sideroblastic b. Hemolytic c. Pernicious d. Iron deficiency

ANS: D The retina is the innermost layer of the eye, converting light energy into nerve impulses. Light entering the eye is focused on the retina by the lens. The pupil allows light to enter the eye. The cornea is the portion of the sclera in the central anterior region that allows light to enter the eye. REF: p. 347

A 50-year-old diabetic patient experiences visual disturbances and decides to visit his primary care provider. After examination, the primary care provider tells the patient that the cells that allow him to see are degenerated. Which of the following structures is most likely damaged? a. Lens b. Pupil c. Cornea d. Retina

ANS: A The skin of the patient with sideroblastic anemia is bronze in color. The patient with sideroblastic anemia shows signs of iron overload. The erythrocytes of individuals with sideroblastic anemia are hypochromic. The bone marrow is not aplastic but contains ringed sideroblasts. REF: p. 519

A 50-year-old female was diagnosed with sideroblastic anemia. Which of the following assessment findings would most likely occur? a. Bronze-colored skin b. Decreased iron c. Normochromic erythrocytes d. Aplastic bone marrow

ANS: C Gastroesophageal reflux is due to loss of muscle tone at the lower esophageal sphincter. The resting tone of the lower esophageal sphincter (LES) tends to be lower than normal from either transient relaxation or weakness of the sphincter. Gastroesophageal reflux is not due to fibrosis, stimulation of sympathetic nerves, or reverse peristalsis. REF: p. 911

A 50-year-old is diagnosed with gastroesophageal reflux. This condition is caused by: a. fibrosis of the lower third of the esophagus. b. sympathetic nerve stimulation. c. loss of muscle tone at the lower esophageal sphincter. d. reverse peristalsis of the stomach.

ANS: A Ulcerative colitis is characterized by abdominal pain, fever, elevated pulse rate, frequent diarrhea (10-20 stools/day), urgency, obviously bloody stools, and continuous, crampy pain. Hiatal hernia is most often asymptomatic and would not be manifested by abdominal pain. Pyloric obstruction would be manifested by forceful or projectile vomiting. Achalasia would be manifested by difficulty or uncomfortable swallowing. REF: p. 921

A 50-year-old male reports episodes of frequently recurring crampy abdominal pain, diarrhea, and bloody stools. A possible diagnosis would be: a. ulcerative colitis. b. hiatal hernia. c. pyloric obstruction. d. achalasia.

ANS: C Excessive movement is the definition of hyperkinesia. Hypokinesia is decreased movement. Akinesia is loss of movement. Dyskinesia is abnormal movement. REF: p. 378

A patient has excessive movement. What disorder will the nurse see documented on the chart? a. Hypokinesia b. Akinesia c. Hyperkinesia d. Dyskinesia

ANS: D Jaundice is due to obstruction of the common bile duct. This form of jaundice is not due to obstruction of the intrahepatic canaliculi, gallbladder, or the cystic duct. REF: p. 930

A 55-year-old is diagnosed with extrahepatic obstructive jaundice that is a result of the obstruction of the: a. intrahepatic bile canaliculi. b. gallbladder. c. cystic duct. d. common bile duct.

ANS: A, B, C The pattern of pain is different from that of duodenal ulcers as it frequently occurs immediately after eating. Gastric ulcers cause more anorexia and are associated with NSAID use. Gastric ulcers also tend to be chronic rather than alternating between periods of remission and exacerbation. There appears to be no gender preference. REF: p. 917

A 56-year-old is diagnosed with gastric ulcers. Which of the following characterizes this disorder? (select all that apply) a. Pain occurs immediately after eating b. Anorexia c. History of NSAID use d. Occurrence is typically a single event e. Occur more frequently in females

ANS: A PV is characterized by excessive proliferation of erythrocyte precursors in the bone marrow. Leukemia is not manifested by changes in erythrocytes. Sideroblastic anemia is characterized by sideroblastic rings. Hemosiderosis is iron overload. REF: p. 521

A 57-year-old male presents to his primary care provider for red face, hands, feet, ears, and headache and drowsiness. A blood smear reveals an increased number of erythrocytes, indicating: a. polycythemia vera (PV). b. leukemia. c. sideroblastic anemia. d. hemosiderosis.

ANS: B Pernicious anemia is manifested by tingling paresthesias of feet and fingers. The symptomology is not associated with hemolytic, iron deficiency, or aplastic anemias. REF: pp. 515-516

A 58-year-old female presents in the clinic with fatigue, weight loss, and tingling in her fingers. Laboratory findings show low hemoglobin and hematocrit, a high mean corpuscular volume, and normal plasma iron. These assessment findings are consistent with which type of anemia? a. Hemolytic anemia b. Pernicious anemia c. Iron deficiency anemia d. Aplastic anemia

ANS: B The rate and depth of breathing increase in an effort to increase oxygen availability accompanied by an increase in the release of oxygen from hemoglobin. Bronchodilation occurs, not constriction. Dyspnea is not a compensatory mechanism but a side effect of the body's attempt to increase oxygen. The respiratory system does not activate the renin-angiotensin response; the kidneys are involved. REF: p. 515

A 60-year-old patient diagnosed with emphysema experiences a rapid and pounding heart, dizziness, and fatigue with exertion. Which respiratory assessment findings indicate the respiratory system is compensating for the increased oxygen demand? a. Bronchoconstriction b. Increased rate and depth of breathing c. Dyspnea d. Activation of the renin-angiotensin response

ANS: C The primary causative factor of subarachnoid hemorrhagic bleeding is an aneurysm, not thrombi, which would lead to thrombotic stroke. A thrombi would lead to a CVA from blockage, but not to hemorrhagic bleeding. Hypertension, not hypotension, would lead to a hemorrhagic stroke. Rheumatic heart disease is not associated with subarachnoid hemorrhages. REF: pp. 404-405

A 60-year-old patient with a recent history of head trauma and a long-term history of hypertension presents to the ER for changes in mental status. MRI reveals that the patient has experienced a subarachnoid hemorrhage. What does the nurse suspect caused this type of stroke? a. Rheumatic heart disease b. Thrombi c. Aneurysm d. Hypotension

ANS: B The patient has symptoms of Hodgkin lymphoma. About a third of individuals will have some degree of systemic symptoms. Intermittent fever, without other symptoms of infection, drenching night sweats, itchy skin (pruritus), and fatigue are relatively common. The patient with Epstein-Barr virus does not have night sweats. The patient with acute leukemia does not have night sweats or enlarged lymph nodes. The patient with Burkitt lymphoma does not have enlarged lymph glands or night sweats. REF: p. 533

A 62-year-old reports experiencing regular night sweats and unintentionally losing weight. Physical exam reveals enlarged neck lymph nodes that do not appear to be painful. These findings support a diagnosis of which type of cancer? a. Epstein-Barr virus b. Hodgkin lymphoma c. Acute leukemia d. Burkitt lymphoma

ANS: C When the hemoglobin has decreased to 7-8 grams per deciliter, the individual experiences the classic symptoms of anemia: weakness, fatigue, paresthesias of feet and fingers, difficulty walking, loss of appetite, abdominal pain, weight loss, and a sore tongue that is smooth and beefy red. The skin may become lemon yellow (sallow), which is caused by a combination of pallor and jaundice. Hepatomegaly, indicating right-sided heart failure, may be present in the elderly. The symptoms are not associated with brain or liver hypoxia, or kidney failure. REF: p. 516

A 65-year-old experienced loss of appetite, weight loss, lemon-yellow skin, liver enlargement, and a beefy red tongue shortly before her death. Autopsy suggested pernicious anemia, and the cause of death would most likely reveal: a. brain hypoxia. b. liver hypoxia. c. heart failure. d. kidney failure.

ANS: C With grade III, the patient experiences drowsiness and confusion with or without focal neurologic deficits and pronounced meningeal signs. With grade I, neurologic status is intact with mild headache and slight nuchal rigidity. With grade II, neurologic deficit is evidenced by cranial nerve involvement and moderate-to-severe headache with more pronounced meningeal signs (e.g., photophobia, nuchal rigidity). With grade IV, the patient is stuporous with pronounced neurologic deficits (e.g., hemiparesis, dysphasia) and nuchal rigidity. REF: p. 406, Table 16-6

A 65-year-old patient diagnosed with a subarachnoid hemorrhage secondary to uncontrolled hypertension appears drowsy and confused with pronounced focal neurologic deficits. This symptomology would place this hemorrhage at which grade? a. I b. II c. III d. IV

ANS: C Coronary blood flow may be affected, precipitating angina. Patients will experience signs of interrupted blood flow due to increased blood viscosity. They will not experience hyperactivity. Blood pressure will be increased, not decreased. The skin will be red over the face hands, feet, and ears. REF: p. 521

A 67-year-old female is admitted to the emergency department with a diagnosis of polycythemia vera. Upon taking the history, the patient will most likely report: a. hyperactivity. b. decreased blood pressure. c. chest pain. d. a pale skin color

ANS: D A significant potential outcome of PV is the conversion to acute myeloid leukemia (AML), occurring spontaneously in 10% of individuals and generally being resistant to conventional therapy. Neither chronic lymphocytic leukemia, Burkitt lymphoma, nor multiple myeloma is a significant outcome of PV. REF: p. 522

A 67-year-old male was diagnosed with polycythemia vera (PV) but refused treatment. His condition is at risk for converting to: a. chronic lymphocytic leukemia. b. Burkitt lymphoma. c. multiple myeloma. d. acute myeloid leukemia.

ANS: C The symptoms of PV are due to increased blood viscosity, the erythrocyte count is elevated, there will be decreased blood flow to tissues and organs but vessel injury does not occur. REF: p. 521

A 68-year-old is admitted to the emergency department with a diagnosis of polycythemia vera (PV). A nurse realizes the patient's symptoms are mainly the result of: a. a decreased erythrocyte count. b. rapid blood flow to the major organs. c. increased blood viscosity. d. vessel injury.

ANS: B Chronic subdural hematomas are commonly found in persons who abuse alcohol and develop over weeks to months. A concussion is more acute in nature. Epidural hematomas are not associated with the patient's history or symptoms. Subacute subdural hematomas present with confusion but are more acute in nature. REF: p. 393

A 69-year-old patient with a history of alcohol abuse presents to the emergency room (ER) after a month-long episode of headaches and confusion. The patient's history and symptomology support which medical diagnosis? a. Concussion b. Chronic subdural hematoma c. Epidural hematoma d. Subacute subdural hematoma

. ANS: B Fifty percent of individuals with PV die within 18 months of the onset of initial symptoms because of thrombosis or hemorrhage, not renal failure, not infection and sepsis, and not acute leukemia. REF: pp. 521-522

A 70-year-old is brought to the emergency department, where he dies shortly thereafter. Autopsy reveals polycythemia vera (PV). His death was most likely the result of: a. acute renal failure. b. cerebral thrombosis. c. sepsis. d. acute leukemia.

ANS: C Stage 3 is characterized by decreasing levels of arousal or central neurogenic hyperventilation, widened pulse pressure, bradycardia, and pupils that become small and sluggish. Stage 1 is characterized by an ICP that may not change because of the effective compensatory mechanisms, and there may be few symptoms. Stage 2 is characterized by subtle and transient symptoms, including episodes of confusion, restlessness, drowsiness, and slight pupillary and breathing changes. Stage 4 is characterized by cessation of cerebral blood flow. REF: p. 374

A 70-year-old patient is being closely monitored in the neurological critical care unit for a severe closed head injury. After 48 hours, signs of deterioration occur: pupils are small and sluggish, pulse pressure is widening, and heart rate is bradycardic. These clinical findings are evidence of what stage of intracranial hypertension? a. Stage 1 b. Stage 2 c. Stage 3 d. Stage 4

ANS: B Age-related macular degeneration (AMD) is a severe and irreversible loss of vision and a major cause of blindness in older individuals. Hypertension and cigarette smoking are risk factors. Presbyopia is a condition associated with aging in which the patient experiences reduced near vision. In strabismus, one eye deviates from the other when the person is looking at an object. In amblyopia, vision is reduced in the affected eye caused by cerebral blockage of the visual stimuli. REF: p. 349

A 70-year-old patient presents to the primary care provider reporting loss of vision. A history that includes hypertension and cigarette smoking supports which visual diagnosis? a. Presbyopia b. Macular degeneration c. Strabismus d. Amblyopia

ANS: C When symptoms resolve with complete recovery, it is a transient ischemic attack. A stroke in evolution is an impending stroke, and symptoms would not resolve. An arteriovenous malformation is an abnormal arrangement of blood vessels that could lead to stroke but is not a disorder in itself. Cerebral hemorrhage would not resolve. REF: p. 402

A 72-year-old patient demonstrates left-sided weakness of upper and lower extremities. The symptoms lasted less than an hour and resolved with no evidence of infarction. The patient most likely experienced a(n): a. stroke in evolution. b. arteriovenous malformation. c. transient ischemic attack. d. cerebral hemorrhage.

ANS: B A lacunar stroke is associated with occlusion of a single, deep perforating artery that supplies small penetrating subcortical vessels, causing ischemic lesions, not an embolus, hemorrhage, or aneurysm. REF: p. 403

A 75-year-old patient experienced a lacunar stroke. When looking through the history of the patient's chart, which of the following would the nurse expect to find? a. An embolus b. An ischemic lesion c. A hemorrhage d. An aneurysm

ANS: B Individuals with an open basilar skull fractures should be observed for meningitis. Such a basilar skull fracture does not increase a patient's risk for hematoma formation, ICP, or cognitive deficits. REF: p. 393

A CT scan reveals that a patient has an open basilar skull fracture. Which major complication should the nurse observe for in this patient? a. Hematoma formation b. Meningeal infection c. Increased intracranial pressure (ICP) d. Cognitive deficits

ANS: D The most common cause of chronic mesenteric ischemia is atherosclerosis. Neither poor nutrition nor anemia leads to vascular insufficiency. An aneurysm would lead to acute vascular insufficiency. REF: p. 924

A common cause of chronic mesenteric ischemia among the elderly is: a. anemia. b. aneurysm. c. lack of nutrition in gut lumen. d. atherosclerosis.

ANS: C Autoregulation is the compensatory alteration in the diameter of the intracranial blood vessels designed to maintain a constant blood flow during changes in cerebral perfusion pressure. Herniation is the downward protrusion of the brainstem. Vasodilation is an enlargement in vessel diameter and a part of autoregulation, but the vessels should not dilate in the presence of increased intracranial pressure. Amyotrophy is involved with the anterior horn cells of the spinal cord and not related to autoregulation. REF: p. 374

A compensatory alteration in the diameter of cerebral blood vessels in response to increased intracranial pressure is called: a. herniation. b. vasodilation. c. autoregulation. d. amyotrophy.

ANS: A Glaucoma is the result of increased intraocular pressure. Ocular degeneration results in changes in vision, but not intraocular pressure. Diplopia is double vision. Nystagmus is an involuntary unilateral or bilateral rhythmic movement of the eyes. REF: p. 349

A patient has increased intraocular pressure. Which diagnosis will the nurse observe on the chart? a. Glaucoma b. Ocular degeneration c. Diplopia d. Nystagmus

ANS: B Clotting factors are produced by the liver; the liver is not the site of platelet pooling. Bilirubin does not interfere with clotting. The treatment of liver failure does not affect clotting. REF: p. 544

A nurse checks individuals with liver disease for clotting problems because: a. the liver is often the site of platelet pooling. b. clotting factors are produced in the liver. c. high levels of bilirubin interfere with the clotting system. d. treatment medications for liver failure cause fibrinolysis.

ANS: B The affective-motivational system determines an individual's conditioned avoidance behaviors and emotional responses to pain. The sensory-discriminative system is mediated by the somatosensory cortex and is responsible for identifying the presence, character, location, and intensity of pain. The sensory-motivational system is not a system in the response to pain. The cognitive-evaluative system overlies the individual's learned behavior concerning the experience of pain and can modulate perception of pain. REF: p. 338

A nurse is discussing an individual's conditioned or learned approach or avoidance behavior in response to pain. Which system is the nurse describing? a. Sensory-discriminative system b. Affective-motivational system c. Sensory-motivational system d. Cognitive-evaluative system

ANS: C Early-stage growths that are localized to the epithelium and have not invaded are called cancer in situ. Cancer in situ is early-stage growth and not a tumor in differentiation but is more mature growth. Dysplastic cells do not define cancer in situ. REF: p. 234

A nurse is discussing preinvasive epithelial tumors of glandular or squamous cell origin. What is the nurse describing? a. Tumor in differentiation b. Dysplastic c. Cancer in situ d. Cancer beyond (meta) situ

ANS: B The Eustachian tube connects the middle ear to the pharynx. The organ of Corti contains the hair cells. The semicircular canal is one of the three bones of the labyrinth. The auditory canal leads to the middle ear. REF: p. 351

A nurse is teaching about the structure that connects the middle ear with the pharynx. Which structure is the nurse describing? a. Organ of Corti b. Eustachian tube c. Semicircular canal d. Auditory canal

ANS: B Chronic or persistent pain has been defined as lasting for more than 3-6 months. REF: p. 340

A nurse should document on the chart that chronic pain is occurring when the patient reports the pain has lasted longer than: a. 1 month. b. 3-6 months. c. 1 year. d. 2-3 years.

ANS: A Nociceptors are pain receptors and can be found in the skin. Nociceptors are not located in the spinal cord. Nociceptors are not located in efferent, but afferent, pathways. Nociceptors are not located in the hypothalamus but can be found in the meninges. REF: p. 337, Table 14-1

A patient asks the nurse where nociceptors can be found. How should the nurse respond? One location in which nociceptors can be found is the: a. skin. b. spinal cord. c. efferent pathways. d. hypothalamus.

ANS: C The described symptoms indicate autonomic hyperreflexia and are due to a distended bladder or rectum. The described symptoms are not due to the accumulation of free radicals, pain stimulation, or an abnormal vagal response. REF: pp. 398-399

A patient diagnosed with a spinal cord injury experienced spinal shock lasting 15 days. The patient is now experiencing an uncompensated cardiovascular response to sympathetic stimulation. What does the nurse suspect caused this condition? a. Toxic accumulation of free radicals below the level of the injury b. Pain stimulation above the level of the spinal cord lesion c. A distended bladder or rectum d. An abnormal vagal response

ANS: D Central neurogenic hyperventilation is a sustained, deep, rapid, but regular, pattern (hyperpnea) of breathing. Gasping is a pattern of deep "all-or-none" breaths accompanied by a slow respiratory rate. Ataxic breathing is completely irregular breathing that occurs with random shallow and deep breaths and irregular pauses. Apneusis is manifested by a prolonged inspiratory pause alternating with an end-expiratory pause. REF: p. 362, Table 15-4

A patient experiences a severe head injury hitting a tree while riding a motorcycle. Breathing becomes deep and rapid but with normal pattern. What term should the nurse use for this condition? a. Gasping b. Ataxic breathing c. Apneusis d. Central neurogenic hyperventilation

ANS: D Occlusion of the left middle cerebral artery leads to the inability to find words and difficulty with writing. The inability to find words and difficulty with writing are not associated with occlusions of the anterior or posterior communicating arteries or the circle of Willis. REF: p. 367

A patient experiences a stroke and now has difficulty writing and producing language. This condition is most likely caused by occlusion of the: a. anterior communicating artery. b. posterior communicating artery. c. circle of Willis. d. middle cerebral artery

ANS: C Retrograde amnesia is manifested by loss of past personal history memories or past factual memories. In selective memory deficit, the person reports inability to focus attention and has failure to perceive objects and other stimuli. Anterograde amnesia is a loss of the ability to form new memories. Executive memory deficit involves the failure to stay alert and oriented to stimuli. REF: p. 365

A patient has memory loss of events that occurred before a head injury. What cognitive disorder does the nurse suspect the patient is experiencing? a. Selective memory deficit b. Anterograde amnesia c. Retrograde amnesia d. Executive memory deficit

ANS: A, B, C Causes of microcytic hypochromic anemia are decreased erythrocyte life span, failure of mechanisms of compensatory erythropoiesis, or disturbance of the iron cycle. Increased metabolic rate and swelling of tissues does not lead to anemia. REF: p. 514, Table 21-1

A patient has microcytic hypochromic anemia. Which of the following pathogenic mechanisms may cause anemia in this patient? (select all that apply) a. Decreased erythrocyte life span b. Failure of mechanisms of compensatory erythropoiesis c. Disturbances of the iron cycle d. Increased basal metabolic rate e. Swelling in the tissues

ANS: A Paraplegia is the paralysis of both legs. Quadriplegia is the paralysis of all four extremities. Infraparaplegia is not a description of paralysis. Paresthesia is a loss of sensation, not paralysis. REF: p. 382, Box 15-6

A patient has paralysis of both legs. What type of paralysis does the patient have? a. Paraplegia b. Quadriplegia c. Infraparaplegia d. Paresthesia

ANS: A Treatment of PV is phlebotomy to reduce red cell mass and blood volume; increasing blood volume will increase symptoms as will the administration of additional cells and iron replacement therapy. REF: p. 522

A patient is admitted to the emergency department with a diagnosis of polycythemia vera (PV). Which treatment should the nurse discuss with the patient? a. Therapeutic phlebotomy b. Restoration of blood volume by plasma expanders c. Administration of packed red blood cells d. Iron replacement therapy

ANS: B The pathophysiology of MS includes demyelination of nerve fibers. Depletion of dopamine is related to Parkinson disease. The development of neurofibrils is related to Alzheimer disease. Myasthenia gravis is due to decreased amounts of acetylcholine at the junction. REF: p. 411

A patient is newly diagnosed with multiple sclerosis (MS). What physiological change is causing the patient's symptoms? a. Depletion of dopamine in the central nervous system (CNS) b. Demyelination of nerve fibers in the CNS c. The development of neurofibril webs in the CNS d. Reduced amounts of acetylcholine at the neuromuscular junction

ANS: C REM sleep is called paradoxical sleep because the EEG pattern is similar to the normal awake pattern and the brain is very active with dreaming. Neither non-REM, fast wave sleep, nor delta wave sleep produces EEG patterns similar to the normal awake pattern. REF: p. 345

A patient is undergoing a sleep lab test. When the sleep lab worker notices EEG patterns with brain activity similar to the normal awake pattern, which phase of sleep is occurring? a. Non-rapid eye movement (REM) b. Fast wave c. REM d. Delta wave

ANS: B Bacterial meningitis can occur secondary to sinusitis and is manifested by fever, tachycardia, chills, and a petechial rash with a severe throbbing headache, severe photophobia, and nuchal rigidity. The clinical manifestations of aseptic meningitis are milder than bacterial meningitis and are not associated with a previous infection such as sinusitis. Fungal meningitis presents as dementia. Nonpurulent meningitis is the same as aseptic and is milder and not associated with conditions such as sinusitis. REF: p. 409

A patient presents to a primary care provider reporting fever, headache, nuchal rigidity, and decreased consciousness. History includes a previously treated sinusitis. Which medical diagnosis is best supported by this assessment data? a. Aseptic meningitis b. Bacterial meningitis c. Fungal meningitis d. Nonpurulent meningitis

ANS: B, C, E Functional constipation is a type of or primary constipation that is triggered by a low residue diet and low fluid intake. A sedentary lifestyle and lack of regular exercise are other common causes of this type of constipation. Secondary constipation is a result of a pre-existing condition that may be a neurogenic disorder or colon cancer. REF: p. 907

A patient presents to the physician with reports infrequent bowel movements and is diagnosed with functional constipation. What can be a contributing factor to this condition? (select all that apply) a. Neurogenic disorder b. Sedentary lifestyle c. Low residue diet d. Colon cancer e. Low fluid intake

ANS: C Spondylolisthesis occurs when there are vertebra slides forward or slips in relation to below it. Degenerative disk disease is a pathophysiological cause of spondylolisthesis but is not the definition of the displacement. Spondylolysis is a structural defect of the spine. Spinal stenosis is a narrowing of the spinal canal. REF: p. 400 | p. 401

A patient presents with acute low back pain. There is no history of trauma. An MRI reveals that the vertebra at L5 has slipped forward relative to those below it. Which of the following conditions will be documented on the chart? a. Degenerative disk disease b. Spondylolysis c. Spondylolisthesis d. Spinal stenosis

ANS: A Meningioma tumors usually originate from the arachnoidal (meningeal) cap cells in the dura mater. Astrocytes are found in the brain but are not related to meningiomas. The pia mater is the location of the infection meningitis. Neurons are located throughout all regions of the brain. REF: p. 417

A patient presents with seizures. An MRI reveals a meningioma most likely originating from the: a. dura mater and arachnoid membrane. b. astrocytes. c. pia mater. d. CNS neurons.

ANS: C Epinephrine causes vasoconstriction, stimulates glycolysis, and increases metabolic rate, thus increasing secondary heat production. Epinephrine does not lead to decreased vascular tone or increased skeletal muscle tone but does increase metabolic rate. REF: p. 342

A patient received a prescription for a weight loss pill. One effect of the pills is to increase the release of epinephrine. Which of the following would be expected to also occur? a. Decreased vascular tone b. Increased skeletal muscle tone c. Increased heat production d. Decreased basal metabolic rate

ANS: C Somatic pain is superficial, arising from the skin. It is typically well localized and described as sharp, dull, aching, or throbbing. Chronic pain has been defined as lasting for more than 3-6 months. Referred pain is felt in an area removed or distant from its point of origin; the area of referred pain is supplied by the same spinal segment as the actual site of pain. Visceral pain is pain in internal organs and lining of body cavities and tends to be poorly localized, with an aching, gnawing, throbbing, or intermittent cramping quality. REF: p. 340

A patient scrapes both knees while playing soccer and reports sharp and well-localized pain. Which of the following should the nurse document to most accurately characterize the pain? a. Chronic pain b. Referred pain c. Somatic pain d. Visceral pain

ANS: C, D, E Hepatitis C or HIV can predispose the patient to leukemia, not CMV. Drugs that cause bone marrow depression, such as chemotherapeutic agents, predispose an individual to leukemia; eating genetically modified food is not a predisposing factor. Excessive ultraviolet radiation exposure can predispose an individual to myelogenous leukemia. Acute leukemia may also develop secondary to certain acquired disorders, including ovarian cancer. REF: p. 527

A patient wants to know about risk factors for acute leukemia. Which of the following should the nurse include? (select all that apply) a. Cytomegalovirus (CMV) infection b. Eating genetically modified food c. Chemotherapy treatment for other cancers d. Excessive ultraviolet radiation exposure e. Ovarian cancer

ANS: A Delirium and the inability to concentrate are characteristics of acute confusional state. Echolalia is the repeating of words and phrases. Dementia is characterized by loss of recent and remote memory. Dysphagia is difficulty speaking. REF: p. 367, Box 15-3

A patient with an addiction to alcohol checked into a rehabilitation center as a result of experiencing delirium, inability to concentrate, and being easily distracted. What term would be used to document this state? a. Acute confusional state b. Echolalia c. Dementia d. Dysphagia

ANS: C Tumor markers include hormones, enzymes, genes, antigens, and antibodies, but not red blood cells, apoptotic cells, or neurotransmitters. REF: p. 258

A primary care provider is attempting to diagnose cancer and is looking for a tumor marker. Which of the following could be a possible marker? a. Red blood cells b. Apoptotic cells c. Enzymes d. Neurotransmitters

Which of the following is the largest fluid compartment in the body? A) Intracellular B) Interstitial C) Intravascular D) Peritoneal

A) Intracellular

Which of the following cells has the capacity to produce antibodies during an immune response? A) Plasma cells B) T cells C) Memory cells D) Pluripotent cells

A) Plasma cells

Passive mediated transport (facilitated diffusion) depends on the presence of: A) carrier proteins in the plasma membrane. B) energy in the form of ATP. C) microtubules in the cytoplasm. D) all of the above.

A) carrier proteins in the plasma membrane.

Factors that determine osmotic pressure include all of the following except: A) charge of the molecules. B) size of the molecules. C) concentration gradient. D) thickness of the plasma membrane.

A) charge of the molecules.

Edema can result from all of the following alterations except: A) decreased capillary hydrostatic pressure. B) decreased capillary oncotic pressure. C) lymphatic obstruction. D) increased capillary membrane permeability.

A) decreased capillary hydrostatic pressure.

Symptoms of dehydration include: A) decreased urine output. B) increased skin turgor. C) increased blood pressure. D) edema.

A) decreased urine output.

CD4 receptors that bind to the surface of macrophages and B cells are found on: A) helper T cells. B) cytotoxic T cells. C) plasma cells. D) the human immunodeficiency virus (HIV).

A) helper T cells.

Classic symptoms of diabetes insipidus (DI) include all of the following except: A) hypertension. B) dehydration. C) low urine osmolarity. D) thirst.

A) hypertension. Absence of ADH secretion in DI leads to increased urine output, volume depletion, and hypotension.

Respiratory alkalosis is caused by: A) hyperventilation. B) pneumonia. C) pulmonary congestion. D) pulmonary edema.

A) hyperventilation.

High levels of glucocorticoids such as cortisol can result in: A) immunosuppression. B) hypoglycemia. C) weight loss. D) all of the above.

A) immunosuppression.

Referring to cancer, the term anaplasia means: A) loss of cellular differentiation. B) rapid growth of cells. C) independence from normal cellular controls. D) ectopic production of hormones.

A) loss of cellular differentiation.

The level of thyroid-stimulating hormone (TSH) in Graves disease is: A) low. B) high. C) normal. D) variable.

A) low. In Graves disease, the presence of TSI mimics the activity of TSH at the thyroid receptor. The result is increased thyroid hormone production that suppresses TSH production from the anterior pituitary.

Amyotrophic lateral sclerosis (ALS) is caused by the degeneration of: A) motor neurons in the spinal cord and peripheral nerves. B) skeletal muscle tissue. C) myelin in the peripheral nervous system. D) myelin in the central nervous system.

A) motor neurons in the spinal cord and peripheral nerves.

Which of the following assessment findings would be expected in pulmonary embolism (PE)? (Select all that apply.) A. Tachypnea B. Chest pain C. Hemoptysis D. Tachycardia E. Fever

A. Tachypnea B. Chest pain D. Tachycardia

Upon assessment, which of the following would the nurse expect to find in a patient with local inflammation? a. edema b. pallor c. bruising d. necrosis

A. edema

pituitary secretes

ACTH

An 8-year-old female presents with edema of the cutaneous and mucosal tissue layers. Her mother reports that the condition is recurrent and seems to occur more often during stressful situations. The child is diagnosed with hereditary angioedema. Which of the following is deficient in this child? a. C1 esterase inhibitor b. Carboxypeptidase c. Neutrophils d. Plasmin

ANS: A A genetic defect in C1 esterase inhibitor (C1 INH deficiency) results in hereditary angioedema. Hereditary angioedema is due to C1 esterase inhibitor, not neutrophils. Carboxypeptidase degrades kinins. Plasmin is associated with clots. REF: p. 141

What term is used to identify the period that follows depolarization of the myocardium and represents a period during which no new cardiac potential can be propagated? a. Absolute refractory b. Hyperpolarization c. Resting d. Threshold

ANS: A A refractory period is the time during which no new cardiac action potential can be initiated by a stimulus. It follows depolarization. Neither hyperpolarization nor resting correctly identifies this period. Threshold is related to depolarization. REF: p. 577

After activation of the Bainbridge reflex in a patient, the nurse assesses for: a. increased heart rate. b. decreased blood pressure. c. increased rate and depth of respirations. d. decreased myocardial contractility.

ANS: A Activation of the Bainbridge reflex increases heart rate. It does not decrease blood pressure, it does not increase rate and depth of respirations, and it does not decrease myocardial contractility. REF: p. 583

Which patient will develop active immunity? A patient who: a. has natural exposure to an antigen or receives an immunization. b. receives preformed antibodies or T cells from a donor. c. has T cells that become B cells. d. receives immunoglobulin.

ANS: A Active immunity occurs either after natural exposure to an antigen or after immunization, not with preformed antibodies or the transformation of T cells into B cells or as a result of receiving immunoglobulin. REF: p. 159

A 60-year-old with a 25-year history of smoking is diagnosed with emphysema. Assessment shows an increased anterior-posterior chest diameter. The nurse attributes this finding to: a. air trapping. b. decreased inspiratory reserve volumes. c. increased flow rates. d. alveolar destruction.

ANS: A Air trapping, not increased flow rates, expands the thorax, putting the respiratory muscles at a mechanical disadvantage. Neither decreased inspiratory reserve volumes nor alveolar destruction is associated with an increased anterior-posterior chest diameter. REF: p. 701

Which of the following lab results would indicate a slowdown in the rate of parathyroid hormone secretion? a. Increased serum calcium levels b. Decreased serum calcium levels c. Decreased levels of TSH d. Increased levels of TSH

ANS: A An increase, not a decrease, in serum calcium inhibits parathyroid hormone (PTH) secretion. Thyroid-stimulating hormone would not affect PTH secretion. REF: p. 450

Muscle contractions occur when there is a decreased: a. distance between Z lines. b. A band length. c. Z line length. d. H zone distance.

ANS: A Anatomically, contraction occurs when the sarcomere shortens, so adjacent Z lines move closer together. Such a contraction does not occur in relationship to a decrease in A band or Z line length or a decrease in H zone distance. REF: p. 581

A patient has researched bradykinin on the Internet. Which information indicates the patient understands the functions of bradykinin? Bradykinin is involved in: a. increasing vascular permeability. b. vasoconstricting blood vessels. c. stimulating the clotting system. d. increasing degradation of prostaglandins.

ANS: A Bradykinin increases vascular permeability. It is not associated with vasoconstriction, stimulating the clotting system, or increasing degradation of prostaglandins. REF: p. 141

One difference between cardiac muscle and skeletal muscle is that: a. cardiac muscle cells are arranged in branching networks. b. skeletal muscle cells have only one nucleus. c. cardiac muscle cells appear striped. d. skeletal muscle cells contain sarcomeres.

ANS: A Cardiac cells are arranged in branching networks throughout the myocardium, whereas skeletal muscle cells tend to be arranged in parallel units throughout the length of the muscle. Cardiac muscle cells have only one nucleus, whereas skeletal muscle cells have many nuclei. Both cardiac and muscle cells appear striped and contain sarcomeres. REF: p. 578

A 10-year-old male is brought to the emergency room (ER) incoherent and semiconscious. CT scan reveals that he is suffering from cerebral edema. This type of edema is referred to as: a. localized edema. b. generalized edema. c. pitting edema. d. lymphedema.

ANS: A Cerebral edema is a form of localized edema. Generalized edema is manifested by a more uniform distribution of fluid in interstitial spaces. When pressure is applied, pitting edema results in an indention in the skin. Lymphedema is due to swelling in interstitial spaces, primarily in the extremities. REF: p. 116

As a result of a severe head injury, a patient is now experiencing respiratory abnormalities characterized by alternating periods of deep and shallow breathing with periods of apnea. What term should the nurse use when charting this condition? a. Cheyne-Stokes b. Frank-Starling c. Apnea d. Orthopnea

ANS: A Cheyne-Stokes respirations are characterized by alternating periods of deep and shallow breathing, with periods of apnea lasting from 15 to 60 seconds. Frank-Starling is related to the stretch of fibers. Apnea is cessation of respirations. Orthopnea is dyspnea that occurs when an individual lies flat. REF: p. 688

A 22-year-old presents with chronic bronchitis. Tests reveal closure of the airway during expiration. This condition is most likely caused by: a. thick mucus from hypertrophied glands. b. ventilation-perfusion mismatch. c. hyperventilation. d. thinning smooth muscle in the bronchioles.

ANS: A Chronic bronchitis is defined by hypersecretion of thick mucus. Ventilation-perfusion mismatch may occur, but chronic bronchitis is defined as hypersecretion of mucus. Neither hyperventilation nor thinning of smooth muscle occurs. REF: p. 701

A 65-year-old diagnosed with emphysema presents to the ER for difficulty breathing. Physical exam reveals both bluish skin and mucous membranes. What term will the nurse use to document these observations? a. Cyanosis b. Hemoptysis c. Hematemesis d. Ischemia

ANS: A Cyanosis is a blue color to the skin. Hemoptysis is the coughing up of blood or bloody secretions, and hematemesis is blood in the vomitus. Ischemia is a lack of blood supply to tissues. REF: p. 688

A patient presents reporting visual disturbances. When CT reveals a pituitary tumor and lab tests reveal elevated prolactin, the diagnosis of prolactinoma is made. Which intervention is the treatment of choice for this condition? a. Dopaminergic agonists b. Calcium c. Insulin d. Radiation

ANS: A Dopaminergic agonists (bromocriptine and cabergoline) are the treatment of choice for prolactinomas. Calcium is used to treat parathyroid disease. Insulin is used to treat diabetes. Radiation is not used to treat prolactionomas. REF: p. 466

Besides hyposecretion and hypersecretion, endocrine system dysfunction can result from: a. abnormal receptor activity. b. abnormal hormone levels. c. increased synthesis of second messengers. d. extracellular electrolyte alterations.

ANS: A Dysfunction may result from abnormal cell receptor function or from altered intracellular response to the hormone-receptor complex. Abnormal hormone levels can occur but are not the cause of endocrine dysfunction. Intracellular storage of second-messenger hormones would not lead to dysfunction; receptor function does. Extracellular electrolyte alterations may result from dysfunction, but they are not a cause. REF: p. 460

An infant develops a fever secondary to a bacterial infection. Which of the following most likely triggered the fever? a. Interleukin-1 b. Interleukin-6 c. Interleukin-10 d. Interferons (IFNs)

ANS: A Interleukin-1 is responsible for fever production. Interleukin-6 stimulates growth and differentiation of blood cells. Interleukin-10 helps decrease the immune response. INFs are members of a family of cytokines that protect against viral infections. REF: p. 143, Figure 6-6

How do lipid-soluble hormone receptors cross the plasma membrane? a. Diffusion b. Osmosis c. Active transport d. Endocytosis

ANS: A Lipid-soluble hormones cross by diffusion, not by osmosis, active transport, or endocytosis. REF: p. 443 | p. 444, Figure 18-6

A patient is admitted to the medical unit for complications of long-term, poorly controlled type 2 DM. Which of the following would the nurse expect to find in addition to elevated glucose? a. Atherosclerosis testbanks_and_xanax b. Metabolic alkalosis c. Elevated liver enzymes d. Anemia

ANS: A Macrovascular disease (lesions in large and medium-sized arteries) increases morbidity and mortality and increases risk for accelerated atherosclerosis. Acidosis, rather than alkalosis, would occur in this patient. Neither elevated liver enzymes nor anemia would be expected. REF: p. 480

A 56-year-old presents to his primary care provider for a checkup. Physical exam reveals edema, hepatomegaly, and muffled heart sounds. Which of the following is of greatest concern to the nurse? a. Tamponade b. Exudate c. Aneurysm d. Pulsus paradoxus

ANS: A Muffled heart sounds are an indication of tamponade, and with tamponade the blood backs up into the venous system, leading to hepatomegaly. Muffled heart sounds with hepatomegaly are symptoms of tamponade, not exudates. An aneurysm would present without symptoms. Pulsus paradoxus is manifested by a change in blood pressure during inspiration and expiration. REF: p. 623

A 54-year-old female is diagnosed with nephritic syndrome. Which of the following is a common symptom of this disease? a. Hematuria b. Dysuria c. Oliguria d. Proteinuria

ANS: A Nephritic syndrome is hematuria and red blood cell casts in the urine. Proteinuria is usually less severe than in nephrotic syndrome. The patient with nephritic syndrome does not experience dysuria or oliguria. REF: p. 759

Water movement between the ICF and ECF compartments is determined by: a. osmotic forces. b. plasma oncotic pressure. c. antidiuretic hormone. d. buffer systems.

ANS: A Osmotic forces determine water movement between the ECF and ICF compartments. Oncotic pressure pulls water at the end of the capillary, which makes it move between intra and extra as interstitial is considered extra. The antidiuretic hormone regulates water balance, which would make water move between the intra- and extracellular spaces. Buffer systems help regulate acid balance. REF: p. 115

A nurse is discussing the pressure generated at the end of diastole. Which term is the nurse describing? a. Preload b. Afterload c. Systemic vascular resistance d. Total peripheral resistance

ANS: A Preload, not afterload, is the volume and associated pressure generated in the ventricle at the end of diastole. Afterload is the resistance to ejection of blood from the left ventricle. Systemic vascular resistance is related to afterload. Total peripheral resistance increases afterload. REF: p. 581

A 25-year-old female has sexual relations with her boyfriend. Later she is told that the boyfriend is infected with Neisseria gonorrhoeae. Testing reveals that she does not have the disease. How is it possible that she did not contract the disease? a. Antibodies covered sites of attachment. b. She was vaccinated against it. c. Platelets provided protection. d. IgE was released

ANS: A Some bacteria, such as Neisseria gonorrhoeae that causes gonorrhea, must attach to specific sites on urogenital epithelial cells. Antibodies may protect the host by covering sites on the microorganism that are needed for attachment, thereby preventing infection. Neither a vaccination, the protection of platelets, nor the release of IgE was relevant to the client's ability to avoid contracting this disease. REF: pp. 162-163

A 51-year-old male is at the health clinic for an annual physical exam. After walking from the car to the clinic, he developed substernal pain. He also reported discomfort in his left shoulder and his jaw, lasting 2-3 minutes and then subsiding with rest. He indicates that this has occurred frequently over the past few months with similar exertion. The nurse suspects he is most likely experiencing: a. Stable angina b. Unstable angina c. Prinzmetal angina d. Myocardial infarction (MI)

ANS: A Stable angina is associated with activity and subsides with rest. Unstable angina is a form of acute coronary syndrome that results from reversible myocardial ischemia and occurs at rest. Chest pain that occurs at rest and at night is descriptive of Prinzmetal angina. MI pain does not subside with rest. REF: p. 614

The complement, clotting, and kinin systems share which of the following characteristics? a. Activation of a series of proenzymes b. Phagocytosis initiation c. Granulocyte production d. Activated by interferon

ANS: A The complement system, the clotting system, and the kinin system are normally in inactive forms but can activate in a series as proenzymes and are involved in the inflammatory process. This system is not associated with phagocytosis, granulocyte production, or the activation of interferon. REF: p. 138

A 57-year-old presents with cough, sputum production, dyspnea, and decreased lung volume and is diagnosed with pneumoconiosis. When taking the patient's history, which finding is the most probable cause of the illness? a. Inhalation of silica b. Autoimmune disease c. Allergic reactions d. Flail chest

ANS: A The dusts of silica, asbestos, and coal are the most common causes of pneumoconiosis. Pneumoconiosis is neither an autoimmune nor an allergic disorder; it is not due to flail chest. REF: p. 695

A 20-year-old male received a knife wound to the arm during an altercation. Which of the following types of immunity was compromised? a. Innate immunity b. Inflammatory response c. Adaptive immunity d. Specific immunity

ANS: A The epithelial cells of the skin are a part of innate immunity. The inflammatory response is not a type of immunity. Adaptive immunity is represented by the normal flora of the bowel. Specific immunity is a type of adaptive immunity and is not associated with a break in skin integrity. REF: pp. 134-135

For a patient with respiratory acidosis, chronic compensation by the body will include: a. kidney excretion of H+. b. kidney excretion of HCO3. c. prolonged exhalations to blow off CO2. d. protein buffering.

ANS: A The kidneys excrete H+ to compensate for respiratory acidosis. The kidneys do not excrete HCO3 to compensate; this would increase acidosis. Prolonged exhalations would not be effective for compensation, especially in a chronic state. Protein buffering is intracellular and would not be effective enough to compensate for respiratory acidosis. REF: pp. 128-130

While planning care for a patient with renal calculi, the nurse remembers the most important factor in renal calculus formation is: a. urine pH. b. body temperature. c. gender. d. serum mineral concentrations.

ANS: A The most important factor in renal calculus formation is urine pH, not gender, although calculi form more often in men. Neither body temperature nor serum mineral concentrations are as important as urine pH. REF: p. 749

The organism that causes tuberculosis is a: a. bacterium. b. fungus. c. virus. d. parasite.

ANS: A The organism that causes tuberculosis is a bacterium. Tuberculosis is not a fungus, a virus, or a parasite. REF: p. 706

A 42-year-old presents with dyspnea; rapid, shallow breathing; inspiratory crackles; decreased lung compliance; and hypoxemia. Tests reveal a fulminant form of respiratory failure characterized by acute lung inflammation and diffuse alveolocapillary injury. What is the most likely diagnosis supported by the patient's condition? a. Acute respiratory distress syndrome (ARDS) b. Sarcoidosis c. Postoperative respiratory failure d. Malignant respiratory failure

ANS: A The patient is experiencing ARDS. The patient's symptoms do not support a diagnosis of sarcoidosis or either postoperative or malignant respiratory failure. REF: p. 696

If the patient has a problem with the pineal gland, which substance would the nurse monitor? a. Melatonin b. Epinephrine c. Cortisol d. Somatostatin

ANS: A The pineal glands secrete melatonin. The adrenal medulla secretes epinephrine. The adrenal cortex secretes cortisol. The thyroid gland secretes somatostatin. testbanks_and_xanax REF: p. 448

An 80-year-old develops pneumonia in the hospital. An assessment identifies that the patient is cyanotic and tachycardic and has developed a fever and a cough. Chest x-ray reveals pus in the pleural space. This symptomology supports which medical diagnosis? a. Empyema b. Emphysema c. Pleurisy d. Chyle

ANS: A The presence of pus in the pleural space is termed empyema; emphysema is a total lung disorder. Pleurisy causes pain with inspiration. Chyle is milky fluid with lymph and fat. REF: p. 692

A 55-year-old female presents to her primary care provider and reports dizziness, confusion, and tingling in the extremities. Blood tests reveal an elevated pH, decreased PCO2, and slightly decreased HCO3. Which of the following is the most likely diagnosis? a. Respiratory alkalosis with renal compensation b. Respiratory acidosis with renal compensation c. Metabolic alkalosis with respiratory compensation d. Metabolic acidosis with respiratory compensation

ANS: A With an elevated pH, the diagnosis must be alkalosis. Since the PCO2 is low, it is likely respiratory, with a slight decrease in HCO3 indicating renal compensation. REF: p. 130

A 29-year-old female presents with cloudy urine, flank pain, and hematuria. These signs and symptoms support which diagnosis? a. Acute cystitis b. Renal calculi c. Chronic renal failure d. Postrenal renal failure

ANS: A testbanks_and_xanax The patient is demonstrating symptoms of acute cystitis. Although renal calculi can cause pain and hematuria, they are not manifested by fever and cloudy urine. Chronic renal failure is not manifested by cloudy urine. Postrenal renal failure is not manifested by hematuria and cloudy urine. REF: p. 754

Direct effects of antibodies include: (select all that apply) a. neutralization. b. agglutination. c. precipitation. d. phagocytosis. e. division

ANS: A, B, C Directly, antibodies can affect infectious agents or their toxic products by neutralization (inactivating or blocking the binding of antigen to receptors), agglutination (clumping insoluble particles that are in suspension), or precipitation (making a soluble antigen into an insoluble precipitate). Indirectly, antibodies activate components of innate resistance, including complement and phagocytes. Antibodies are generally a mixed population of classes, specificities, and capacity to provide the functions listed above. REF: p. 162

Physical barriers that offer the body protection from damage and infection are located in the: (select all that apply) a. gastrointestinal tract. b. genitourinary tract. c. respiratory tract. d. lymph system. e. hematopoietic system.

ANS: A, B, C The physical barriers that cover the external parts of the human body offer considerable protection from damage and infection. These barriers are composed of tightly associated epithelial cells of the skin and of the linings of the gastrointestinal, genitourinary, and respiratory tracts. REF: p. 135

A nurse recalls acute orthostatic hypotension can be caused by: (select all that apply) a. prolonged immobility. b. drug action. c. starvation. d. volume depletion. e. exercise.

ANS: A, B, C, D Acute orthostatic hypotension occurs as a result of drug action, prolonged immobility, starvation, and volume depletion. Physical exhaustion, rather than exercise, could cause orthostatic hypotension. REF: p. 604

Blood flow is affected by: (select all that apply) a. blood viscosity. b. blood vessel diameter. c. blood pressure. d. blood vessel length. e. blood composition.

ANS: A, B, C, D Resistance to flow is generally greater in longer tubes because resistance increases with length but decreases with a wider diameter. Blood flow varies inversely with the viscosity of the fluid. Thick fluids move more slowly and experience greater resistance to flow than thin fluids. Increased blood pressure decreases blood flow because resistance is increased. Blood volume, not composition, plays a role in blood flow. REF: p. 587

Which assessment finding would be expected in pulmonary embolism (PE)? (select all that apply) a. Chest pain b. Tachycardia c. Tachypnea d. Fever e. Hemoptysis

ANS: A, B, C, D, E An individual with PE usually presents with the sudden onset of pleuritic chest pain, dyspnea, tachypnea, tachycardia, and unexplained anxiety. Occasionally syncope (fainting) or hemoptysis occurs. With large emboli, a pleural friction rub, pleural effusion, fever, and leukocytosis may be noted. Recurrent small emboli may not be detected until progressive incapacitation, precordial pain, anxiety, dyspnea, and right ventricular enlargement are exhibited. Massive occlusion causes severe pulmonary hypertension and shock. REF: p. 707

A nurse remembers that the primary actions of the complement cascade include: (select all that apply) a. increased vascular permeability. b. vasoconstriction. c. chemotaxis. d. opsonization. e. cell killing. f. increased clotting

ANS: A, C, D, E The actions of the complement cascade include increased vascular permeability and vasodilation, chemotaxis, opsonization, and cell killing. They do not involve vasoconstriction or increased clotting. REF: pp. 138-139

A nurse is evaluating stroke volume. Which of the following factors affect stroke volume? (select all that apply) a. Preload b. Peripheral vascular resistance c. Afterload d. Ejection fraction e. Contractility

ANS: A, C, E Preload, afterload, and contractility affect stroke volume. REF: p. 581

Mechanisms for defense against urinary pathogens in men include: (select all that apply) a. the long length of the urethra. b. the alkaline pH of urine. c. the secretion of mucus that traps bacteria. d. the antimicrobial secretions from the prostate. e. the implantation of the ureters in the bladder.

ANS: A, D Both the longer urethra and prostatic secretions decrease the risk of infection in men. The urine is not more alkaline, and they do not secrete mucus that traps bacteria. The ureters in men and women are implanted in similar positions and in their normal position; it is not a factor in the development of cystitis. REF: p. 753

A patient has a disease state that results from the secretion of toxins by bacteria. Which medical diagnosis will the nurse see documented on the chart? a. Malaria b. Tetanus c. Smallpox d. Hepatitis

ANS: B Some bacteria secrete toxins that harm individuals. For instance, specific bacterial toxins cause the symptoms of tetanus or diphtheria. This is not true of malaria, smallpox, or hepatitis. REF: p. 163

The most common cause of myocardial ischemia is: a. idiopathic vasospasm. b. arterial emboli from a heart valve. c. atherosclerosis. d. venous emboli.

ANS: C The most common cause of myocardial ischemia is atherosclerosis. Myocardial ischemia is not caused by idiopathic vasospams or venous emboli. Arterial emboli may cause ischemia, but atherosclerosis is the major cause of myocardial ischemia. REF: p. 610

A 35-year-old who was severely burned is now demonstrating symptomology associated with acute tubular necrosis (ATN). Which form of renal failure is this patient experiencing? a. Prerenal b. Intrarenal c. Extrarenal d. Postrenal

ANS: B Intrarenal acute kidney failure is associated with several systemic diseases but is commonly related to ATN. Prerenal renal failure occurs anterior to the kidney. Extrarenal renal failure occurs outside the kidney. Postrenal is due to diseases that obstruct the flow of urine from the kidneys. REF: p. 761

An experiment is designed to determine specific cell types involved in cell-mediated immune response. The experimenter is interested in finding cells that attack cells that have specific antigens. Which cells should be isolated? a. Lymphokine-producing cells b. T-cytotoxic cells c. Helper T cells d. Macrophages

ANS: B Cell-mediated immunity is driven by T-cytotoxic (Tc) cells that attack antigens directly and destroy cells that bear foreign antigens. Lymphokine-producing cells, helper T cells, and macrophages do not attack antigens directly and destroy cells that bear foreign antigens. REF: p. 159

A nurse is teaching the staff about antidiuretic hormone (ADH). Which information should the nurse include? Secretion of ADH is stimulated by: a. increased serum potassium. b. increased plasma osmolality. c. decreased renal blood flow. d. generalized edema.

ANS: B ADH is secreted when plasma osmolality increases or circulating blood volume decreases and blood pressure drops. ADH is not secreted by an increase in potassium, a decrease in renal blood flow, or the presence of generalized edema. REF: p. 118

Acute poststreptococcal glomerulonephritis is primarily caused by: a. swelling of mesangial cells in the Bowman space in response to the presence of bacteria. b. immune complex deposition in the glomerular capillaries and inflammatory damage. c. inflammatory factors that stimulate cellular proliferation of epithelial cells. d. accumulation of antiglomerular basement membrane antibodies.

ANS: B Acute poststreptococcal glomerulonephritis is caused by an antigen-antibody complex. Acute poststreptococcal glomerulonephritis is not due to swelling of the Bowman space. It is not an inflammatory process nor is it caused by an accumulation of antiglomerular basement membrane antibodies. REF: p. 758, Table 30-6

A 25-year-old male is diagnosed with a hormone-secreting tumor of the adrenal cortex. Which finding would the nurse expect to see in the lab results? a. Decreased blood volume b. Decreased blood K+ levels c. Increased urine Na+ levels d. Increased white blood cells

ANS: B Aldosterone is secreted from the adrenal cortex. It promotes renal sodium and water reabsorption and excretion of potassium, leading to decreased potassium levels. Blood volume actually increases with aldosterone secretion. Aldosterone promotes sodium reabsorption, leading to normal or decreased Na+ levels, and is not associated with white blood cells. REF: p. 117

A 15-year-old male who is allergic to peanuts eats a peanut butter cup. He then goes into anaphylactic shock. Which assessment findings will the nurse assess for? a. Bradycardia, decreased arterial pressure, and oliguria b. Bronchoconstriction, hives or edema, and hypotension c. Hypertension, anxiety, and tachycardia d. Fever, hypotension, and erythematous rash

ANS: B Anaphylactic shock is characterized by bronchoconstriction, hives, and hypotension; it does not involve oliguria, bradycardia, or hypotension. Septic shock, not anaphylactic shock, is manifested by fever and rash. REF: pp. 643-644

A 51-year-old male presents with recurrent chest pain on exertion and is diagnosed with angina pectoris. The pain occurs when: a. cardiac output has fallen below normal levels. b. the myocardial oxygen supply has fallen below demand. c. myocardial stretch has exceeded the upper limits. d. the vagus nerve is stimulated.

ANS: B Angina is chest pain caused by myocardial ischemia, which develops if the flow or oxygen content of coronary blood is insufficient to meet the metabolic demands of myocardial cells. A decrease in cardiac output would lead to general systemic symptoms, not just chest pain, which is due to a decrease in myocardial oxygenation. Myocardial stretch does not affect angina symptoms. When the vagus nerve is stimulated, a change in rate occurs; it does not precipitate chest pain. REF: p. 614

ADH release from the posterior pituitary is stimulated by which process? a. Low blood pressure sensed by baroreceptors in the kidneys. b. High plasma osmolarity sensed by osmoreceptors in the hypothalamus. c. Low osmolality sensed by osmoreceptors in the kidneys. d. High concentration of potassium sensed by chemoreceptors in the carotid body.

ANS: B As plasma osmolality increases, osmoreceptors are stimulated, the rate of ADH secretion increases, more water is reabsorbed from the kidney, and the plasma is diluted back to its set point osmolality. ADH release is stimulated by high serum osmolality, not lowered blood pressure, low osmolality, or high concentrations of potassium. REF: p. 448

What is the most common type of renal stone composed of? testbanks_and_xanax a. Struvite b. Cystine c. Calcium d. Uric acid

ANS: C The most common stone types are calcium oxalate or phosphate (70-80%). Struvite (magnesium-ammonium-phosphate) occurs 15% of the time. Cystine stoes are rare (less than 1%). Uric acid stones occur 7% of the time. REF: p. 748

Which of the following findings in the patient with Raynaud disease would indicate a need for further teaching? a. The patient eats bananas twice a day. b. The patient smokes cigarettes. c. The patient wears mittens outside. d. The patient takes calcium channel blockers.

ANS: B Cigarette smoking should be stopped to eliminate the vasoconstricting effects of nicotine. The bananas do not cause problems in this patient. The patient should wear mittens outside. Calcium channel blockers are an acceptable treatment for Raynaud disease. REF: p. 607

An 11-year-old is newly diagnosed with type 1 DM. Which classic symptoms should the nurse assess the patient for? a. Recurrent infections, visual changes, fatigue, and paresthesia b. Polydipsia, polyuria, polyphagia, and weight loss c. Vomiting, abdominal pain, sweet, fruity breath, dehydration, and Kussmaul breathing d. Weakness, vomiting, hypotension, and mental confusion

ANS: B Classic symptoms of type 1 DM include polydipsia, polyuria, polyphagia, and weight loss. Recurrent infections and visual changes are complications of diabetes. Vomiting, abdominal pain, and sweet breath are signs of diabetic ketoacidosis. Weakness, hypotension, and mental confusion are signs of hypoglycemia. REF: p. 473

Depolarization of a cardiac muscle cell occurs as the result of: a. a decrease in the permeability of the cell membrane to ions. b. the rapid movement of ions across the cell membrane. c. a blockade by calcium ions. d. stimuli instigated during the refractory period.

ANS: B Depolarization is caused by the movement of electrically charged solutes (ions) across cardiac cell membranes. Permeability must be increased for movement to occur. A blockage of calcium ions would decrease depolarization. No cardiac action potential can be initiated in the refractory period. REF: p. 576

A patient is admitted to the intensive care unit with a closed head injury sustained in a motorcycle accident. The injury has caused severe damage to the posterior pituitary. Which of the following complications should the nurse anticipate? a. Dilutional hyponatremia b. Dehydration from polyuria c. Cardiac arrest from hyperkalemia d. Metabolic acidosis

ANS: B Diabetes insipidus is a well-recognized complication of closed head injury and is manifested by polyuria leading to dehydration. The patient will experience hypernatremia, not hyponatremia. Electrolytes other than sodium are typically not affected with diabetes insipidus. Acidosis is not associated with diabetes insipidus. REF: p. 462

A 25-year-old female presents to her primary care provider reporting vaginal discharge of a white, viscous, and foul-smelling substance. She reports that she has been taking antibiotics for the past 6 months. Which finding will the nurse most likely see on the microorganism report? a. Clostridium difficile overgrowth b. Decreased Lactobacillus c. Streptococcus overgrowth d. Decreased Candida albicans

ANS: B Diminished colonization with Lactobacillus that occurs as a result of prolonged antibiotic treatment increases the risk for vaginal infections, such as vaginosis. Clostridium difficile and Candida albicans occur in the colon, not the vagina. Streptococcus overgrowth will occur in the mouth. REF: p. 136

A 10-year-old develops pneumonia. Physical exam reveals subcostal and intercostal retractions. The child reports that breathing is difficult with feelings that, "I cannot get enough air." What term should the nurse use to document this condition? a. Cyanosis b. Dyspnea c. Hyperpnea d. Orthopnea

ANS: B Dyspnea is defined as "a subjective experience of breathing discomfort that consists of qualitatively distinct sensations that vary in intensity." Cyanosis is a bluish discoloration to the skin. Hyperpnea is an increased ventilatory rate and orthopnea is dyspnea that occurs when an individual lies flat. REF: p. 687

A 10-year-old male is diagnosed with a parasite. Which lab result should the nurse check for a response to the parasite? a. Monocytes b. Eosinophils c. Neutrophils d. Macrophages

ANS: B Eosinophils serve as the body's primary defense against parasites. Monocytes and neutrophils are phagocytic. Macrophages are not active against parasites; they act as long-term defense against infections. REF: p. 146

While planning care for a patient from general anesthesia, which principle should the nurse remember? A side effect of some general anesthetic agents is _____ diabetes insipidus. a. neurogenic b. nephrogenic c. psychogenic d. allogenic

ANS: B General anesthetics can lead to nephrogenic diabetes insipidus (DI). General anesthetics are not associated with any of the other forms of DI. testbanks_and_xanax REF: p. 462

What are the thickened areas of the sarcolemma of myocardial cells that enable electrical impulses to travel in a continuous cell-to-cell fashion called? a. Myosins b. Intercalated disks c. Troponin Ts d. I bands

ANS: B Intercalated disks allow electrical impulses to be transmitted rapidly from cardiac fiber to cardiac fiber because the network of fibers is connected at these disks. Myosins do not play a role in conduction. Troponin Ts are involved in contraction but are not part of the sarcolemma. I bands are a part of the filaments bud, which do not conduct impulses in a cell-to-cell fashion. REF: p. 578

A 30-year-old male was involved in a motor vehicle accident. The glass from the shattered window cut his face and neck. The scar, however, was raised and extended beyond the original boundaries of the wound. This pattern of scarring is caused by impaired: a. nutritional status. b. collagen synthesis. c. epithelialization. d. contraction.

ANS: B Impaired production of collagen can cause surface overhealing, leading to a keloid or a hypertrophic scar. Nutritional deficiencies would lead to healing problems, but not extended scarring. Necrosis or steroid use leads to impaired epithelialization. Impaired contraction would lead to drawing of tissues, not raised. REF: p. 153

Which of the following conditions would cause the nurse to monitor for hyperkalemia? a. Excess aldosterone b. Acute acidosis c. Insulin usage d. Metabolic alkalosis

ANS: B In acidosis, ECF hydrogen ions shift into the cells in exchange for ICF potassium and sodium; hyperkalemia and acidosis therefore often occur together. Acidosis does not cause excess aldosterone. Insulin would help treat hyperkalemia, not cause it. Alkalosis does not lead to hyperkalemia. REF: p. 124

A 42-year-old is diagnosed with constrictive pericarditis. The nurse assesses the blood pressure for decreased cardiac output because of: a. pericardial effusions. b. fibrosis and calcification of the pericardial layers. c. cardiomyopathy. d. hemorrhage in the pericardial cavity.

ANS: B In constrictive pericarditis, fibrous scarring compresses the heart and eventually reduces cardiac output. Pericardial effusion is manifested by chest pain. Cardiomyopathy is a general term for pathophysiological changes in the heart. Hemorrhage in the pericardial cavity will lead to tamponade. REF: pp. 623-624

Which assessment findings will the nurse observe in a patient diagnosed with severe pulmonary edema? a. Thick mucous secretions b. Pink, frothy sputum c. Hypocapnia d. Wheezing

ANS: B In severe edema, pink frothy sputum is expectorated, not thick mucous secretions. Neither hypocapnia nor wheezing is an expected assessment finding associated with pulmonary edema. REF: p. 695

A 49-year-old male presents reporting chest pain. EKG reveals ST elevation. He is diagnosed with myocardial ischemia. Which of the following interventions would be most beneficial? a. Administer a diuretic to decrease volume. b. Apply oxygen to increase myocardial oxygen supply. c. Encourage exercise to increase heart rate. d. Give an antibiotic to decrease infection.

ANS: B Increasing the myocardial oxygen supply is indicated to treat ischemia. A decrease in fluid volume is not appropriate. Heart rate should be decreased to decrease cardiac workload. Antibiotics are not the most beneficial; oxygen is. REF: pp. 615-616

A public health nurse is teaching the community about health promotion. Which information should the nurse include for innate immunity? Innate immunity is gained: a. following an illness. b. at birth. c. via injection of specific antibodies. d. in adulthood.

ANS: B Innate immunity is present at birth. It is not dependent on illness or injection. REF: p. 134

What type of breathing will the nurse observe while assessing a patient experiencing both metabolic acidosis and Kussmaul respirations? a. Audible wheezing or stridor b. Increased rate, large tidal volumes, and no expiratory pause c. Rapid respirations with periods of apnea d. Very slow inhalations and rapid expirations

ANS: B Kussmaul respirations are characterized by a slightly increased ventilatory rate, very large tidal volumes, and no expiratory pause. Audible wheezing is usually associated with conditions such as asthma, and stridor indicates a narrowed airway. Cheyne-Stokes respirations are characterized by alternating periods of deep and shallow breathing, with periods of apnea lasting from 15 to 60 seconds. Kussmaul respirations do not have slow inhalations; bronchiolar disorders have these characteristics. REF: p. 688

Which of the following patients should the nurse assess for decreased oncotic pressure in the capillaries? A patient with: a. a high-protein diet. b. liver failure. c. low blood pressure. d. low blood glucose.

ANS: B Liver failure leads to lost or diminished plasma albumin production, and this contributes to decreased plasma oncotic pressure. A high-protein diet would provide albumin for the maintenance of oncotic pressure. Low blood pressure would lead to decreased hydrostatic pressure. Decreased glucose does not affect oncotic pressure. REF: p. 116

A patient was admitted to the intensive care unit with a diagnosis of acute myocardial infarction (MI) and is being treated for shock. The primary cause of shock is most likely: a. rapid heart rate. b. decreased cardiac contractility. c. increased capillary permeability. d. decreased afterload due to vasodilation.

ANS: B MI leads to decreased cardiac contractility due to a damaged myocardium and would lead to shock. A rapid heart rate would not cause shock. Increased capillary permeability would not lead to shock. Decreased afterload will not lead to shock associated with MI. REF: p. 637

The post-surgical patient is experiencing delayed wound healing. The dietitian believes the delay is related to nutritional intake. A deficiency in which of the following substances could directly affect healing? a. Vitamin D b. Ascorbic acid c. Melanin d. Cholesterol

ANS: B Most of the factors that interfere with the production of collagen in healing tissues are nutritional. Scurvy, for example, is caused by lack of ascorbic acid—one of the cofactors required for collagen formation by fibroblasts. The results of scurvy are poorly formed connective tissue and greatly impaired healing. Healing is not associated with vitamin D, melanin, or cholesterol. REF: p. 153

Neurogenic shock can be caused by any factor that inhibits the: a. parasympathetic nervous system. b. sympathetic nervous system. c. somatic nervous system. d. thalamus.

ANS: B Neurogenic shock is caused by any factor that inhibits the sympathetic nervous system or overstimulates the parasympathetic nervous system. Neurogenic shock is not caused by inhibition of the somatic system or of the thalamus. REF: pp. 642-643

The predominant phagocyte of early inflammation is the: a. eosinophil. b. neutrophil. c. lymphocyte. d. macrophage

ANS: B Neutrophils are the predominant phagocytes in the early inflammatory site, arriving within 6-12 hours after the initial injury. Eosinophils help limit and control inflammation, but they are not the prominent phagocyte. Lymphocytes are part of the innate immune response. Macrophages kill microorganisms. REF: p. 146

A 20-year-old male shoots his hand with a nail gun while replacing roofing shingles. Which of the following cell types would be the first to aid in killing bacteria to prevent infection in his hand? a. Eosinophils b. Neutrophils c. Leukotrienes d. Monocytes

ANS: B Neutrophils are the predominant phagocytes in the early inflammatory site, arriving within 6-12 hours after the initial injury. Eosinophils help limit and control inflammation. Leukotrienes are activators of the inflammatory response. Monocytes enter much later and replace leukocytes. REF: p. 146

A nurse takes an adult patient's blood pressure and determines it to be normal. What reading did the nurse obtain? a. Systolic pressure between 140 and 150 mm Hg b. Systolic pressure less than 120 mm Hg and diastolic pressure less than 80 mm Hg c. Systolic pressure less than 100 mm Hg regardless of diastolic pressure d. Systolic pressure greater than 140 mm Hg and a diastolic pressure of 100 mm Hg

ANS: B Normal blood pressure has a systolic pressure less than 120 mm Hg and diastolic pressure less than 80 mm Hg. A systolic pressure of 140 mm Hg or more would indicate stage I hypertension. A systolic pressure of less than 100 mm Hg would indicate low blood pressure. A diastolic pressure greater than 90 mm Hg would indicate hypertension. REF: p. 600, Table 24-1

A 20-year-old presents reporting difficulty breathing when lying down. What term should the nurse use to document this condition? a. Dyspnea b. Orthopnea c. Apnea d. Tachypnea

ANS: B Orthopnea is dyspnea that occurs when an individual lies flat. Dyspnea is shortness of breath that occurs with activity. Apnea is cessation of breathing, and tachypnea is rapid breathing. REF: p. 687

A 60-year-old undergoes surgery for a bone fracture. Which nursing measure would be most effective for preventing pulmonary embolism (PE) in this patient? a. Ensure that patient uses supplemental oxygen. b. Prevent deep vein thrombosis formation. c. Check hematocrit and hemoglobin levels frequently during the postoperative period. d. Promote aggressive fluid intake.

ANS: B PE most commonly results from embolization of a clot from deep venous thrombosis (DVT) involving the lower leg; thus, preventing these will help prevent pulmonary emboli. Oxygen will neither prevent emboli nor check hemoglobin and hematocrit. Fluid intake will help, but it is not as important as preventing DVT. REF: p. 706

A 73-year-old has increased pulmonary pressure resulting in right heart failure. A potential cause for the right heart to fail is: a. hypertension. b. left heart failure. c. acute pneumonia. d. pericarditis.

ANS: B Right-sided failure often follows left-sided failure when pulmonary congestion forces backward flow of blood into the left ventricle. It is not due to hypertension, pneumonia, or pericarditis. REF: p. 636

Superior vena cava syndrome (SVCS), causing venous distention in the upper extremities, is a result of progressive superior vena cava: a. inflammation. b. occlusion. c. distention. d. sclerosis.

ANS: B SVCS is a progressive occlusion of the SVC that leads to venous distention in the upper extremities and head. This distention is not a result of progressive inflammation, distention, or sclerosis. REF: p. 599

A 52-year-old male presents with pooling of blood in the veins of the lower extremities and edema. The diagnosis is chronic venous insufficiency, and an expected assessment finding of this disorder is: a. deep vein thrombus formation. b. skin hyperpigmentation. c. gangrene. d. edema above the knee.

ANS: B Symptoms include edema of the lower extremities and hyperpigmentation of the skin of the feet and ankles but deep vein thrombi do not form. Edema in these areas may extend to the knees but not above. Gangrene does not occur in veins but in arteries. REF: p. 598

What structure conducts action potentials down the atrioventricular septum? a. Bachmann bundle b. Bundle of His c. Sinoatrial node d. Atrioventricular node

ANS: B The bundle of His conducts action potentials down the atrioventricular septum, not the Bachmann bundle. The sinoatrial node conducts the potential along the atria while the atrioventricular node conducts impulses to the ventricles. REF: p. 576

Which of the following buffer pairs is considered the major plasma buffering system? a. Protein/fat b. Carbonic acid/bicarbonate c. Sodium/potassium d. Amylase/albumin

ANS: B The carbonic acid/bicarbonate buffer pair operates in both the lung and the kidney and is a major extracellular buffer. Protein and fat are nutrients not related to the buffering system. Sodium and potassium are electrolytes for fluid and electrolyte balance, not the major plasma buffering system for acid-base balance. Amylase is a carbohydrate enzyme, and albumin is a protein; neither is a buffering system. REF: p. 125

Which statement indicates teaching was successful regarding the classical pathway of the complement system? The classical pathway of the complement system is activated by: a. histamine. b. antigen-antibody complexes. c. leukotrienes. d. prostaglandins.

ANS: B The classical pathway of the complement system is activated by antibodies of the immune system, not by histamine, leukotrienes, or prostaglandins. REF: p. 139

What common neurologic disturbances should the nurse assess for in a patient with a pituitary adenoma? a. Coma b. Visual disturbances c. Confused states d. Breathing abnormalities

ANS: B The clinical manifestations of pituitary adenomas are visual changes including visual field impairments (often beginning in one eye and progressing to the other) and temporary blindness. Coma, confusion, and breathing abnormalities are not associated with pituitary adenomas. REF: p. 464

A 75-year-old male has severe chest pain and dials 911. Based upon the lab findings indicating a patient has elevated levels of cardiac troponins I and T, the nurse suspects which of the following has occurred? a. Raynaud disease b. Myocardial infarction (MI) c. Orthostatic hypotension d. Angina

ANS: B The diagnosis of acute MI is made on the basis of serial cardiac biomarker alterations. The cardiac troponins (troponins I and T) are the most specific indicators of MI. Elevated troponins I and T are indicative of MI, not Raynaud disease, orthostatic hypotension, or angina REF: p. 620

While planning care for elderly individuals, the nurse remembers the elderly are at a higher risk for developing dehydration because they have: a. a higher total body water volume. b. decreased muscle mass. c. increased thirst. d. an increased tendency toward developing edema.

ANS: B The elderly are at higher risk for dehydration due to a decrease in muscle mass. The elderly have a decrease in total body water and thirst. The increased tendency to develop edema is not related to dehydration. REF: p. 131, Geriatric Considerations

After teaching the staff about the clotting system, which statement indicates the teaching was successful? The end product of the clotting system is: a. plasmin. b. fibrin. c. collagen. d. factor X.

ANS: B The end product of the clotting system is fibrin. Plasmin activates the complement cascade. Collagen plays a factor in wound healing. Factor X is a first step in the clotting system. REF: p. 139

A 20-year history of smoking causes airways to be obstructed as a result of: a. excessive mucus production. b. loss of elastic recoil. c. infection and inflammation. d. airway edema.

ANS: B The major mechanism of airflow limitation is a loss of elastic recoil, not excessive mucus as with bronchitis. The major mechanism of airflow limitation in this situation is not associated with infection or airway edema. REF: p. 702

If a patient has a problem with the adrenal medulla, which of the following hormones should the nurse monitor? a. Cortisol b. Epinephrine c. Androgens d. Estrogens

ANS: B The major products stored and secreted by the adrenal medulla are the catecholamines epinephrine (adrenaline) and norepinephrine. The adrenal cortex secretes cortisol and androgens. The pituitary secretes estrogens. REF: p. 456, Figure 18-19

When a staff member asks the nurse which gland secretes ADH and oxytocin, how should the nurse respond? a. Anterior pituitary b. Posterior pituitary c. Hypothalamus d. Pineal

ANS: B The posterior pituitary secretes ADH, which is also called vasopressin, and oxytocin. The anterior pituitary secretes ACTH, melanocyte-stimulating hormone (MSH), somatotropic hormones (GH, prolactin), and glycoprotein hormones—follicle-stimulating hormone (FSH), LH, and TSH. The hypothalamus secretes PRF, which stimulates secretion of prolactin; PIF (dopamine), which inhibits prolactin secretion; TRH, which affects release of thyroid hormones; GH-releasing hormone (GHRH), which stimulates the release of GH; somatostatin, which inhibits the release of GH; gonadotropin-releasing hormone (GnRH), which facilitates the release of FSH and LH; corticotropin-releasing hormone (CRH), which facilitates the release of ACTH and endorphins; and substance P, which inhibits ACTH release and stimulates release of a variety of other hormones. The pineal gland secretes melatonin. REF: p. 448

A 75-year-old obese female presents to her primary care provider reporting edema in the lower extremities. Physical exam reveals that she has varicose veins. Upon performing the history, which of the following is a possible cause for the varicose veins? a. Extreme exercise b. Long periods of standing c. Trauma to the deep veins d. Ischemia

ANS: B The probable cause of the patient's varicose veins is gradual venous distention caused by the action of gravity on blood in the legs due to long periods of standing. Varicose veins are most likely due to long periods of standing leading to the action of gravity promoting venous distention. Exercise would help prevent this. Trauma can occur, but usually this affects the more superficial veins. Ischemia affects arteries not veins. REF: p. 598

During inflammation, the liver is stimulated to release plasma proteins, collectively known as: a. opsonins. b. acute phase reactants. c. antibodies. d. phagolysosomes.

ANS: B The synthesis of many plasma proteins by the liver is increased during inflammation. These proteins, which can be either proinflammatory or anti-inflammatory in nature, are referred to as acute phase reactants. Opsonins coat the surface of bacteria and increase their susceptibility to being phagocytized. Antibodies are proteins of the immune system. Phagolysosomes destroy bacteria. REF: p. 149

How are ions such as sodium and potassium transported in and out of the cell across the plasma membrane? A) Via the process of endocytosis and exocytosis B) Through small pores in the lipid bilayer C) By glycolipid carrier molecules D) Through leak and gated protein channels

D) Through leak and gated protein channels

A patient with one kidney underwent surgery for an adrenal tumor that removed a large portion of the zona glomerulosa. The nurse would expect a postsurgical decrease in the patient's: a. sodium. b. aldosterone. c. potassium. d. acid.

ANS: B The zona glomerulosa, the outer layer, constitutes about 15% of the cortex and primarily produces the mineralocorticoid aldosterone. The zona glomerulosa is not associated with sodium, potassium, or acid production. testbanks_and_xanax REF: p. 453

A 68-year-old male presents to the ER reporting chest pain. He has a history of stable angina that now appears to be unstable. He most likely has: a. mild to moderate atherosclerosis. b. impending myocardial infarction (MI). c. electrical conduction problems in the heart. d. decreased myocardial oxygen demand.

ANS: B Unstable angina is an indication of impending MI. Unstable angina could be caused by moderate atherosclerosis, altered electrical conduction, or a decrease in myocardial oxygen, but it is an indication of impending MI. REF: p. 616

Which physical feature supports the diagnosis of Cushing syndrome? a. Weight loss and muscle wasting b. Truncal obesity and moon face c. Pallor and swollen tongue d. Depigmented skin and eyelid lag

ANS: B Weight gain is the most common feature and results from the accumulation of adipose tissue in the trunk, facial, and cervical areas. These characteristic patterns of fat deposition have been described as "truncal obesity," "moon face," and "buffalo hump." Weight gain, not loss, is the most common feature of Cushing syndrome. Pallor is not associated with Cushing syndrome. The skin of the patient with Cushing syndrome is bronze in color. testbanks_and_xanax REF: p. 482

What phrase describes the condition in which a series of alveoli in the left lower lobe receive adequate ventilation but lack adequate perfusion? a. A right-to-left shunt b. Alveolar dead space c. A low ventilation-perfusion ratio d. Pulmonary hypotension

ANS: B When certain areas of the alveoli experience inadequate perfusion, it is referred to as dead space. The situation is not referred to as either a right-to-left shunt or as pulmonary hypotension. Shunting is due to a low ventilation-perfusion ratio. REF: p. 690

A 65-year-old male with a history of untreated hypertension is now experiencing left heart failure. A nurse recalls his untreated hypertension led to: a. ventricular dilation and wall thinning. b. myocardial hypertrophy and ventricular remodeling. c. inhibition of renin and aldosterone. d. alterations in alpha and beta receptor function.

ANS: B With hypertension comes increased afterload and resistance to ventricular emptying and more workload for the ventricle, which responds with hypertrophy of the myocardium and ventricular remodeling. Ventricular dilation can occur, but the wall will thicken, not thin. Renin and aldosterone release are increased, not inhibited. Alterations in alpha and beta functioning may occur, but the response to hypertension is myocardial hypertrophy and ventricular remodeling. REF: p. 632

While planning care for an elderly patient, the nurse remembers that increased age is associated with: (select all that apply) a. increased T-cell function. b. decreased immune function. c. increased production of antibodies. d. decreased numbers of circulating immune complexes. e. decreased ability to fight infection.

ANS: B, D, E Increased age is associated with diminished T-cell function, decreased immune function, diminished production of antibody responses, decreased circulating immune complexes, and decreased ability to fight infection. REF: p. 173, Geriatric Considerations

When a patient's CT scan reveals a lesion above the pontine micturition center, which condition would the nurse expect? a. Dyssynergia b. Detrusor hyperreflexia c. Detrusor areflexia d. Detrusor sphincter dyssynergia

ANS: B testbanks_and_xanax Neurologic disorders that develop above the pontine micturition center result in detrusor hyperreflexia, also known as an uninhibited or reflex bladder. Lesions that develop in upper motor neurons of the brain and spinal cord result in dyssynergia. Lesions that involve the sacral micturition center (below S1; may also be termed cauda equina syndrome) or peripheral nerve lesions result in detrusor areflexia (acontractile detrusor), a lower motor neuron disorder. Neurologic lesions that occur below the pontine micturition center but above the sacral micturition center (between C2 and S1) are also upper motor neuron lesions and result in detrusor hyperreflexia with vescico-sphincter dyssynergia. REF: p. 750

While planning care for a heart patient, which principle should the nurse recall? Right ventricular afterload is affected by: A. Vascular resistance in the systemic vessels B. Right end-diastolic pressure C. Pressures in the vena cava D. Pulmonary vascular resistance

D. Pulmonary vascular resistance

The nurse identified each of the following clinical manifestations of inflammation. Which would the nurse classify as a local response? (select all that apply) a. Fever b. Redness c. Swelling d. Heat e. Pain

ANS: B, C, D, E Local manifestations of inflammation are the result of the vascular changes associated with the inflammatory process, including vasodilation and increased capillary permeability. The symptoms include redness, heat, swelling, and pain. Fever is a symptom of infection. REF: p. 149

Biochemical secretions that trap and kill microorganisms include: a. hormones. b. neurotransmitters. c. earwax. d. gastric acid.

ANS: C Epithelial cells secrete several substances that protect against infection, including earwax. Hormones do not contain biochemical secretions that trap and kill microorganisms. Neurotransmitters carry important messages, and gastric acid helps break down food into its component parts, but neither contains biochemical secretions. REF: p. 135

How are ions such as sodium and potassium transported in and out of the cell across the plasma membrane? A) Via the process of endocytosis and exocytosis B) Through small pores in the lipid bilayer C) By glycolipid carrier molecules D) Through leak and gated protein channels

D) Through leak and gated protein channels

A 54-year-old male with a long history of smoking complains of excessive tiredness, shortness of breath, and overall ill feelings. Lab results reveal decreased pH, increased CO2, and normal bicarbonate ion. These findings help to confirm the diagnosis of: a. respiratory alkalosis. b. metabolic acidosis. c. respiratory acidosis. d. metabolic alkalosis.

ANS: C A decreased pH indicates acidosis. With increased CO2, it is respiratory acidosis. The bicarbonate is normal, so it cannot be metabolic acidosis. REF: pp. 128-129

What term should the nurse use to document a detached blood clot? a. Thrombus b. Embolus c. Thromboembolus d. Infarction

ANS: C A thrombus is a clot that remains attached to a vessel wall; a detached thrombus is a thromboembolus. An embolus is a bolus of material floating in the bloodstream. An infarction is death of tissue. REF: p. 599

A 26-year-old recently underwent surgery and is now experiencing dyspnea, cough, fever, and leukocytosis. Tests reveal a collapsed lung caused by the removal of air from obstructed alveoli. Which term is used to document this condition? a. Compression atelectasis b. Bronchiectasis c. Absorption atelectasis d. Hypoventilation

ANS: C Absorption atelectasis results from removal of air from obstructed or hypoventilated alveoli or from inhalation of concentrated oxygen or anesthetic agents. Compression atelectasis is caused by external pressure exerted by tumor, fluid, or air in the pleural space or by abdominal distention pressing on a portion of lung. Bronchiectasis is a dilation of the bronchi, not atelectasis. Hypoventilation is inadequate alveolar ventilation of the lungs; it is not due to removal of air. REF: p. 693

Warmth and redness of the skin are indicators of inflammation. Which of the following processes is responsible for this clinical manifestation? A) Increased vascular permeability B) Phagocytosis C) Vasoconstriction D) Vasodilation

D) Vasodilation

Which statement by the staff indicates teaching was successful concerning aldosterone? Secretion of aldosterone results in: a. decreased plasma osmolality. b. increased serum potassium levels. c. increased blood volume. d. localized edema.

ANS: C Aldosterone promotes renal sodium and water reabsorption and excretion of potassium, thus increasing blood volume. Aldosterone secretion would cause increased plasma osmolality but it does not promote the development of localized edema; it affects blood volume. REF: p. 117

What regulates aldosterone secretion? a. The sympathetic nervous system b. ACTH feedback c. The renin-angiotensin system d. Positive feedback

ANS: C Aldosterone synthesis and secretion are regulated primarily by the renin-angiotensin system. Aldosterone synthesis and secretion are not regulated by the sympathetic nervous system, adrenocorticotropic hormone feedback, or positive feedback. REF: p. 455

A 50-year-old male is diagnosed with pulmonary embolism (PE). Which of the following symptoms most likely occurred before treatment is initiated? a. Dry cough and inspiratory crackles b. Shallow respirations and wheezing c. Chest pain and shortness of breath d. Kussmaul respirations and back pain

ANS: C An individual with PE usually presents with the sudden onset of pleuritic chest pain, dyspnea, tachypnea, tachycardia, and unexplained anxiety. Individuals with PE do not experience a cough or wheezing. Kussmaul respirations are associated with acidosis. REF: p. 707

Warmth and redness of the skin are indicators of inflammation. Which of the following processes isresponsible for this clinical manifestation? A) Increased vascular permeability B) Phagocytosis C) Vasoconstriction D) Vasodilation

D) Vasodilation

Upon assessment of the patient, the nurse finds a widened pulse pressure and throbbing peripheral pulses. Which valve disorder does the nurse suspect? a. Mitral regurgitation b. Mitral stenosis c. Aortic regurgitation d. Aortic stenosis

ANS: C Aortic regurgitation is manifested by widened pulse pressure resulting from increased stroke volume and diastolic backflow. Mitral regurgitation is manifested by heart failure. Mitral stenosis is manifested by pulmonary edema and heart failure. Aortic stenosis is manifested by narrowed pulse pressure. REF: p. 627

A 72-year-old has a history of hypertension and atherosclerosis. An echocardiogram reveals backflow of blood into the left ventricle. Which of the following is the most likely diagnosis documented on the chart? a. Mitral regurgitation b. Mitral stenosis c. Aortic regurgitation d. Aortic stenosis

ANS: C Aortic regurgitation would allow backward flow of blood into the left ventricle. Mitral regurgitation would allow backward flow of blood into the left atrium. Mitral stenosis would impede blood flow from the right atrium into the right ventricle. Aortic stenosis would impede blood flow into the aorta. REF: p. 627

A patient presents to a primary care provider reporting chest pain and is diagnosed with atherosclerosis. This disease is caused by: a. arterial wall thinning and weakening. b. abnormally dilated arteries and veins. c. abnormal thickening and hardening of vessel walls. d. autonomic nervous system imbalances.

ANS: C Atherosclerosis is a form of arteriosclerosis characterized by thickening and hardening of the vessel wall. Affected arteries are narrowed, not dilated. Atherosclerosis is not related to autonomic nervous system imbalances, which would lead to changes in rate or rhythm. REF: p. 607

In autocrine signaling: A) a cell releases a hormone. B) a cell signals a neighboring cell. C) a neuron signals another neuron. D) a cell signals itself.

D) a cell signals itself.

What is the cause of the hyperpigmentation seen in people with Cushing syndrome? a. Abnormal levels of cortisol b. Permissive effects of aldosterone when cortisol levels are altered c. Elevated levels of ACTH d. Hypersensitivity of melanocytes with sun exposure

ANS: C Bronze or brownish hyperpigmentation of the skin, mucous membranes, and hair occurs when there are very high levels of ACTH. The pigmentation changes associated with Cushing syndrome and Addison disease are not due to increased levels of cortisol or hypersensitivity of melanocytes. Aldosterone affects fluid balance. REF: p. 482

A 70-year-old female has brittle bones secondary to osteoporosis. Her primary care provider prescribes calcitonin to: a. activate vitamin D. b. stimulate osteoclastic activity. c. inhibit calcium resorption from bones. d. promote thyroid hormone release.

ANS: C Calcitonin lowers serum calcium levels by inhibition of bone-resorbing osteoclasts. Calcitonin does not activate vitamin D, stimulate bone-resorbing osteoclasts, or promote thyroid hormone release. REF: p. 449

A nurse observes a cardiologist multiplying the heart rate by stroke volume. What is the cardiologist measuring? a. Vascular resistance b. Preload c. Cardiac output d. Ejection fraction

ANS: C Cardiac output is found by the heart rate times stroke volume. Vascular resistance is not a factor in the calculation of cardiac output; it does play a role in blood pressure. Preload affects cardiac output but is not the result of heart rate times stroke volume. Ejection fraction is the amount ejected per beat. REF: p. 581

Which condition is considered the ultimate cause of death in the patient with diabetes? a. Renal disease b. Stroke c. Cardiovascular disease d. Cancer

ANS: C Cardiovascular disease, not renal disease, stroke, or cancer, is the ultimate cause of death in up to 68% of people with diabetes. REF: p. 481

A 62-year-old male presents to his primary care provider reporting chest pain at rest and with exertion. He does not have a history of coronary artery disease and reports that the pain often occurs at night. He is most likely experiencing which type of angina? a. Unstable b. Stable c. Prinzmetal d. Silent

ANS: C Chest pain that occurs at rest and at night is descriptive of Prinzmetal angina. Unstable angina is a form of acute coronary syndrome that results from reversible myocardial ischemia. Stable angina is predictable and occurs with activity. Silent angina has few, if any, symptoms. REF: p. 614

A nurse recalls the mast cell, a major activator of inflammation, initiates the inflammatory response through the process of: a. chemotaxis. b. endocytosis. c. degranulation. d. opsonization.

ANS: C Degranulation of mast cells is a major cellular component of inflammation. Chemotaxis is the process of white cell migration. Endocytosis and opsonization are parts of phagocytosis but are not factors in mast cell response. REF: pp. 144-145

Which of the following patients is the most at risk for developing hypernatremia? A patient with: a. vomiting. b. diuretic use. c. dehydration. d. hypoaldosteronism.

ANS: C Dehydration leads to hypernatremia because an increase in sodium occurs with a net loss in water. Vomiting and diuretic use leads to hyponatremia. Hypoaldosteronism leads to hyponatremia. REF: p. 119

Active mediated transport is used to transport molecules: A) between organelles in the cytoplasm. B) in the blood stream. C) in the interstitial fluid. D) across a membrane.

D) across a membrane.

A 20-year-old female is being admitted to the hospital with fever and septic shock. Which set of assessment findings would the nurse expect the patient to exhibit? a. Bradycardia, palpitations, confusion, truncal rash b. Severe respiratory distress, jugular venous distention, chest pain c. Low blood pressure and tachycardia d. Reduced cardiac output, increased systemic vascular resistance, moist cough

ANS: C Clinical manifestations of shock will include a low blood pressure and tachycardia. Tachycardia, not bradycardia, will occur. Severe respiratory distress, jugular vein distention, and chest pain are symptoms of heart failure, particularly pulmonary edema. Cardiac output is reduced, but there is a decrease in systemic vascular resistance. REF: p. 637

Which statement indicates teaching was successful regarding collectins? Collectins are produced by the: a. kidneys. b. bowel. c. lungs. d. integument.

ANS: C Collectins are produced by the lungs. Collectins are not associated with the kidneys, bowel, or integumentary system. REF: p. 135

The nurse is reviewing the lab data of a newly admitted patient. The nurse notes the patient had an erythrocyte sedimentation done, and the results are quite elevated. The nurse would focus the care plan on which of the following conditions? a. Anemia b. Infection c. Inflammation d. Electrolyte imbalance

ANS: C Common laboratory tests for inflammation measure levels of acute phase reactants. An increase in fibrinogen is associated with an increased erythrocyte sedimentation rate, which is considered a good indicator of an acute inflammatory response. An elevated sedimentation rate is not associated with anemia, infection, or an electrolyte imbalance. REF: p. 149

A nurse checks lab results as both Cushing disease and Addison disease can manifest with elevated levels of: a. ADH. b. estrogen. c. adrenocorticotropic hormone (ACTH). d. aldosterone.

ANS: C Cushing disease and Addison disease are related to elevated levels of ACTH, not ADH, estrogen, or aldosterone. REF: p. 482

A 52-year-old is diagnosed with primary hypertension but has no other health problems. Present treatment would cause the nurse to anticipate administering which drug to the patient? a. A beta-adrenergic agonist b. An alpha-adrenergic agonist c. A diuretic d. A calcium channel agonist

ANS: C Diuretics have been shown to be the safest and most effective medications for lowering blood pressure and preventing the cardiovascular complications of hypertension. A beta-adrenergic, an alpha-adrenergic, or a calcium channel agonist drug would be used for patients with other concurrent health problems. REF: p. 604

Individuals with a recent diagnosis of emphysema should be assessed for which most common presenting factor? a. A productive cough b. Cyanosis c. Dyspnea d. Cor pulmonale

ANS: C Dyspnea, not a productive cough, is the most common presenting factor of emphysema. Neither cyanosis nor cor pulmonale is a common presenting factor of emphysema. REF: p. 701, Table 27-2

The process by which an action potential in the plasma membrane of a myocardial cell triggers the events that directly cause contraction of the myocardial cells is referred to as: a. electrocontraction. b. intercalated communication. c. excitation-contraction coupling. d. myosin communication.

ANS: C Excitation-contraction coupling, not electrocontraction, is the process by which an action potential in the plasma membrane of the muscle fiber triggers the cycle, leading to cross-bridge activity and contraction. Neither intercalated nor myosin communication is associated with leading to cross-bridge activity and contraction. REF: p. 581

Foam cells in a fatty streak are: a. deposited adipose cells. b. injured neutrophils. c. macrophages that engulf low-density lipoprotein (LDL). d. lipid-laden mast cells.

ANS: C Foam cells are lipid-laden macrophages that engulf LDL. They are deposited in vessels, not adipose cells. Foam cells are not injured neutrophils nor are they mast cells. REF: p. 607 | p. 609

A 60-year-old female is diagnosed with hyperkalemia. Which assessment finding should the nurse expect to observe? a. Weak pulse b. Excessive thirst c. Oliguria d. Constipation

ANS: C Hyperkalemia is manifested by oliguria. Hypokalemia is manifested by a weak pulse; it is not caused by hyperkalemia. Hypokalemia is manifested by excessive thirst. Diarrhea, not constipation, is a manifestation of hyperkalemia. REF: p. 124, Table 5-6

Which symptom would the nurse expect in a patient diagnosed with hyperaldosteronism? a. Hypovolemia b. Hypotension c. Hypokalemia d. Hyponatremia

ANS: C Hypertension, hypokalemia, and neuromuscular manifestations are the hallmarks of primary hyperaldosteronism. Neither hypovolemia nor hyponatremia is associated with hyperaldosteronism. REF: p. 483

Bone fractures are a risk factor in chronic renal failure because: a. calcium is lost in the urine. b. osteoblast activity is excessive. c. the kidneys fail to activate vitamin D. d. autoantibodies to calcium molecules develop.

ANS: C Hypocalcemia is accelerated by impaired renal synthesis of 1,25 - dihydroxy-vitamin D3. The combined effect of vitamin D deficiency can result in renal osteodystrophies with increased risk for fractures. Calcium is not lost in the urine. Osteoblast activity is not accelerated. Antibodies do not develop. REF: pp. 765-766

A 45-year-old diagnosed with Graves disease underwent surgical removal of the thyroid gland. During the postoperative period, the patient's serum calcium is low. The most probable reason for her low serum calcium is: a. hyperparathyroidism secondary to Graves disease. b. myxedema secondary to surgery. c. hypoparathyroidism caused by surgical injury to the parathyroid glands. d. hypothyroidism resulting from lack of thyroid replacement.

ANS: C Hypoparathyroidism is most commonly caused by damage to the parathyroid glands during thyroid surgery, not secondary to Graves disease, myxedema, or the lack of thyroid replacement. REF: p. 470

A patient has done research on monoclonal antibodies on the Internet. Which statement indicates a correct understanding? Pure monoclonal antibodies are produced by: a. T lymphocytes. b. bone marrow. c. laboratories. d. fetuses.

ANS: C Monoclonal antibodies are produced in the laboratory from one B cell that has been cloned; thus, the entire antibody is of the same class, specificity, and function. Pure monoclonal antibodies are not produced by T lymphocytes, bone marrow, or fetuses. REF: p. 164, Box 7-1

A patient diagnosed with thyroid carcinoma would be expected to have T3 and T4 levels that are: a. high. b. low. c. normal. d. variable.

ANS: C Most individuals with thyroid carcinoma have normal T3 and T4 levels and are therefore euthyroid. REF: p. 470

Inflammatory cells have difficulty limiting the colonization of microorganisms in infective endocarditis because the: a. microorganisms are resistant. b. valves are avascular. c. microorganisms are sequestered in a fibrin clot. d. colonies overwhelm the phagocytes.

ANS: C In endocarditis, bacterial colonies are inaccessible to host defenses because they are embedded in the protective fibrin clots; it is not because the microorganisms are resistant, that the valves are avascular, or that the colonies overwhelm the phagocytes. REF: p. 630

When a person is in shock, a nurse remembers impairment in cellular metabolism is caused by: a. release of toxic substances. b. free radical formation. c. inadequate tissue perfusion. d. lack of nervous or endocrine stimulation.

ANS: C In shock, impaired cellular metabolism is caused by inadequate tissue perfusion. Impairment of cellular metabolism is the result of inadequate tissue perfusion; toxic substances could develop, but it would be secondary to the inadequate perfusion. Free radicals are not the cause of impaired cellular metabolism. The lack of nervous control would lead to vasodilation, but perfusion still occurs. REF: p. 637

A 70-year-old male with chronic renal failure presents with edema. Which of the following is the most likely cause of this condition? a. Increased capillary oncotic pressure. b. Decreased interstitial oncotic pressure. c. Increased capillary hydrostatic pressure. d. Increased interstitial hydrostatic pressure.

ANS: C Increased capillary hydrostatic pressure would facilitate increased movement from the capillary to the interstitial space, leading to edema. Increased capillary (plasma) oncotic pressure attracts water from the interstitial space back into the capillary. Decreased interstitial oncotic pressure would keep water in the capillary. Increased interstitial hydrostatic pressure would facilitate increased water movement from the interstitial space into the capillary. REF: pp. 115-116

Which nutrient would the nurse encourage the patient to consume for thyroid hormone synthesis? a. Zinc b. Sodium c. Iodine d. Calcium

ANS: C Iodine, not zinc, sodium, or calcium, is necessary for synthesis of thyroid hormone. REF: p. 450

Which organ system should the nurse monitor when the patient has long-term potassium deficits? a. Central nervous system (CNS) b. Lungs c. Kidneys d. Gastrointestinal tract

ANS: C Long-term potassium deficits lasting more than 1 month may damage renal tissue, with interstitial fibrosis and tubular atrophy. Long-term potassium deficits are not associated with damage to the CNS, GI tract, or lungs. REF: pp. 123-124

A macrophage was isolated and analyzed for major histocompatibility complex. Which of the following would be expected? a. MHC I only b. MHC II only c. MHC I and II d. Neither MHC I nor MHC II

ANS: C MHC I and II would be expected. REF: p. 168

The most common cause of primary hypothyroidism in adults is: A) bacterial infection of the thyroid gland. B) viral infection of the thyroid gland. C) congenital hypothyroidism. D) autoimmune thyroiditis.

D) autoimmune thyroiditis. Autoimmune thyroiditis (Hashimoto) is the most common cause of primary hypothyroidism in adults.

A 5-month-old child is admitted to the hospital with recurring respiratory infections. A possible cause of this condition is: a. hypergammaglobulinemia. b. increased maternal IgG. c. immune insufficiency. d. decreased maternal antibody breakdown, resulting in hyposensitivity.

ANS: C Normal human infants are immunologically immature when born, with deficiencies in antibody production, phagocytic activity, and complement activity, especially components of alternative pathways. They do not possess hypergammaglobulinemia. Possessing increased maternal IgG would not lead to recurring infections. The recurrent infections are due to decreased immunity, not maternal antibody breakdown. REF: p. 173, Pediatric Considerations

The patient generally acquires nosocomial pneumonia: a. at daycare centers. b. on airplanes. c. during hospitalization. d. in the winter season.

ANS: C Nosocomial infections are acquired in the hospital. Nosocomial infections are not associated with daycare centers or airplanes and may happen in any season. REF: p. 703

When an aide asks the nurse about the purpose of the inflammatory process, how should the nurse respond? a. To provide specific responses toward antigens. b. To lyse cell membranes of microorganisms. c. To prevent infection of the injured tissue. d. To create immunity against subsequent tissue injury.

ANS: C One purpose of the inflammatory process is to prevent infection and further damage by contaminating microorganisms. Specific response toward antigens is a part of the complement system that assists in the inflammatory response, but not its purpose. Lysis of cell membranes is part of the process of phagocytosis, which removes foreign material. Immunity cannot be achieved against future tissue injury. REF: p. 137

What is the earliest manifestation of diabetes-induced kidney dysfunction? a. Polyuria b. Glycosuria c. Microalbuminuria d. Decreased glomerular filtration

ANS: C Polyuria occurs due to increased fluid in the vascular space, and microalbuminuria is the earliest manifestation. While glycosuria occurs due to hyperglycemia, it is not the first sign of kidney dysfunction. Decreased glomerular filtration and polyuria can occur due to changes, but neither are initial manifestations. REF: p. 479

Which finding would support the diagnosis of respiratory acidosis? a. Vomiting b. Hyperventilation c. Pneumonia d. An increase in noncarbonic acids

ANS: C Respiratory acidosis occurs with hypoventilation, and pneumonia leads to hypoventilation. Vomiting leads to loss of acids and then to alkalosis. Hyperventilation leads to respiratory alkalosis. Metabolic acidosis is caused by an increase in noncarbonic acids. REF: pp. 128-129

A 10-year-old male presents with fever, lymphadenopathy, arthralgia, and nosebleeds. He is diagnosed with rheumatic heart disease. The most likely cause of this disease is: a. congenital heart defects. b. human immunodeficiency virus (HIV) infections. c. group A beta-hemolytic streptococcus infections. d. acute pericarditis.

ANS: C Rheumatic fever is a systemic, inflammatory disease caused by a delayed exaggerated immune response to infection by the group A beta-hemolytic streptococcus, not by congenital heart defects, HIV infections, or acute pericarditis. REF: pp. 628-629

A 50-year-old male with a 30-year history of smoking was diagnosed with bronchogenic cancer. He developed edema and venous distention in the upper extremities and face. Which of the following diagnosis will the nurse observe on the chart? a. Thromboembolism b. Deep vein thrombosis c. Superior vena cava syndrome (SVCS) d. Chronic venous insufficiency

ANS: C SVCS is a progressive occlusion of the superior vena cava that leads to venous distention in the upper extremities and head. Thromboembolism would not lead to the generalized symptoms described in the patient. Deep vein thrombosis would not lead to upper extremity symptoms. Chronic venous insufficiency would primarily affect one extremity. REF: p. 599

During shock states, glucose uptake is usually: a. enhanced. b. normal. c. impaired. d. energy intensive.

ANS: C Some compensatory mechanisms activated by shock contribute to decreased glucose uptake by the cells, not enhanced uptake, not normal uptake, and not energy-intensive uptake. REF: p. 637

A 13-year-old took a weight loss drug that activated the sympathetic nervous system. Which of the following assessment findings would the nurse expect? a. Decreased myocardial contraction b. Decreased heart rate c. Increased cardiac conduction d. Increased intranodal conduction time

ANS: C Stimulation of the SA node by the sympathetic nervous system rapidly increases cardiac conduction. Stimulation of the sympathetic nervous system would increase myocardial contraction and heart rate. Such simulation would not affect conduction time within the node. These are inherent rates. REF: p. 576

Where is the main site of aldosterone synthesis? a. Liver b. Kidneys c. Adrenal cortex d. Hypothalamus

ANS: C The adrenal cortex, not the liver, kidneys, or the hypothalamus, synthesizes aldosterone. REF: p. 453

Airway obstruction contributing to increased airflow resistance and hypoventilation in asthma is caused by: a. type II alveolar cell injury and decreased surfactant. b. alveolar fibrosis and pulmonary edema. c. mucous secretion, bronchoconstriction, and airway edema. d. collapse of the cartilaginous rings in the bronchi.

ANS: C The mediators of asthma cause vasodilation, increased capillary permeability, mucosal edema, bronchial smooth muscle contraction (bronchospasm), and mucous secretion from mucosal goblet cells with narrowing of the airways and obstruction to airflow. Acute respiratory distress syndrome involves type II injury. Alveolar fibrosis with pulmonary edema is not associated with asthma. Asthma is not associated with collapse of bronchiolar rings. REF: p. 698

A 5-year-old male is diagnosed with a bacterial infection. Cultures of the bacteria revealed lipopolysaccharides on the bacterial cell surface. Which of the complement pathways would be activated in this case? a. Classical pathway b. Lectin pathway c. Alternative pathway d. Kinin pathway

ANS: C The alternative pathway is activated by several substances found on the surface of infectious organisms, such as those containing lipopolysaccharides. The classical pathway is primarily activated by antibodies that are proteins of the acquired immune system. The lectin pathway is similar to the classical pathway but is independent of antibody. It is activated by several plasma proteins. The kinin pathway is involved in coagulation. REF: p. 139

A 55-year-old male died of a myocardial infarction. Autopsy would most likely reveal: a. embolization of plaque from the aorta. b. decreased ventricular diastolic filling time. c. platelet aggregation within the atherosclerotic coronary artery. d. smooth muscle dysplasia in the coronary artery.

ANS: C The autopsy would reveal platelet aggregation within an atherosclerotic coronary artery. The cause of death is most likely occlusion of the coronary artery, not emboli from the aorta, decreased filling time, or dysplasia in the artery. REF: p. 613

A patient with left heart failure starts to have a cough and dyspnea. Pulmonary symptoms common to left heart failure are a result of: a. inflammatory pulmonary edema. b. decreased cardiac output. c. pulmonary vascular congestion. d. bronchoconstriction.

ANS: C The clinical manifestations of left heart failure are the result of pulmonary vascular congestion and inadequate perfusion of the systemic circulation. Pulmonary edema does occur, but it is not due to inflammation. Decreased cardiac output does occur, but the pulmonary symptoms are related to pulmonary congestion. Pulmonary symptoms are not due to bronchoconstriction. REF: p. 633

A patient most prone to multiple organ dysfunction syndrome (MODS) is a patient with: a. myocardial infarction (MI). b. pulmonary disease. c. septic shock. d. autoimmune disease.

ANS: C The most common cause of MODS is septic shock, not MI, not pulmonary disease, and not autoimmune disease. REF: p. 644

Which of the following is responsible for initiating clonal selection? a. T cells b. B cells c. Antigens d. Lymphocytes

ANS: C The lymphocytes remain dormant until an antigen initiates clonal selection. T cells do not initiate clonal selection. B cells are antibodies. Lymphocytes are released into the circulation as immature cells that react with antigens. REF: p. 159

What is the most likely cause of chronic bronchitis in a 25-year-old? a. Chronic asthma b. Air pollution c. Cigarette smoke d. Recurrent pneumonias

ANS: C The most likely cause of chronic bronchitis is cigarette smoke. Chronic bronchitis in such a patient is not commonly associated with chronic asthma. Air pollution and recurrent infections may contribute, but smoking is the most common cause. REF: p. 701

A patient presents with severe chest pain and shortness of breath and is diagnosed with pulmonary embolism. The embolism most likely originated from the: a. left ventricle. b. systemic arteries. c. deep veins of the leg. d. superficial veins of the arm.

ANS: C The most likely origin of the embolism is from the deep veins of the legs. An embolism is not likely to originate in the left ventricle, the systemic arteries, or the arms. REF: p. 606

Which is the most potent naturally occurring glucocorticoid? a. Aldosterone b. Testosterone c. Cortisol d. Prolactin

ANS: C The most potent naturally occurring glucocorticoid is cortisol, not aldosterone, testosterone, or prolactin. REF: p. 453

Local myogenic regulation of blood vessel diameter and thus blood flow through a vessel is an example of: a. autonomic regulation. b. somatic regulation. c. autoregulation. d. metabolic regulation.

ANS: C The myogenic hypothesis proposes that autoregulation originates in vascular smooth muscle, presumably of the arterioles, as a response to changes in arterial perfusion pressure. It is not a part of autonomic, somatic, or metabolic regulation. REF: p. 592

The pericardium is: a. the outer muscular layer of the heart. b. the innermost layer of the heart chambers. c. a membranous sac that encloses the heart. d. the heart's fibrous skeleton.

ANS: C The pericardium is the membranous sac that surrounds the heart. The outer layer that acts as the fibrous skeleton of the heart is the myocardium. The innermost layer of the heart is the endocardium. REF: p. 570

The molecule that aids in bonding of the troponin complex to actin and tropomyosin is troponin: a. C. b. I. c. T. d. M.

ANS: C Troponin T aids in the binding of the troponin complex to actin and tropomyosin. Troponin C contains binding sites for the calcium ions involved in contraction. Troponin I inhibits the ATPase of actomyosin. Troponin M is not a substance. REF: p. 580

The patient diagnosed with tuberculosis can transmit this disease through: a. skin contact. b. fecal-oral contact. c. airborne droplets. d. blood transfusions.

ANS: C Tuberculosis is transmitted through airborne droplets. Tuberculosis is not transmitted through skin contact, fecal-oral contact, or through a blood transfusion. REF: p. 706

A 65-year-old female is diagnosed with metastatic breast cancer. She has developed muscle wasting. Which of the following substances would be produced in large quantities to eliminate the tumor cells and cause muscle wasting? a. Interleukin-6 b. Eosinophils c. Tumor necrosis factor-alpha d. Platelets

ANS: C Tumor necrosis factor causes muscle wasting. Interleukin-6 stimulates growth and differentiation of blood cells. Eosinophils are stimulated by parasites. Platelets stimulate clotting. REF: p. 144

A 30-year-old male is demonstrating hematuria with red blood cell casts and proteinuria exceeding 3-5 grams per day, with albumin being the major protein. The most probable diagnosis the nurse will see documented on the chart is: a. cystitis. b. chronic pyelonephritis. c. acute glomerulonephritis. d. renal calculi.

ANS: C Two major symptoms distinctive of more severe glomerulonephritis are: (i) hematuria with red blood cell casts and (ii) proteinuria exceeding 3-5 grams per day with albumin (macroalbuminuria) as the major protein. Cystitis is not associated with proteinuria and so is not relevant to the diagnosis of glomerulonephritis. Chronic pyelonephritis is manifested by dysuria, not proteinuria. Proteinuria is not seen with renal calculi. REF: p. 755 | p. 757

A 25-year-old male presents to his primary care provider reporting changes in facial features. CT scan reveals a mass on the anterior pituitary, and lab tests reveal severely elevated growth hormone (GH). Which of the following would the nurse also expect to find? a. Decreased IGF-1 b. Hypotension c. Muscular atrophy d. Height increases

ANS: C With elevated levels of GH, there is resulting bony and soft tissue overgrowth; nerve entrapment occurs, leading to peripheral nerve damage manifested by weakness, muscular atrophy, foot drop, and sensory changes in the hands. IGF-1 increases, and there is an overgrowth of bone but not an increase in height or hypotension. REF: p. 465

Which valvular condition is characterized by the valve opening being constricted and narrowed, causing the valve leaflets, or cusps, to fail to open completely? a. Regurgitation b. Insufficiency c. Stenosis d. Incompetence

ANS: C Valvular stenosis occurs when the valve opening is constricted and narrowed. Valvular regurgitation occurs when blood moves backward into the chamber from which it came. Valvular insufficiency occurs when blood regurgitates backward into the chamber from which it came. Valvular incompetence leads to regurgitation. REF: p. 625

Which patient is most prone to metabolic alkalosis? A patient with: a. retention of metabolic acids. b. hypoaldosteronism. c. excessive loss of chloride (Cl). d. hyperventilation.

ANS: C When acid loss is caused by vomiting, renal compensation is not very effective because loss of Cl stimulates renal retention of bicarbonate, leading to alkalosis. Retention of metabolic acids would lead to acidosis, not alkalosis. Hypoaldosteronism leads to hyponatremia and does not cause alkalosis. Hyperventilation leads to respiratory alkalosis, not metabolic alkalosis. REF: p. 127

A nurse is caring for a patient diagnosed with SIADH. What severe complication should the nurse assess for? a. Stroke b. Diabetes insipidus c. Neurologic damage d. Renal failure

ANS: C When the hyponatremia of SIADH becomes severe, 110-115 milliequivalents per liter, confusion, lethargy, muscle twitching, convulsions, and severe and sometimes irreversible neurologic damage may occur. Neither stroke, diabetes insipidus, nor renal failure is associated with SIADH. REF: p. 462

A 42-year-old male is involved in a motor vehicle accident that has resulted in prerenal failure. What is the most likely cause of this patient's condition? a. Kidney stones b. Immune complex deposition in the glomerulus c. Inadequate renal blood flow d. Obstruction of the proximal tubule

ANS: C With blood loss, renal failure is due to inadequate blood flow since the trigger occurs outside of the kidney (prerenal). Kidney stones will lead to postrenal renal failure. Intrarenal renal failure is due to glomerular nephritis or obstruction of the proximal tubule. testbanks_and_xanax REF: p. 760

Which assessment finding would the nurse expect to be increased in a patient with deficient ADH production? a. Blood volume b. Urine osmolality c. Urine volume d. Arterial vasoconstriction

ANS: C With deficient ADH, the kidneys would not concentrate urine, leading to increased urine output. Blood volume would decrease with increased renal excretion of fluid. Urine osmolality would decrease. Arteries would dilate with deficient ADH production. testbanks_and_xanax REF: p. 448

A patient diagnosed with Addison disease reports weakness and is easily fatigued. What is the root of these symptoms? a. Hyperkalemia b. Hypoglycemia c. Hypocortisolism d. Metabolic acidosis

ANS: C With mild-to-moderate hypocortisolism, symptoms usually begin with weakness and easy fatigability. The weakness is not due to hyperkalemia, hypoglycemia, or metabolic acidosis. REF: p. 484

A patient diagnosed with Graves disease is admitted to a medical-surgical unit. Which of the following symptoms would the nurse expect to find before treatment? a. Weight gain, cold intolerance b. Slow heart rate, rash c. Skin hot and moist, rapid heart rate d. Constipation, confusion

ANS: C testbanks_and_xanax Symptoms of Graves disease include heat intolerance and increased tissue sensitivity to stimulation by the sympathetic division of the autonomic nervous system. Weight loss, rather than weight gain, and heat intolerance would result. Tachycardia, not slow heart rate, would occur. Diarrhea would occur as opposed to constipation. REF: p. 467, Figure 19-5

What is the cause of type 1 diabetes mellitus (DM)? a. A familial, autosomal dominant gene defect b. Obesity and lack of exercise c. Immune destruction of the pancreas d. Hyperglycemia from eating too many sweets

ANS: C testbanks_and_xanax The most common cause of type 1 DM is a slowly progressive autoimmune T-cell-mediated disease that destroys the beta cells of the pancreas; it is not due to a gene defect. Although obesity can contribute to diabetes, it doesn't cause type 1 DM. Eating too many sweets may contribute to the development of obesity. REF: p. 472

A 30-year-old is diagnosed with emphysema. Changes in this patient's lungs are caused by: a. viral infections. b. destruction of alveolar macrophages. c. α1-antitrypsin deficiency. d. fibrotic lung disease.

ANS: C α1-Antitrypsin deficiency is suggested in individuals who develop emphysema before 40 years of age; it is not due to viral infections. Changes in the lungs are not associated with alveolar macrophage destruction or with fibrotic lung disease. REF: p. 702

The most common cause of hypoxic injury is: A) free radicals. B) malnutrition. C) chemical toxicity. D) ischemia.

D) ischemia.

Target cells for aldosterone are located in the: A) adrenal cortex. B) adrenal medulla. C) liver. D) kidneys.

D) kidneys. Aldosterone stimulates sodium reabsorption by the kidneys.

The most common cause of hypoparathyroidism is: A) hypothalamic inactivity. B) pituitary hyposecretion. C) parathyroid adenoma. D) parathyroid gland injury or removal.

D) parathyroid gland injury or removal.

Which of the following patients is at highest risk for developing pulmonary embolism (PE)? a. 21-year-old male with a hemophilia bleeding disorder b. 28-year-old woman who had a baby 6 months earlier c. 36-year-old woman with a history of alcohol abuse who is recovering from a gastric ulcer d. 72-year-old male who is recovering from hip replacement surgery in the hospital

ANS: D A 72-year-old is at risk for immobility and at increased risk for PE. REF: p. 707

Palpation of the neck of a patient diagnosed with Graves disease would most likely reveal: a. a normal-sized thyroid. b. a small discrete thyroid nodule. c. multiple discrete thyroid nodules. d. diffuse thyroid enlargement.

ANS: D A patient with Graves disease would reveal stimulation of the gland causing diffuse thyroid enlargement. In Graves disease, the thyroid will not have nodules present. REF: p. 467, Figure 19-5

A 70-year-old hospitalized for a pelvic fracture develops a pulmonary embolism. The nurse realizes this embolus is most commonly composed of: a. fat. b. air. c. tissue fragment. d. blood clot.

ANS: D A thromboembolism is most commonly composed of a blood clot. A thromboembolism is less commonly composed of fat, air, or tissue fragments. REF: p. 706

A nurse assesses the heart after acetylcholine because the effect of acetylcholine on the heart is to: a. decrease the refractory period. b. increase calcium influx. c. increase the strength of myocardial contraction. d. decrease the heart rate.

ANS: D Acetylcholine causes decreased heart rate and slows conduction through the AV node; it does not decrease the refractory period nor does it increase calcium influx. It does not increase the strength of myocardial contraction. REF: p. 578

Diabetes insipidus, diabetes mellitus (DM), and SIADH share which of the following assessment manifestations? a. Polyuria b. Edema c. Vomiting d. Thirst

ANS: D All three share thirst as a common clinical manifestation. SIADH does not have polyuria as a clinical manifestation. Diabetes insipidus does not have edema as a clinical manifestation. SIADH is manifested by gastrointestinal symptoms; the other two are not. REF: pp. 461-462 | p. 473

Which of the following is a water-soluble amine? a. Growth hormone (GH) testbanks_and_xanax b. Luteinizing hormone (LH) c. Antidiuretic hormone (ADH) d. Epinephrine

ANS: D An example of a water-soluble amine is epinephrine. GH is a water-soluble hormone but is a peptide. LH and ADH are water-soluble hormones, but both are polypeptides. REF: p. 440, Table 18-1

Asthma is thought to be caused by: a. an autosomal recessive trait. b. autoimmunity. c. excessive use of antibiotics as a young child. d. interactions between genetic and environmental factors.

ANS: D Asthma is an interaction between genetic and environmental factors. Asthma is not a recessive trait or an autoimmune response. Asthma is not due to excessive antibiotic use in childhood. REF: p. 698

When planning care for a dehydrated patient, the nurse remembers that the principle of water balance is closely related to the balance of: a. potassium. b. chloride. c. bicarbonate. d. sodium.

ANS: D Because water follows the osmotic gradients established by changes in salt concentration, water balance is tied to sodium balance, not that of potassium, chloride, or bicarbonate. REF: pp. 116-117 | p. 118, Figure 5-5

As a result of blockage in the pulmonary artery, blood would first back up into the: a. aorta. b. left ventricle. c. pulmonary veins. d. right ventricle.

ANS: D Blockage in the pulmonary artery would cause blood to back up into the right ventricle, not the aorta since these two vessels do not communicate. The pulmonary artery and left ventricle do not communicate. Blockage in the pulmonary artery would not cause blood to back up into the pulmonary vein since the pulmonary vein takes blood to the left atrium. REF: p. 571

A nurse is reviewing lab reports. The nurse recalls blood plasma is located in which of the following fluid compartments? a. Intracellular fluid (ICF) b. Extracellular fluid (ECF) c. Interstitial fluid d. Intravascular fluid

ANS: D Blood plasma is the intravascular fluid. ICF is fluid in the cells. ECF is all the fluid outside the cells. Interstitial fluid is fluid between the cells and outside the blood vessels. REF: p. 114

A 30-year-old White female was recently diagnosed with primary hypertension. She reports that she eats fairly well, usually moderate red meat consumption. She also reports that her father has hypertension as well. A nurse determines which of the following risk factors is most likely associated with this diagnosis? a. Race b. Diet c. Age d. Genetic

ANS: D Genetic factors, such as family history of hypertension, are the number one factor in the development of hypertension. Race and diet may be factors, but genetic factors are primary. Age is a factor, but not in this case; since the patient is 30, genetics are a greater factor. REF: p. 600

The cardiac electrical impulse normally begins spontaneously in the sinoatrial (SA) node because it: a. has a superior location in the right atrium. b. is the only area of the heart capable of spontaneous depolarization. c. has rich sympathetic innervation via the vagus nerve. d. depolarizes more rapidly than other automatic cells of the heart.

ANS: D Cardiac impulses occur in the SA node because its cells depolarize more rapidly than other automatic cells, not because of its superior location. The SA node is not the only area of the heart capable of spontaneous depolarization. The SA node does have rich innervations but by parasympathetic innervation from the vagus nerve. REF: p. 576

When catecholamines are released in a patient, what should the nurse assess for? a. Nutrient absorption b. Fluid retention c. Hypotension d. Hyperglycemia

ANS: D Catecholamines cause hyperglycemia and immune suppression. Catecholamines do not cause nutrient absorption, fluid retention, or hypotension. REF: p. 456

A patient is diagnosed with coronary artery disease. Which of the following modifiable risk factors would the nurse suggest the patient change? a. Eating meat b. Living arrangements c. Drinking tomato juice d. Smoking cigarettes

ANS: D Cigarette smoking leads to vasoconstriction and should be the first behavior the patient changes. Eating meat alone would not lead to the development of coronary artery disease. The living arrangement of the patient's home would not lead to the development of coronary artery disease. Drinking tomato juice would not lead to the development of coronary artery disease. REF: p. 611| p. 612

A 44-year-old patient with pulmonary tuberculosis is evaluated for SIADH. Which assessment finding would support this diagnosis? a. Peripheral edema b. Tachycardia testbanks_and_xanax c. Low blood pressure d. Concentrated urine

ANS: D Clinical manifestations of SIADH include urine that is inappropriately concentrated with respect to serum osmolarity. Symptomology of SIADH does not include peripheral edema, tachycardia, or low blood pressure. REF: p. 462

Which condition should cause the nurse to assess for high-output failure in a patient? a. Metabolic alkalosis b. Hypothyroidism c. Hypovolemia d. Anemia

ANS: D Common causes of high-output failure include anemia, not metabolic alkalosis, not hypothyroidism, and not hypovolemia. REF: p. 636

A 35-year-old male is diagnosed with lobar pneumonia (lung infection). Which of the following exudates would be present in highest concentration at the site of this advanced inflammatory response? a. Serous b. Purulent c. Hemorrhagic d. Fibrinous

ANS: D Fibrinous exudates occur in the lungs of individuals with pneumonia. Serous fluid is watery fluid, as in a blister. Purulent is characterized by an abscess, such as pus. Hemorrhagic occurs when the exudates are filled with erythrocytes. REF: p. 149

A 15-year-old is diagnosed with restrictive lung disease caused by fibrosis. Which pulmonary function test finding is expected? a. Increased compliance b. Increased tidal volume c. Decreased respiratory rate d. Decreased functional residual capacity

ANS: D Fibrosis progressively obliterates the alveoli, respiratory bronchioles, and interstitium (fibrosing alveolitis), which can result in chronic pulmonary insufficiency, and functional residual capacity declines. Compliance decreases in restrictive disease. In restrictive disease, tidal volume decreases. Respirations may increase with restrictive disease. REF: p. 696

For an infection to progress to septic shock, what must occur? a. The individual must be immunosuppressed. b. The myocardium must be impaired. c. The infection must be gram negative. d. Bacteria must enter the bloodstream.

ANS: D For septic shock to occur, bacteria must enter the bloodstream. Septic shock can occur in individuals who are not immunosuppressed. In septic shock, the myocardium is not impaired. Many organisms in addition to gram-negative bacteria can cause septic shock. REF: p. 645

A 54-year-old male intravenous (IV) drug user is diagnosed with chronic hepatitis C. Testing revealed that he is a candidate for treatment. Which of the following could be used to treat his condition? a. Interleukin-1 b. Interleukin-6 c. Interleukin-10 d. INFs

ANS: D INFs are members of a family of cytokines that protect against viral infections. Interleukin-1 is responsible for fever production. Interleukin-6 stimulates growth and differentiation of blood cells. Interleukin-10 helps decrease the immune response. REF: p. 144

A 75-year-old female has been critically ill with multiple organ dysfunction syndrome (MODS) for longer than a week and has developed a severe oxygen supply and demand imbalance. The statement that best describes this imbalance is which of the following? a. Increased oxygen delivery to cells fails to meet decreased oxygen demands. b. The amount of oxygen consumed by cells depends only on the needs of cells, because there is oxygen in reserve. c. The situation results in supply-independent consumption. d. The reserve has been exhausted, and the amount of oxygen consumed depends on the amount the circulation is able to deliver.

ANS: D In MODS, the reserve has been exhausted and the body cannot meet the oxygenation demands. It is true that oxygen fails to meet demand, but there is no increase in oxygen because reserves are exhausted. There is no oxygen in reserve. The situation is supply and demand, but the demand cannot be met. REF: p. 648

An experiment was designed to test the effects of the Starling forces on fluid movement. Which of the following alterations would result in fluid moving into the interstitial space? a. Increased capillary oncotic pressure. b. Increased interstitial hydrostatic pressure. c. Decreased capillary hydrostatic pressure. d. Increased interstitial oncotic pressure.

ANS: D Increased interstitial oncotic pressure would attract water from the capillary into the interstitial space. Increased capillary oncotic pressure would attract water from the interstitial space back into the capillary. Increased interstitial hydrostatic pressure would attract movement of water from the interstitial spaces into the capillary. Decreased capillary hydrostatic pressure would move water into the capillaries. REF: p. 115

A child fell off a swing and scraped his right knee. The injured area becomes painful. What else will the nurse observe upon assessment? a. Vasoconstriction at injured site b. Decreased RBC concentration at injured site c. Pale skin at injured site d. Edema at injured site

ANS: D Increased vascular permeability and leakage of fluid out of the vessel cause edema at the site of injury. Vasodilation occurs, bringing increased RBCs to the site and causing redness. REF: p. 137

A nurse recalls insulin has an effect on which of the following groups of electrolytes? a. Sodium, chloride, phosphate b. Calcium, magnesium, potassium c. Hydrogen, bicarbonate, chloride d. Potassium, magnesium, phosphate

ANS: D Insulin facilitates the intracellular transport of potassium (K+), phosphate, and magnesium. Insulin does not facilitate the intracellular transport of sodium, chloride, calcium, hydrogen, or bicarbonate. REF: pp. 451-452

Exposure to ionizing radiation is linked to all of the following cancers except: A) leukemia. B) thyroid cancer. C) breast cancer. D) skin cancer.

D) skin cancer.

A 42-year-old female presents to her primary care provider reporting muscle weakness and cardiac abnormalities. Laboratory tests indicate that she is hypokalemic. Which of the following could be the cause of her condition? a. Respiratory acidosis b. Constipation c. Hypoglycemia d. Laxative abuse

ANS: D Losses of potassium from body stores are usually caused by gastrointestinal and renal disorders. Diarrhea, intestinal drainage tubes or fistulae, and laxative abuse also result in hypokalemia. Acidosis is related to hyperkalemia, not hypokalemia. Constipation can occur with hypokalemia but does not cause it. Hypoglycemia is not related to muscle weakness. REF: p. 123

The macrophage secretion that stimulates procollagen synthesis and secretion is: a. angiogenesis factor. b. matrix metalloproteinase. c. vascular endothelial growth factor. d. transforming growth factor-beta

ANS: D Macrophages secrete transforming growth factor-beta to stimulate fibroblasts to secrete the collagen precursor procollagen. Angiogenesis factor supports the growth of new vessels. Matrix metalloproteinase remodels proteins at the site of injury. Vascular endothelial growth factors are also involved in vessel growth. REF: p. 152

How do the releasing hormones that are made in the hypothalamus travel to the anterior pituitary? a. Vessels of the zona fasciculata b. Chromophils c. Median eminence d. Hypophyseal portal system

ANS: D Neurons in the hypothalamus secrete releasing hormones into veins that carry the releasing hormones directly to the vessels of the adenohypophysis via the hypophyseal portal system, thus bypassing the normal circulatory route. Zona fasciculata secretes abundant amounts of cortisol from the adrenal gland. Chromophils are the secretory cells of the adenohypophysis. The median eminence is part of the posterior pituitary, not the anterior. REF: p. 8

A nurse is preparing to teach on the subject of opsonins. Which information should the nurse include? Opsonins are molecules that: a. are composed of fatty acids. b. regulate inflammation. c. degranulate mast cells. d. enhance phagocytosis.

ANS: D Opsonins coat the surface of bacteria and increase their susceptibility to being phagocytized. Opsonins are not associated with fatty acids, the regulation of the inflammatory process, or the degranulation of mast cells. REF: p. 138

A patient is diagnosed with orthostatic hypotension. Which of the following symptoms would most likely be reported? a. Headache and blurred vision b. Nausea and vomiting c. Chest pain and palpitations d. Syncope and fainting

ANS: D Orthostatic hypotension is often accompanied by dizziness, blurring or loss of vision, and syncope or fainting, not by headache or blurred vision, which are symptoms of hypertension. Chest pain and palpitations may be symptomatic of myocardial infarction. Nausea and vomiting are associated with gastrointestinal issues. REF: p. 604

Where are the target cells for oxytocin located? a. Renal tubules b. Thymus c. Liver d. Uterus

ANS: D Oxytocin causes uterine contraction and lactation in women and may have a role in sperm motility in men. Oxytocin does not stimulate the renal tubules, the thymus, or the liver. REF: p. 448

A 39-year-old female is recovering from the birth of her third child. Which hormone would help prevent uterine bleeding? a. Aldosterone b. Cortisol c. Prolactin d. Oxytocin

ANS: D Oxytocin functions near the end of labor to enhance effectiveness of contractions, promote delivery of the placenta, and stimulate postpartum uterine contractions, thereby preventing excessive bleeding. Aldosterone regulates water balance. Cortisol helps protect from stress. Prolactin assists with milk production. REF: p. 448

A 60-year-old with a history of cirrhosis presents with dyspnea, impaired ventilation, and pleural pain. A diagnosis of pleural effusion is made, and a watery fluid is drained. When giving report, the nurse will refer to this fluid as: a. exudative. b. purulent. c. infected. d. transudative.

ANS: D Pleural effusions that enter the pleural space from intact blood vessels can be transudative (watery). The fluid is watery; thus, it is not exudative, which is composed of white cells. Effusion is not purulent or infected. REF: p. 692

Pneumonia is caused by: a. use of anesthetic agents in surgery. b. atelectasis. c. chronic lung changes seen with aging. d. viral or bacterial infections.

ANS: D Pneumonia is caused by a viral or bacterial infection; infections are not caused by anesthetic agents. Neither atelectasis nor lung changes associated with normal aging cause pneumonia. REF: p. 703

Which patient should the nurse assess for both hyperkalemia and metabolic acidosis? A patient diagnosed with: a. diabetes insipidus. b. pulmonary disorders. c. Cushing syndrome. d. renal failure.

ANS: D Renal failure is associated with hyperkalemia and metabolic acidosis. Diabetes insipidus results in hypernatremia. Pulmonary disorders are a cause of respiratory acidosis or alkalosis but do not affect hyperkalemia. Cushing syndrome results in hypernatremia. REF: p. 124 | p. 127

A patient who has elevated thyroxine production should be assessed for which accompanying condition? a. Increased thyroid-releasing hormone (TRH) b. Increased anterior pituitary stimulation c. Decreased T4 d. Decreased thyroid-stimulating hormone (TSH)

ANS: D Secretion of TSH stimulates the synthesis and secretion of thyroid hormones. Increasing levels of T4 and T3 then feedback negatively on the pituitary and hypothalamus to inhibit TRH and TSH synthesis. With increased thyroxine production, TRH will be decreased. Increased thyroxine would lead to decreased anterior pituitary stimulation. Thyroxine is T4; its level will be elevated. REF: p. 440

A patient is diagnosed with renal calculus that is causing a urinary obstruction. Which symptoms would be most likely experienced? a. Anuria b. Hematuria c. Pyuria d. Flank pain

ANS: D Significant flank pain is the most common manifestation. Neither anuria nor pyuria is a common manifestation. Hematuria does occur, but it is not the most common manifestation. REF: p. 749

What can trigger ACTH to be released? a. High serum levels of cortisol b. Hypotension c. Hypoglycemia d. Stress

ANS: D Stress increases ACTH secretion. ACTH regulates the release of cortisol from the adrenal cortex. It is not stimulated by high serum cortisol levels. Neither hypotension nor hypoglycemia stimulates ACTH secretion. REF: p. 455

A 30-year-old presents with hypertension, headache, tachycardia, impaired glucose tolerance, and weight loss. Which of the following diagnoses is supported by this symptomology? a. Addison disease b. Conn disease c. Cushing disease d. Pheochromocytoma

ANS: D Symptoms of pheochromocytoma include hypertension, palpitations, tachycardia, glucose intolerance, excessive sweating, and constipation. Manifestations of Addison disease include weakness, fatigability, hypoglycemia and related metabolic problems, lowered response to stressors, hyperpigmentation, vitiligo, and manifestations of hypovolemia and hyperkalemia. Hypertension and hypokalemia are the hallmarks of Conn disease. Weight gain is the most common feature in Cushing disease and results from the accumulation of adipose tissue in the trunk, facial, and cervical areas. These characteristic patterns of fat deposition have been described as "truncal obesity," "moon face," and "buffalo hump." REF: p. 485

Which structures would not receive an electrical impulse when the patient is experiencing a left bundle branch block? a. Atrioventricular (AV) node b. Sinoatrial (SA) node c. Bundle of His d. The left ventricle

ANS: D The left bundle branch conducts impulses to the left ventricle; the AV node feeds conduction of the bundles. The SA node initiates conduction while the bundle of His precedes the division of the left and right bundles. REF: p. 576

Which of the following statements indicates more teaching is needed regarding secondary lymph organs? ________ is/are a secondary lymphoid organ. a. The spleen b. Peyer patches c. Adenoids d. The liver

ANS: D The liver is not a secondary lymph organ. The spleen, Peyer patches, and adenoids are secondary lymphoid organs. REF: p. 161, Figure 7-3

A 25-year-old female is diagnosed with a urinary tract infection. When the nurse checks the culture results, which of the following organisms is most likely infecting her urinary tract? a. Streptococcus b. Candida albicans c. Chlamydia d. Escherichia coli

ANS: D The most common infecting microorganisms are uropathic strains of Escherichia coli. Urinary tract infections are not associated with Streptococcus, Candida albicans, or Chlamydia. REF: p. 754

The body's inability to conserve water and sodium when affected by Addison disease is explained by which of the following conditions? a. Elevated levels of cortisol b. Decreased levels of ACTH c. Hypersecretion of ADH d. Aldosterone deficiency

ANS: D The symptoms of Addison disease are primarily a result of hypocortisolism, elevated serum ACTH, and hypoaldosteronism. ADH does not play a role in Addison disease. testbanks_and_xanax REF: pp. 484-485

The thoracic lymphatic duct drains into the: a. left subclavian artery. b. right atrium. c. right subclavian vein. d. left subclavian vein.

ANS: D The thoracic duct drains lymph into the left subclavian vein, not the right subclavian vein. The right lymphatic duct drains into the right subclavian vein. The thoracic duct does not drain lymph into the right atrium. REF: p. 593

A 19-year-old female with type 1 DM was admitted to the hospital with the following lab values: serum glucose 500 milligrams per deciliter (high), urine glucose and ketones 4+ (high), and arterial pH 7.20 (low). Her parents state that she has been sick with the "flu" for a week. Which of the following statements best explains her acidotic state? a. Increased insulin levels promote protein breakdown and ketone formation. b. Her uncontrolled diabetes has led to renal failure. c. Low serum insulin promotes lipid storage and a corresponding release of ketones. d. Insulin deficiency promotes lipid metabolism and ketone formation.

ANS: D testbanks_and_xanax With insulin deficiency, lipolysis is enhanced, and there is an increase in the amount of nonesterified fatty acids delivered to the liver. The consequence is increased glyconeogenesis contributing to hyperglycemia and production of ketone bodies (acetoacetate, hydroxybutyrate, and acetone) by the mitochondria of the liver at a rate that exceeds peripheral use. Insulin levels are decreased. There is no evidence that the patient is in renal failure. Insulin is low, but the ketones are the result of fatty acid breakdown due to lack of insulin, not because of lipid storage. REF: p. 477

a 30-year-old female presents with a gunshot wound to the head. the wound has seared edges and a deep penetration of smoke and gunpowder fragments. This wound would be documented as a(n): a. exit b. intermediate range entrance c. contact range entrance d. indeterminate range entrance

ANS: c a contact range entrance wound is a distinctive typeof wound that happens when a gun is held so the muzzle rests on or presses into the skin surface; in addition to the hole, there is earing of the edges of the wound from flame and soot or smoke on the edges of the wound. It is unlikely it is an exit wound since there is only one wound and it has seared edges and gunpowder fragments. An intermediate range entrance woudn is surrounded by gunpowder tattooing or stippling. An indeterminate range entrance wound occurs when flame, soot, or gunpowder does not reach the skin surface but the bullet does. REF: p.95, table 4-8

A 10-year-old is brought to the ER with prolonged bronchospasm and severe hypoxemia. The symptomology supports which diagnosis? a. Exercise-induced asthma b. Chronic obstructive pulmonary disease (COPD) c. Status asthmaticus d. Bronchiectasis

ANS: c When bronchospasm is not reversed by the usual measures, the individual is considered to have severe bronchospasm or status asthmaticus since exercise-induced asthma resolves. COPD is manifested by air trapping and hypercapnia, not by bronchospasm. Bronchiectasis is manifested by bronchiolar changes, not by bronchospasm. REF: p. 699

ANS: C Epigastric or midabdominal pain ranging from mild abdominal discomfort to severe, incapacitating pain is one of the manifestations of pancreatitis. Right lower pain would be a symptom of appendicitis. Right upper quadrant pain would be manifestation of liver inflammation. Suprapubic pain would be related to full bladder or colon problems. REF: p. 937

Acute pancreatitis often manifests with pain to which of the following regions? a. Right lower quadrant b. Right upper quadrant c. Epigastric d. Suprapubic

ANS: B In moderate diffuse axonal injury, the score on the GCS is 4-8 initially and 6-8 by 24 hours, and the person is confused and suffers a long period of posttraumatic anterograde and retrograde amnesia. In mild diffuse axonal injury, coma lasts 6-24 hours, with 30% of persons displaying decerebrate or decorticate posturing. They may experience prolonged periods of stupor or restlessness. In severe diffuse axonal injury, the person experiences immediate autonomic dysfunction that disappears in a few weeks. Increased ICP appears 4-6 days after the injury. In postconcussive syndrome, the individual experiences headache, nervousness or anxiety, irritability, insomnia, depression, inability to concentrate, forgetfulness, and fatigability. REF: p. 394

After falling, a patient's Glasgow Coma Scale (GCS) was 5 initially and 7 after 1 day. The patient remained unconscious for 2 weeks but is now awake, confused, and experiencing anterograde amnesia. This history supports which medical diagnosis? a. Mild diffuse brain injury b. Moderate diffuse brain injury c. Severe diffuse brain injury d. Postconcussive syndrome

ANS: D After initial compensation, the blood flows faster and more turbulently than normal blood, causing a hyperdynamic circulatory state. Blood viscosity decreases rather than increases. Cardiac output increases. Alteration in coagulation does not occur. REF: p. 513 | p. 515

After initial compensation, what hemodynamic change should the nurse monitor for in a patient who has a reduction in the number of circulating erythrocytes? a. Increased viscosity of blood b. Decreased cardiac output c. Altered coagulation d. Hyperdynamic circulatory state

ANS: B An immediate concern for the infant with noncommunicating hydrocephalus is interstitial edema. Neither metabolic, vasogenic, nor ischemic edema is observed as a result of noncommunicating hydrocephalus. REF: p. 376

An infant is diagnosed with noncommunicating hydrocephalus. What is an immediate priority concern for this patient? a. Metabolic edema b. Interstitial edema c. Vasogenic edema d. Ischemic edema

ANS: B Anaplasia, not autonomy, is the loss of differentiation. The term pleomorphic refers to a marked variability of size and shape. A malignant tumor has the ability to spread far beyond the tissue of origin by the process of metastasis. REF: p. 234

An oncologist is discussing when a cancer cell loses differentiation. Which of the following is the oncologist describing? a. Autonomy b. Anaplasia c. Pleomorphic d. Metastasis

ANS: D In aplastic anemia, erythrocyte stem cells are underdeveloped, defective, or absent; iron is not deficient, there are no excess levels of erythropoietin, nor do cells undergo hemolysis. REF: p. 520, Table 21-2

Aplastic anemia is caused by: a. iron deficiency. b. excess levels of erythropoietin. c. hemolysis. d. stem cell deficiency.

respiratory alkalosis

Arise in blood pH due to hyperventilation (excessive breathing) and a resulting decrease in CO2.

A 50-year-old diabetic male did not take his medication and is now in metabolic acidosis. He is experiencing Kussmaul respirations. What type of breathing will the nurse observe upon assessment? A. Rapid respirations with periods of apnea B. A slightly increased ventilatory rate, large tidal volumes, and no expiratory pause C. Audible wheezing or stridor D. Very slow inhalations and rapid expirations

B. A slightly increased ventilatory rate, large tidal volumes, and no expiratory pause

What fraction of total body water (TBW) volume is contained in the intracellular space? A) 3/4 B) 2/3 C) 1/2 D) 1/3

B) 2/3

A monocyte is a circulating white blood cell that transforms into which of the following cells once it enters the tissue during an inflammatory response? A) Neutrophil B) Macrophage C) Mast cell D) Fibroblast

B) Macrophage

Cellular atrophy involves: A) an increase in cell size. B) a decrease in cell size. C) an increase in the number of cells. D) a decrease in the number of cells.

B) a decrease in cell size.

Hair loss from chemotherapy treatment is condition known as: A) folliculitis. B) alopecia. C) erythema. D) sterility.

B) alopecia.

The process by which cells program themselves to die is called: A) karyolysis. B) apoptosis. C) pyknosis. D) necrosis.

B) apoptosis.

The most common cause of Addison disease is: A) adrenal cancer. B) autoimmune injury to the adrenal cortex. C) viral infection of the pituitary gland. D) bacterial infection of the adrenal medulla.

B) autoimmune injury to the adrenal cortex.

The condition in which the immune cells attack the individual's own tissues as if they were invaders is called: A) alloimmunity. B) autoimmunity. C) hypersensitivity. D) graft rejection.

B) autoimmunity.

Hypernatremia can result from: A) decreased aldosterone secretion. B) decreased antidiuretic hormone secretion. C) compulsive water drinking. D) excessive dietary potassium.

B) decreased antidiuretic hormone secretion.

Neurological symptoms of SIADH are related to: A) hypotension and cellular dehydration B) decreased serum sodium concentrations. C) increased serum osmolarity. D) hypokalemia.

B) decreased serum sodium concentrations. ADH secretion in SIADH causes a dilutional hyponatremia and decreased serum osmolarity. Blood pressure in SIADH in elevated because of volume retention. Alterations in ADH secretion do not seriously affect potassium levels in the body since the majority of total body potassium is intracellular.

The process of a phagocyte squeezing through retracted endothelial cells to enter into the tissues is called: A) fusion. B) diapedesis. C) phagocytosis. D) margination.

B) diapedesis.

Chronic infection of the cervix by the human papilloma virus results in cervical: A) atrophy. B) dysplasia. C) metaplasia. D) hormonal hyperplasia.

B) dysplasia.

Chronic infection of the cervix by the human papillomavirus results in cervical: A) atrophy. B) dysplasia. C) metaplasia. D) hormonal hyperplasia.

B) dysplasia.

A 60-year-old male is diagnosed with cerebral aneurysm. Where does the nurse suspect the cerebral aneurysm is located? A. Basilar artery B. Circle of Willis C. Carotid arteries D. Vertebral arteries

B. Circle of Willis

A 23-year-old female begins having problems with tiredness, weakness, and visual changes. Her diagnosis is multiple sclerosis (MS). What is occurring in the patient's body? A. Depletion of dopamine in the central nervous system (CNS) B. Demyelination of nerve fibers in the CNS C. The development of neurofibril webs in the CNS D. Reduced amounts of acetylcholine at the neuromuscular junction

B. Demyelination of nerve fibers in the CNS

A nurse is teaching about the functions of the pulmonary system. Which information should the nurse include? One of the functions of the pulmonary system is the: A. Expelling of bacteria B. Exchange of gases between the environment and blood C. Movement of blood into and out of the capillaries D. Principle mechanism for cooling of the heart

B. Exchange of gases between the environment and blood

ANS: C Bleeding from the upper GI tract can also be rapid enough to produce hematochezia (bright red stools). Melena is a black or tarry stool. Occult bleeding is hidden bleeding. Hematemesis is vomiting blood. REF: p. 909

Bright red bleeding from the rectum is referred to as: a. melena. b. occult bleeding. c. hematochezia. d. hematemesis.

Which of the following cells plays an active role in collagen deposition during wound contraction and scar tissue formation? A) Mast cell B) Macrophage C) Fibroblast D) Osteocyte

C) Fibroblast

In general, respiratory acidosis is caused by: A) states that induce hyperventilation. B) compensation for metabolic alkalosis. C) respiratory disease causing retention of carbon dioxide. D) hormone imbalances.

C) respiratory disease causing retention of carbon dioxide.

Osmosis describes the movement of: A) electrolytes. B) glucose. C) water. D) blood.

C) water.

Which of the following is not a function of antibodies? A) Neutralizing bacterial toxins B) Preventing viruses from entering tissue cells C) Direct cell killing through the release of toxic chemicals D) Opsonizing foreign antigen

C) Direct cell killing through the release of toxic chemicals

When a person has an allergic reaction to bee stings, which antibody causes the life-threatening hypersensitivity response? A) IgM B) IgA C) IgE D) IgG

C) IgE

Which of the following statements about mutations is correct? A) Mutations always lead to genetic disease. B) Spontaneous mutations occur as a result of exposure to a mutagen. C) Mutations are alterations in normal DNA sequence. D) Mutations are always inherited.

C) Mutations are alterations in normal DNA sequence.

Which of the following groups of symptoms are caused by hyponatremia? A) Weak pulses, low blood pressure, and increased heart rate B) Thirst, dry mucous membranes, and diarrhea C) Nausea, muscle cramps, and confusion D) Cardiac arrhythmias, paresthesias, and acid-base imbalances

C) Nausea, muscle cramps, and confusion

A tumor that has distant lymph node involvement but no evidence of distant metastasis to other tissues is classified as: A) Stage I. B) Stage II. C) Stage III. D) Stage IV.

C) Stage III.

Molecules that are capable of inducing an immune response are called: A) lymphocytes. B) haptens. C) antigens. D) antibodies.

C) antigens.

The mast cell, a major activator of inflammation, initiates the inflammatory response through the process of: A) cytokinesis. B) endocytosis. C) degranulation. D) hemolysis.

C) degranulation.

A 20-year-old female has been diagnosed with AIDS. Laboratory testing would reveal diminished levels of: A) cytotoxic T cells. B) B cells. C) helper T cells. D) T suppressor cells

C) helper T cells.

If the surface barriers such as the skin or mucus membranes are breached, the second line of defense in innate immunity is the: A) lymph node. B) lymphocyte response. C) inflammatory response. D) memory cells.

C) inflammatory response.

Hyperglycemia and lipid abnormalities in type 2 diabetes mellitus (DM) are a result of: A) production of inactive insulin. B) glucagon deficiency. C) insulin resistance. D) glycogen excess.

C) insulin resistance.

One systemic manifestation of the acute inflammatory response is fever, which is induced by several mediators, including: A) histamine. B) interferon. C) interleukin 1. D) nitrous oxide.

C) interleukin 1.

Plasma membrane receptors are available to bind to: A) carbon dioxide. B) ribosomes. C) ligands. D) cell adhesion molecules.

C) ligands.

The extension of cancer to distant tissues and organs is called: A) apoptosis. B) tropism. C) metastasis. D) malignancy.

C) metastasis.

In contrast with the inflammatory response, the immune response: A) is more effective at fighting microorganisms. B) is faster. C) recognizes specific invaders. D) has no memory.

C) recognizes specific invaders.

Syndrome of inappropriate diuretic hormone (SIADH) results in excessive: A) renal retention of sodium and water. B) renal retention of sodium without water retention. C) renal retention of water without sodium retention. D) renal excretion of water without sodium retention.

C) renal retention of water without sodium retention. Elevated ADH secretion in SIADH stimulates increased water reabsorption in the distal and collecting tubules which dilutes serum electrolyte levels.

Which of the following patients is at highest risk for developing pulmonary embolism (PE)? A. 21-year-old male with a hemophilia bleeding disorder B. 36-year-old woman with a history of alcohol abuse who is recovering from a gastric ulcer C. 72-year-old male who is recovering from hip replacement surgery in the hospital D. 28-year-old woman who had a baby 6 months earlier

C. 72-year-old male who is recovering from hip replacement surgery in the hospital

A 52-year-old female is admitted to the cardiac unit with a diagnosis of pericarditis. She asks the nurse to explain where the infection is. In providing an accurate description, the nurse states that the pericardium is: A. The outer muscular layer of the heart B. The innermost layer of the heart chambers C. A membranous sac that encloses the heart D. The heart's fibrous skeleton

C. A membranous sac that encloses the heart

A 26-year-old female recently underwent surgery and is now experiencing dyspnea, cough, fever, and leukocytosis. Tests reveal that she has a collapsed lung caused by removal of air from obstructed alveoli. What condition will the nurse observe on the chart? A. Compression atelectasis B. Bronchiectasis C. Absorption atelectasis D. Hypoventilation

C. Absorption atelectasis

A 67-year-old female was previously diagnosed with rheumatic heart disease. Tests reveal lipoprotein deposition with chronic inflammation that impairs blood flow from the left ventricle into the aorta. Which of the following is the most likely diagnosis recorded on the chart? A. Mitral stenosis B. Aortic regurgitation C. Aortic stenosis D. Mitral regurgitation

C. Aortic stenosis

A compensatory alteration in the diameter of cerebral blood vessels in response to increased intracranial pressure is called: A. Herniation B. Vasodilation C. Autoregulation D. Amyotrophy

C. Autoregulation

A patient researches baroreceptors online. Which information indicates a good understanding? Baroreceptors are located in the: A. Renal artery B. Superior vena cava C. Carotid sinus D. Circle of Willis

C. Carotid sinus

Which term is used to describe a bone fracture that results in two or more fragments? A. Complete. B. Open. C. Comminuted. D. Oblique.

C. Comminuted.

A man comes into the doctors office complaining of his stomach being in terrible pain for the last month and that he has had bloody diarrhea nine different times already today, these symptoms point towards? A. Crohns disease B. Ulcerative colitis C. Diverticulitis D. Irritable bowel syndrome

C. Diverticulitis

When a staff member asks where venous blood from the coronary circulation drains into, what is the best response by the nurse? The: A. Superior vena cava B. Inferior vena cava C. Right atrium D. Left atrium

C. Right atrium

abdominal pain

bowel obstruction,inflammation and infection

Forces favoring filtration

Capillary hydrostatic pressure (blood pressure) and interstitial oncotic pressure (water-pulling)

ANS: B Cholecystitis can be acute or chronic, but both forms are almost always caused by a gallstone lodged in the cystic duct. Accumulation of bile in the hepatic duct would not lead to cholecystitis. Neither the accumulation of fat nor a viral infection leads to cholecystitis. REF: p. 936

Cholecystitis is inflammation of the gallbladder wall usually caused by: a. accumulation of bile in the hepatic duct. b. obstruction of the cystic duct by a gallstone. c. accumulation of fat in the wall of the gallbladder. d. viral infection of the gallbladder.

___ is the major anion in the ECF.

Cl

ANS: A Clinical manifestations of bile salt deficiency are related to poor intestinal absorption of fat and fat-soluble vitamins (A, D, E, and K). Neither water-soluble vitamins nor minerals require bile salts for absorption; thus, they are not affected. Protein breakdown is facilitated by bile, but its absorption is not dependent upon it. REF: p. 920

Clinical manifestations of bile salt deficiencies are related to poor absorption of: a. fats and fat-soluble vitamins. b. water-soluble vitamins. c. proteins. d. minerals.

hypersecretion of adrenocorticotropic hormone (ACTH)

Cushing syndrome

If a person has resistance to a disease from natural exposure to an antigen, the person has which form of immunity? A) Natural B) Innate C) Passive acquired D) Active naturally acquired

D) Active naturally acquired

What is the purpose of vasodilatation and increased vascular permeability during inflammation? A) To bring white blood cells to the area of injury B) To transport inflammatory chemicals to the area of injury C) To dilute toxins D) All of the above

D) All of the above

Which of the following conditions can result in a pure water deficit (hypertonic dehydration)? A) Hyperventilation caused by fever B) Inability to concentrate the urine C) Coma D) All of the above

D) All of the above

A child fell off the swing and scraped her knee. The injured area becomes red and painful. Which of the following would also occur? A) Vasoconstriction B) Decreased RBC concentration at injured site C) Fluid movement from tissue to vessels D) Edema at injured site

D) Edema at injured site

Which of the following alterations is evidence that the kidneys are compensating for a respiratory acidosis condition? A) Decreased carbon dioxide B) Elevated carbon dioxide C) Decreased bicarbonate ion D) Elevated bicarbonate ion

D) Elevated bicarbonate ion

A 35-year-old male is diagnosed with lobar pneumonia (lung infection). Which of the following exudates would be present in highest concentration at the site of this advanced inflammatory response? A) Serous B) Purulent C) Hemorrhagic D) Fibrinous

D) Fibrinous

Which cell stimulates both the cell-mediated and humoral immune responses? A) Plasma cells B) Cytotoxic T cells C) B lymphocytes D) Helper T cells

D) Helper T cells

Which cell ingests microorganisms for the purposes of presenting their antigen to the immune system and activating an immune response? A) Helper T cell B) Mast cell C) Neutrophil D) Macrophage

D) Macrophage

Which of the following exudates would be present in highest concentration at the site of a persistent bacterial infection? A) Fibrinous B) Serous C) Hemorrhagic D) Purulent

D) Purulent

A patient has been diagnosed with prostate cancer that has metastasized to the bone. The stage of this cancer is: A. 1 B. 2 C. 3 D. 4

D. 4

A patient comes into the ER and is shaking and states that she has a sharp pain that's in her upper stomach going all the way up into her back. Her labs come back and show she is experiencing hyperglycemia and a high WBC count, what is she most likely experiencing? A. Diabetes crisis B. Type II diabetes C. Cholilithiasis D. Acute pancreatitis

D. Acute pancreatitis

A 72-year-old female has a history of hypertension and atherosclerosis. An echocardiogram reveals backflow of blood into the left ventricle. Which of the following is the most likely diagnosis documented on the chart? A. Mitral stenosis B. Aortic stenosis C. Mitral regurgitation D. Aortic regurgitation

D. Aortic regurgitation

Which statement by a patient indicates teaching was successful regarding myasthenia gravis? Myasthenia gravis results from: A. Viral infection of skeletal muscle B. Atrophy of motor neurons in the spinal cord C. Demyelination of skeletal motor neurons D. Autoimmune injury at the neuromuscular junction

D. Autoimmune injury at the neuromuscular junction

A 42-year-old male was involved in a motor vehicle accident during which he suffered a severe head injury. He died shortly after the accident from loss of respiration. The nurse suspects the area of the brain most likely involved is the: A. Cerebral cortex B. Thalamus C. Basal ganglia D. Brainstem

D. Brainstem

As a portion of the assessment of the patient with meningitis, the health care provider flexes the patient's neck to the chest. The patient responds with flexion of the legs and thighs. The name for this response is: A. Decorticate posturing B. Decerebrate posturing C. Kerning's sign D. Brudzinski's sign

D. Brudzinski's sign

A 30-year-old female received a severe head injury in a motor vehicle accident. She is now experiencing respiratory abnormalities characterized by alternating periods of deep and shallow breathing with periods of apnea. What term should the nurse use when charting this condition? A. Frank-Starling B. Orthopnea C. Apnea D. Cheyne-Stokes

D. Cheyne-Stokes

A nurse assesses the heart after acetylcholine because the effect of acetylcholine on the heart is to: A. Decrease the refractory period B. Increase calcium influx C. Increase the strength of myocardial contraction D. Decrease the heart rate

D. Decrease the heart rate

A patient wants to know what causes atherosclerosis. How should the nurse respond? In general, atherosclerosis is caused by: A. An increase in antithrombotic substances B. Congenital heart disease C. High serum cholesterol levels D. Endothelial injury and inflammation

D. Endothelial injury and inflammation

A nurse recalls the internal lining of the cardiovascular system is formed by what tissue? A. Tunica adventitia B. Connective C. Mesothelium D. Endothelium

D. Endothelium

A patient has diffuse pain, fatigue, and tender joints without inflammation. This is a description of which diagnosis? A. Disuse atrophy. B. Contracture. C. Stress-induced muscle tension. D. Fibromyalgia.

D. Fibromyalgia.

A patient comes into the ER complaining of trouble breathing and enlargement of the neck which began occurring a month after she stopped eating iodized salt due to an allergy. Her symptoms describe what pathophysiology? A. Hyperplasia of the jugular ligaments B. Cushing's syndrome C. Thyroid storm D. Goiter

D. Goiter

A 73-year-old female has increased pulmonary pressure resulting in right heart failure. The nurse should monitor for a possible complication because a potential cause for her heart to fail is: A. Pericarditis B. Acute pneumonia C. Hypertension D. Left heart failure

D. Left heart failure

Which patient would the nurse assess for paroxysmal nocturnal dyspnea (PND)? A patient with: A. Asthma B. Pulmonary fibrosis C. Hypotension D. Left ventricular failure

D. Left ventricular failure

A 19 Y/O male comes to the physician office complaining that the light is hurting his eyes, nausea and a stiffness in his neck. He states that he lives in a dorm with community bathroom and his symptoms began about a day ago. What do you suspect he has? A. Encephalitis B. Malaria C. Stroke D. Meningitis

D. Meningitis

A nurse is preparing to teach the staff about asthma. Which information should the nurse include? Airway obstruction contributing to increased airflow resistance and hypoventilation in asthma is caused by: A. Alveolar fibrosis and pulmonary edema B. Type II alveolar cell injury and decreased surfactant C. Collapse of the cartilaginous rings in the bronchi D. Mucus secretion, bronchoconstriction, and airway edema

D. Mucus secretion, bronchoconstriction, and airway edema

A nurse is preparing to teach the staff about asthma. Which information should the nurse include? Airway obstruction contributing to increased airflow resistance and hypoventilation in asthma is caused by: A. Type II alveolar cell injury and decreased surfactant B. Alveolar fibrosis and pulmonary edema C. Collapse of the cartilaginous rings in the bronchi D. Mucus secretion, bronchoconstriction, and airway edema

D. Mucus secretion, bronchoconstriction, and airway edema

Most cases of combined systolic and diastolic hypertension have no known cause and are documented on the chart as _____ hypertension. A. Acquired B. Secondary C. Congenital D. Primary

D. Primary

A 72-year-old female has a history of right heart failure caused by a right ventricular myocardial infarction. Which of the following symptoms are specifically related to her right heart failure? A. Dyspnea upon exertion B. Decreased urine output C. Hypertension D. Significant edema to both lower legs and feet

D. Significant edema to both lower legs and feet

A 54-year-old male is diagnosed with empyema. Upon receiving and reviewing the culture result, which organism does the nurse suspect is the most likely cause? A. Moraxella catarrhalis B. Klebsiella pneumonia C. Virus D. Staphylococcus aureus

D. Staphylococcus aureus

A patient asks what it means that he was just diagnosed with diabetes mellitus type II. What is the best explanation? A. Your body is not producing the ADH it needs B. Your alpha cells in the pancreas is producing insulin that is no longer effective C. You have an autoimmune disease what has destroyed your insulin secreting cells D. Your cells are becoming immune to insulin

D. Your cells are becoming immune to insulin

wound disruption

Dehiscence (increases risk of infection) commonly seen after abdominal surgery

Body Fluids (cont'd)

Electrolytes are measured in milliequivalents per liter (mEq/L) or milligrams per deciliter (mg/dl) Milliequivalent indicates the chemical-combining activity of an ion, which depends on the electrical charge, or valence (number of plus or minus signs) ØMonovalent—one charge (+) ØDivalent—2 charges (++) Body fluid compartments (intracellular, intravascular, interstitial) maintain electrochemical balance (number of negative charges equals positive charges)

Body Fluids

Electrolytes, which are electrically charged, make up 95% of solutes ØCations are positively charged and migrate toward the negative pole (e.g., Na+) ØAnions are negatively charged and migrate toward the positive pole (e.g., Cl-)

Types of Tissue

Epithelial tissue ØThere are several types of epithelial tissues but they all have one thing in common---they all act as a lining. •Blood vessels •Air sacs •Respiratory tract •Bladder and other hollow structures •Gastrointestinal tract •Ducts •Skin

A deficiency of which substances have been associated with osteoporosis? Select all that apply. Iron. Estrogen. Calcium. Vitamin D. Magnesium.

Estrogen. Calcium. Vitamin D.

ANS: B Exogenous pyrogens are endotoxins produced by pathogens. They are not interleukins, prostaglandins, or corticotropin-releasing factors. REF: p. 343

Exogenous pyrogens are: a. interleukins. b. endotoxins. c. prostaglandins. d. corticotropin-releasing factors.

Tissue Formation

Extracellular matrix ØHolds the cells together and creates a latticework where cells can interact together. Pattern formation ØThe creation of tissue and organs from cells that are similar in function and structure.

ANS: A Moisture must be present because heat is lost through evaporation from the surface of skin and lining of mucous membranes, a major source of heat reduction connected with increased sweating in warmer surroundings. Fever is not required for evaporation to occur, but moisture is. Pyrogens are heat producers and do not assist with evaporation, but moisture is required. Trauma is not a portion of the evaporative process of heat loss. REF: p. 343, Table 14-5

For evaporation to function effectively as a means of dissipating excess body heat, which one of the following conditions must be present? a. Moisture b. Fever c. Pyrogens d. Trauma

ANS: A Brain death occurs when there is cessation of function of the entire brain, including the brainstem and cerebellum. Lack of cortical function or brainstem death is not enough to define brain death. A VS is complete unawareness of the self or surrounding environment and complete loss of cognitive function. REF: p. 364

For legal purposes, brain death is defined as: a. cessation of entire brain function. b. lack of cortical function. c. a consistent vegetative state (VS). d. death of the brainstem.

ANS: A Pernicious anemia can develop because the damage to the mucosa makes the intrinsic factor less available to facilitate vitamin B12 absorption in the ileum. None of the other options would result from this damage. REF: p. 914

Gastroscopy reveals degeneration of the gastric mucosa in the body and fundus of the stomach. This condition increases the risk for the development of: a. pernicious anemia. b. osmotic diarrhea. c. increased acid secretion. d. decreased gastrin secretion

The most common source of drug-induced thrombocytopenia is

Heparin

ANS: B Heat exhaustion results from prolonged high core or environmental temperatures, which cause profound vasodilation and profuse sweating, leading to dehydration, decreased plasma volumes, hypotension, decreased cardiac output, and tachycardia. Symptoms include weakness, dizziness, confusion, nausea, and fainting. Heat cramping is severe, spasmodic cramps in the abdomen and extremities that follow prolonged sweating and associated sodium loss. Heat cramping usually occurs in those not accustomed to heat or those performing strenuous work in very warm climates. Heat stroke is a potentially lethal result of an overstressed thermoregulatory center. With very high core temperatures (>40° C; 104° F), the regulatory center ceases to function, and the body's heat loss mechanisms fail. Malignant hyperthermia is a potentially lethal complication of a rare inherited muscle disorder that may be triggered by inhaled anesthetics and depolarizing muscle relaxants. REF: p. 344

Hikers are attempting to cross the Arizona desert with a small supply of water. The temperatures cause them to sweat profusely and become dehydrated. The hikers are experiencing: a. heat cramping. b. heat exhaustion. c. heat stroke. d. malignant hyperthermia.

ANS: C Individuals with extradural hematomas lose consciousness at injury; one third then become lucid for a few minutes to a few days. Mild concussion is characterized by immediate but transitory confusion that lasts for one to several minutes, possibly with amnesia for events preceding the trauma. Subdural hematomas begin with headache, drowsiness, restlessness or agitation, slowed cognition, and confusion. These symptoms worsen over time and progress to loss of consciousness, respiratory pattern changes, and pupillary dilation. Individuals with mild diffuse axonal injury display decerebrate or decorticate posturing and may experience prolonged periods of stupor or restlessness. REF: p. 392

Immediately after being struck by a motor vehicle, a patient is unconscious, but the patient regains consciousness before arriving at the hospital and appears alert and oriented. The next morning the patient is confused and demonstrates impaired responsiveness. The patient's history and symptoms support which medical diagnosis? a. Mild concussion b. Subdural hematoma c. Extradural (epidural) hematoma d. Mild diffuse axonal injury

ANS: A Alcoholic cirrhosis is caused by the toxic effects of alcohol metabolism on the liver. Alcohol is transformed to acetaldehyde, and excessive amounts significantly alter hepatocyte function and activate hepatic stellate cells, a primary cell involved in liver fibrosis. Bile toxicity does not cause alcoholic cirrhosis. Acidosis does not cause alcoholic cirrhosis. Fatty infiltrations do not cause alcoholic cirrhosis. REF: p. 932

In alcoholic cirrhosis, hepatocellular damage is caused by: a. acetaldehyde accumulation. b. bile toxicity. c. acidosis. d. fatty infiltrations.

ANS: B Intravascular clotting leads to depletion of clotting factors. DIC is due to depletion of clotting factors, not activation of prothrombin. Activation of clotting associated with DIC leads to inflammation, but inflammation does not cause DIC. TF is activated during DIC. REF: pp. 545-546

In disseminated intravascular coagulation (DIC), the nurse assesses for active bleeding after intravascular clotting because: a. prothrombin is activated. b. clotting factors are depleted. c. inflammatory mediators are released. d. tissue factor (TF) is inactivated.

ANS: C Hepatic encephalopathy effect on the liver prevents end products of intestinal protein digestion, particularly ammonia, from being converted to urea by the diseased liver. Impaired ammonia metabolism leads to the symptoms of hepatic encephalopathy. Symptoms are primarily neurologic, not jaundice oriented. Manifestations associated with hepatic encephalopathy are not associated with hyperbilirubinemia and jaundice, fluid, and electrolyte imbalances or decreased cerebral blood flow. REF: p. 929

Manifestations associated with hepatic encephalopathy from chronic liver disease are the result of: a. hyperbilirubinemia and jaundice. b. fluid and electrolyte imbalances. c. impaired ammonia metabolism. d. decreased cerebral blood flow.

ANS: B Most causes of encephalitis are viral, not bacterial, fungal, or toxoid. REF: p. 409

Most causes of encephalitis are which of the following? a. Bacterial b. Viral c. Fungal d. Toxoid

Membrane Transport (Cont.)

Osmolality ØMeasures the number of milliosmoles per kilogram (mOsm/kg) of water •Concentration of molecules per weight of water Osmolarity ØMeasures the number of milliosmoles per liter of solution •Concentration of molecules per volume of solution Tonicity

Membrane Transport (Cont.)

Osmosis ØMovement of water down a concentration gradient •Membrane must be more permeable to water than solutes •Concentration of solutes on one side greater than the other ØControls the distribution of water between body compartments ØOsmotic pressure ØRelated to hydrostatic pressure and solute concentration ØOncotic pressure or colloid osmotic pressure

Cellular Energy (cont'd)

Oxidative phosphorylation ØOccurs in the mitochondria ØMechanism producing energy (ATP) from fats, CHO, proteins

Cellular Energy (cont'd)

Oxidative phosphorylation (cont'd) ØAnaerobic glycolysis: if oxygen not available, CHO is converted to pyruvic acid (pyruvate) in cytoplasm with production of two ATP molecules which is insufficient for energy needs; pyruvate then converted to lactic acid ØProcess reverses when oxygen becomes available and lactic acid is converted back to either pyruvic acid or glucose, which moves into the mitochondria and enters the citric acid cycle

ANS: C Pancreatic insufficiency is the deficient production of lipase by the pancreas. Pancreatic insufficiency is not associated with the deficient production of insulin, amylase, or bile. REF: p. 919

Pancreatic insufficiency is manifested by deficient production of: a. insulin. b. amylase. c. lipase. d. bile.

Membrane Transport (Cont.)

Passive transport ØMolecules move easily from region of high concentration to region of low concentration ØRequires no energy ØDriven by osmosis, hydrostatic pressure, and diffusion Active transport ØFlows "uphill" ØRequires energy ØPumps ØEndocytosis ØExocytosis

Renal buffering

Secretion of H+ in the urine and reabsorption of HCO3-

ANS: C The qualities we normally feel from the body, including pain, also can be felt in the absence of inputs from the body, such as is noted with phantom limb pain. Neuropathic pain is initiated or caused by a primary lesion or dysfunction in the nervous system. Visceral pain refers to pain in internal organs and the lining of body cavities. Chronic pain lasts more than 3-6 months and is not associated with loss of a limb. REF: p. 341, Table 14-3

Several years after an amputation the patient continues to sporadically feel pain in the absent hand. What type of pain should the nurse document in the chart? a. Neuropathic pain b. Visceral pain c. Phantom limb pain d. Chronic pain

ANS: C The patient is experiencing autonomic hyperreflexia, which is manifested by paroxysmal hypertension (up to 300 mm Hg, systolic), a pounding headache, blurred vision, sweating above the level of the lesion with flushing of the skin, nasal congestion, nausea, piloerection caused by pilomotor spasm, and bradycardia (30-40 beats/min). The patient in extreme spinal shock experiences paralysis and flaccidity in muscles, absence of sensation, loss of bladder and rectal control, transient drop in blood pressure, and poor venous circulation. The patient may experience acute anxiety, but the symptoms of elevated blood pressure with severe headache are due to autonomic hyperreflexia. It is autonomic hyperreflexia, not parasympathetic areflexia, that produces paroxysmal hypertension (up to 300 mm Hg, systolic), a pounding headache, blurred vision, sweating above the level of the lesion with flushing of the skin, nasal congestion, nausea, piloerection caused by pilomotor spasm, and bradycardia (30-40 beats/min). REF: pp. 398-399

Six weeks ago a patient suffered a T6 spinal cord injury. What complication does the nurse suspect when the patient develops a blood pressure of 200/120, a severe headache, blurred vision, and bradycardia? a. Extreme spinal shock b. Acute anxiety c. Autonomic hyperreflexia d. Parasympathetic areflexia

ANS: A Cheyne-Stokes respirations occur as a result of CO2 levels in the blood. Ataxic breathing occurs as a result of dysfunction of the medullary neurons. Central neurogenic patterns occur as a result of uncal herniation. Normal respirations are based on the levels of oxygen (O2) in the blood. REF: p. 361

The breathing pattern that reflects respirations based primarily on carbon dioxide (CO2) levels in the blood is: a. Cheyne-Stokes. b. ataxic. c. central neurogenic. d. normal.

ANS: B The icteric phase is manifested by jaundice, dark urine, and clay-colored stools. The liver is enlarged, smooth, and tender, and percussion causes pain. Fatigue and vomiting occur during the prodromal stage. Resolution occurs in the recovery phase. Fulminant liver failure does not involve icterus. REF: p. 935

The icteric phase of hepatitis is characterized by which clinical manifestations? a. Fatigue, malaise, vomiting b. Jaundice, dark urine, enlarged liver c. Resolution of jaundice, liver function returns to normal d. Fulminant liver failure, hepatorenal syndrome

ANS: D The most common disks affected by herniation are the lumbosacral disks—that is, L5-S1 and L4-L5. REF: p. 401

The majority of intervertebral disk herniations occur between which vertebral levels (cervical, C; thoracic, T; lumbar, L; sacral, S)? a. C1-C3 b. T1-T4 c. T12-L3 d. L4-S1

ANS: C Vomiting of blood from bleeding esophageal varices is the most common clinical manifestation of portal hypertension. Neither rectal, duodenal, nor intestinal bleeding is a common clinical manifestation of portal hypertension. REF: p. 927

The most common clinical manifestation of portal hypertension is _____ bleeding. a. rectal b. duodenal c. esophageal d. intestinal

ANS: C Esophageal varices is the most common disorder associated with upper GI bleeding. Diverticulosis could lead to bleeding, but it would be lower GI rather than upper. Hemorrhoids can lead to bleeding, but they would be lower GI. Cancer could lead to upper GI bleeding, but peptic ulcers and varices are identified as more common. REF: p. 909

The most common disorder associated with upper GI bleeding is: a. diverticulosis. b. hemorrhoids. c. esophageal varices. d. cancer.

ANS: C Presbycusis is the most common form of sensorineural hearing loss in elderly people. Conductive hearing loss does not occur as frequently as presbycusis. Otitis media is an infection in the middle ear and is not defined as a hearing loss. Ménière disease leads to vertigo, not hearing loss. REF: p. 352

The most common form of sensorineural hearing loss in the elderly is: a. conductive hearing loss. b. acute otitis media. c. presbycusis. d. Ménière disease.

ANS: A Large intestine obstruction usually presents with hypogastric pain and abdominal distention. Neither diarrhea, epigastric pain, nor rectal bleeding occurs. REF: pp. 913-914

The symptoms and signs of large bowel obstruction are: a. abdominal distention and hypogastric pain. b. diarrhea and excessive thirst. c. dehydration and epigastric pain. d. abdominal pain and rectal bleeding.

ANS: C Iron stores are measured directly by bone marrow biopsy, not serum ferritin, not transferrin saturation, and not total iron-binding capacity. REF: p. 518

The nurse will check which of the following tests to directly measure iron stores? a. Serum ferritin b. Transferrin saturation c. Bone marrow biopsy d. Total iron-binding capacity

ANS: C Alterations in proprioception are manifested by a sensation that the room is spinning, not headache, light sensitivity, or loss of feeling in the lips. REF: p. 353

The nurse would expect the patient with an alteration in proprioception to experience vertigo, which is manifested by: a. headache. b. light sensitivity. c. a sensation that the room is spinning. d. loss of feeling in the lips.

ANS: A Brain tissue hypoxia occurs as a result of increased intracranial pressure as it places pressure on the brain. Increased intracranial pressure leads to intracranial hypertension. Ventricular swelling may lead to increased intracranial pressure, but increased pressure does not lead to either ventricular swelling or the expansion of the cranial vault. REF: p. 374

The patient is experiencing an increase in intracranial pressure. This increase results in: a. brain tissue hypoxia. b. intracranial hypotension. c. ventricular swelling. d. expansion of the cranial vault.

ANS: B The diagnosis of myasthenia gravis is made on the basis of a response to edrophonium chloride (Tensilon). This medication is not associated with the diagnosis of ALS, MS, or autonomic hyperreflexia. REF: p. 413

The patient reports generalized muscle weakness. The health care provider orders administration of the medication edrophonium chloride (Tensilon). This medication is used in the diagnosis of: a. amyotrophic lateral sclerosis (ALS). b. myasthenia gravis. c. multiple sclerosis (MS). d. autonomic hyperreflexia.

ANS: B Japan has the lowest rate of Hodgkin lymphoma. The United States, the Netherlands, and Denmark have among the highest rates. REF: p. 533

The people from which country have the lowest risk for Hodgkin lymphoma? a. United States b. Japan c. Denmark d. Netherlands

ANS: B Vasogenic edema is clinically important because the blood-brain barrier (selective permeability of brain capillaries) is disrupted, and plasma proteins leak into the extracellular spaces. Vasogenic edema does not have an infectious cause. ICP is increased, but not more so than other forms of edema. Vasogenic edema does not always cause herniation. REF: p. 375

The primary care provider states that the patient is experiencing vasogenic edema. The nurse realizes vasogenic edema is clinically important because: a. it usually has an infectious cause. b. the blood-brain barrier is disrupted. c. ICP is excessively high. d. it always causes herniation.

ANS: B, C Infection with H. pylori and chronic use of NSAIDs are the major causes of duodenal ulcers. Consuming spicy foods, trauma, and antibiotics do not lead to duodenal ulcer disease. REF: p. 916

The primary causes of duodenal ulcers include: (select all that apply) a. consumption of spicy foods. b. nonsteroidal anti-inflammatory drugs (NSAIDs). c. H. pylori infection. d. trauma. e. side effects of antibiotics.

ANS: D In infratentorial herniation, the cerebellar tonsil shifts through the foramen magnum because of increased pressure within the posterior fossa. Supratentorial herniation involves temporal lobe and hippocampal gyrus shifting from the middle fossa to posterior fossa. Central herniation is a type of supratentorial herniation and is the straight downward shift of the diencephalon through the tentorial notch. Gyrus herniation occurs when the cingulate gyrus shifts under the falx cerebri. Little is known about its clinical manifestations. REF: p. 375, Box 15-5

The progress notes read: the cerebellar tonsil has shifted through the foramen magnum due to increased pressure within the posterior fossa. The nurse would identify this note as a description of _____ herniation. a. supratentorial b. central c. cingulated gyrus d. infratentorial

ANS: B Ischemia and necrosis (intestinal infarction) alter membrane permeability. Initially, there is increased motility, nausea, and vomiting. Mucosal alteration causes fluid to move from the blood vessels into the bowel wall and peritoneum. Fluid loss causes hypovolemia. Arterial insufficiency is not related to liver failure. Bleeding may occur, but hypovolemia is related to fluid shifts. Fluid shifts lead to hypovolemia; it is not due to overstimulation of the parasympathetic nerves. REF: p. 923

The risk of hypovolemic shock is high with acute mesenteric arterial insufficiency because: a. the resulting liver failure causes a deficit of plasma proteins and a loss of oncotic pressure. b. ischemia alters mucosal membrane permeability, and fluid loss occurs. c. massive bleeding occurs in the GI tract. d. overstimulation of the parasympathetic nervous system results in ischemic injury to the intestinal wall.

ANS: C In pancreatitis there is backup of pancreatic secretions and activation and release of enzymes (activated trypsin activates chymotrypsin, lipase, and elastase) within the pancreatic acinar cells. The enzymes become activated, triggering the resulting autodigestion, inflammation, and oxidative stress. The tissue damage associated with pancreatitis is not due to insulin toxicity or to hydrochloric acid reflux. REF: p. 936

Tissue damage in pancreatitis is initially triggered by: a. insulin toxicity. b. autoimmune destruction of the pancreas. c. backup of pancreatic enzymes. d. hydrochloric acid reflux into the pancreatic duct.

A neurologist is teaching about encephalitis. Which information should the neurologist include? Most cases of encephalitis are?

VIRAL!!!!!

ANS: B An explosive, disorderly discharge of cortical neurons is a seizure. A reflex is an expected response. Inattentiveness is a form of neglect. Brain death is a cessation of function. REF: p. 372

What term is used to describe an explosive, disorderly discharge of cortical neurons? a. Reflex b. Seizure c. Inattentiveness d. Brain death

ANS: C Multiple sclerosis is due to an interaction between the autoimmune response and genetics. Multiple sclerosis is an autoimmune disorder that is thought to have developed secondary to a viral infection. It is not bacterial, nor is it related to a neurotransmitter dysfunction or vascular or metabolic factors. REF: p. 411

When a patient asks, "What is the cause of multiple sclerosis?" the nurse bases the answer on the interaction between: a. vascular and metabolic factors. b. bacterial infection and the inflammatory response. c. autoimmunity and genetic susceptibility. d. neurotransmitters and inherited genes.

ANS: D Fever (febrile response) is a temporary "resetting of the hypothalamic thermostat" to a higher level in response to endogenous or exogenous pyrogens. Fever is the result of the body's attempt to raise temperature, not adjust it to a lower level. When fever occurs, the temperature is raised, but the rise is due to a reset of the thermostat. Bacteria do not stimulate peripheral thermogenesis, but their endotoxins do. REF: pp. 342-343

When a patient has a fever, which of the following thermoregulatory mechanisms is activated? a. The body's thermostat is adjusted to a lower temperature. b. Temperature is raised above the set point. c. Bacteria directly stimulate peripheral thermogenesis. d. The body's thermostat is reset to a higher level.

ANS: B Iron overload can be primary, as in hereditary hemochromatosis. Hemochromatosis is not associated with coagulation, leukocytosis, or granulocytosis. REF: p. 522

When the nurse sees a diagnosis of hereditary hemochromatosis on the chart, the nurse knows this is a disorder of: a. intravascular coagulation. b. iron overload. c. leukocytosis. d. granulocytosis.

ANS: D Temperature fluctuation is related to circadian rhythm, not the thermogenesis cycle, thermoconductive phases, or adaptive patterns. REF: p. 342

When the nurse is discussing the patient's cyclical temperature fluctuation occurring on a daily basis, what term should the nurse use? a. Thermogenesis cycle b. Thermoconductive phases c. Adaptive pattern d. Circadian rhythm

ANS: C Temperature regulation (thermoregulation) is mediated primarily by the hypothalamus, not the cerebrum, the brainstem, or the pituitary gland. REF: p. 342

When the nurse is taking a patient's temperature, which principle should the nurse remember? Regulation of body temperature primarily occurs in the: a. cerebrum. b. brainstem. c. hypothalamus. d. pituitary gland.

ANS: B Locked-in syndrome occurs when the individual cannot communicate through speech or body movement but is fully conscious, with intact cognitive function. In cerebral death, the person is in a coma with eyes closed. Dysphagia is difficulty speaking. Cerebellar motor syndrome is characterized by problems with coordinated movement. REF: p. 365

When thought content and arousal level are intact but a patient cannot communicate and is immobile, the patient is experiencing: a. cerebral death. b. locked-in syndrome. c. dysphagia. d. cerebellar motor syndrome.

ANS: C Mild concussion is characterized by immediate but transitory confusion that lasts for one to several minutes, possibly with amnesia for events preceding the trauma. Individuals with extradural hematomas lose consciousness at injury; one third then become lucid for a few minutes to a few days. Persons with diffuse brain injury demonstrate behavioral changes. Individuals with a mild concussion experience transient, not permanent, confusion. REF: p. 394

Which assessment finding by the nurse characterizes a mild concussion? a. A brief loss of consciousness b. Significant behavioral changes c. Retrograde amnesia d. Permanent confusion

ANS: C One of the complications is anemia due to iron malabsorption, which may result from decreased acid secretion. Diarrhea, not constipation, occurs. The reflux would be alkaline, not acidic. Hiccups are not associated with gastrectomy. REF: p. 919

Which complication is associated with gastric resection surgery? a. Constipation b. Acid reflux gastritis c. Anemia d. Hiccups

ANS: C Therapeutic hypothermia is seen with reimplantation surgery, not malnutrition, hypothyroidism, or Parkinson disease. REF: p. 345, Box 14-3

Which condition would be treated with therapeutic hypothermia? a. Malnutrition b. Hypothyroidism c. Reimplantation surgery d. Parkinson disease

ANS: B Symptoms of heat stroke include high core temperature, absence of sweating, rapid pulse, confusion, agitation, and coma, and complications include cerebral edema and degeneration of the CNS. Neither cramping nor alterations in calcium uptake are considered complications of a heat stroke. REF: p. 344

Which finding indicates the patient is having complications from heat stroke? a. Mild elevation of core body temperatures b. Cerebral edema and degeneration of the CNS c. Spasmodic cramping in the abdomen and extremities d. Alterations in calcium uptake

ANS: C Genetic changes may occur by both mutational and epigenetic mechanisms. Mutation generally means an alteration in the DNA sequence affecting expression or function of a gene. Mutations include small-scale changes in DNA, such as point mutations, which are the alteration of one or a few nucleotide base pairs. The process involved with insertions, deletions, or amplification mutations is different. REF: p. 237

Which genetic change causes alterations in only one or a few nucleotide bases? a. Insertions b. Deletions c. Point mutations d. Amplification mutations

ANS: D Cancers can secrete multiple factors that stimulate new blood vessel growth called angiogenesis, not release of growth factors or tumor regression. Apoptosis is cell death. REF: pp. 245-246

Which information should the nurse include when teaching about angiogenic factors? In cancer, angiogenic factors stimulate: a. release of growth factors. b. tumor regression. c. apoptosis. d. new blood vessel growth.

ANS: A Occlusion of blood flow is often precipitated by embolism. This type of occlusion is not associated with pancreatitis, chronic malnutrition, or hypovolemic shock. REF: pp. 923-924

Which of the following characteristics is associated with an acute occlusion of mesenteric blood flow to the small intestine? a. Often precipitated by an embolism b. Commonly associated with disease such as pancreatitis and gallstones c. Caused by chronic malnutrition and mucosal atrophy d. Often a complication of hypovolemic shock

ANS: C The cognitive-evaluative system overlies the individual's learned behavior concerning the experience of pain and can modulate perception of pain. The sensory-discriminative system is mediated by the somatosensory cortex and is responsible for identifying the presence, character, location, and intensity of pain. The affective-motivational system determines an individual's conditioned avoidance behaviors and emotional responses to pain. The reticular-activating system does not play a role in the perception of pain. REF: p. 338

Which system modulates a patient's perception of pain? a. Sensory-discriminative system b. Affective-motivational system c. Cognitive-evaluative system d. Reticular-activating system

ANS: A, B Infants cannot conserve heat; thus, they have a problem with thermoregulation because of their small size. Infants are unable to shiver, do have the ability to sweat, have an increased metabolic rate, and have little subcutaneous fat. REF: p. 342

While planning care for infants, which principles should the nurse remember? (select all that apply) Infants have problems with thermoregulation because they: a. cannot conserve heat. b. do not shiver. c. rarely sweat. d. have decreased metabolic rates. e. have excess subcutaneous fat.

ANS: D Elderly people are particularly at risk from minor trauma that results in serious spinal cord injury because of preexisting degenerative vertebral disorders. Neither females nor infants are at any particular risk for spinal cord injuries. Males are at great risk for spinal cord injury but not as a result of preexisting disorders. REF: p. 395

Who is most at risk of spinal cord injury because of preexisting degenerative disorders? a. Infants b. Men c. Women d. The elderly

A 25-year-old female was diagnosed with cervical cancer. History reveals she had many sexual partners, which she indicates is "too many to count." Which of the following is most likely to have caused her cancer? a. HPV-16 b. HPV-18 c. HPV-31 d. HPV-45

a. HPV-16

Which of the following diseases is cause by a deficiency of cortical hormones? a. Addison's disease b. Cushing's syndrome c. Diabetes mellitus d. Hypothyrodisim

a. Addison's disease

A nurse is evaluating the direct end effect of the renin-angiotensin-aldosterone system. Which principle should the nurse remember? a. Angiotensin II causes systemic vasoconstriction. b. Renin promotes the excretion of sodium and water in the renal tubules. c. Aldosterone increases renal retention of water only. d. Angiotensin I promotes sodium and water reabsorption by the kidneys.

a. Angiotensin II causes systemic vasoconstriction.

While planning care for a patient with acute inflammation and pain, which principle should the nurse remember? The inflammatory chemicals responsible for inducing pain during inflammation are: a. bradykinin and prostaglandin b. histamine and chemotactic cytokines c. lymphokines d. nitrous oxide and platelet activating factor

a. Bradykinin and prostaglandins Bradykinin produced through the kinin system and prostaglandins released by the mast cell induce pain during inflammation.

A 45-year-old male is diagnosed with macrocytic, normochromic anemia. The nurse suspects the most likely cause of this condition is: a. Defective DNA synthesis b. Abnormal synthesis of hemoglobin c. Defective use of vitamin C d. Blocked protein synthesis

a. Defective DNA synthesis

A staff member wants to know where the greatest proportion of iron is located. How should the nurse respond? The greatest proportion of total body iron is located in the: a. Erythrocytes b. Spleen pulp c. Bone marrow d. Liver tissue

a. Erythrocytes

A 45-year-old female undergoes a splenectomy to remove a tumor. Which of the following assessment finding is most likely to occur following surgery? a. Leukocytosis b. Hypoglycemia c. Decreased red blood cell count d. Decreased platelets

a. Leukocytosis

During an infection, the nurse assesses the lymph nodes. Lymph nodes enlarge and become tender because: a. Lymphocytes are rapidly dividing. b. Edema accumulates within the fibrous capsule. c. Microorganisms are accumulating. d. The nodes are not functioning properly.

a. Lymphocytes are rapidly dividing.

A 65-year-old male was recently diagnosed with cancer. He is retired from construction work. Which of the following cancers is he likely to develop secondary to occupational hazards? a. Mesothelioma b. Bladder cancer c. Prostate cancer d. Bone cancer

a. Mesothelioma

A patient has a tissue growth that was diagnosed as cancer. Which of the following terms best describes this growth? a. Neoplasm b. Lipoma c. Meningioma d. Hypertrophy

a. Neoplasm

An adult patient's blood sample is analyzed in a laboratory. Assuming a normal sample, which type of white blood cell accounts for the highest percentage? a. Neutrophils b. Eosinophils c. Basophils d. Lymphocytes

a. Neutrophils

A 42-year-old female presents with abdominal discomfort, epigastric tenderness, and bleeding. Gastroscopy reveals degeneration of the gastric mucosa in the body and fundus of the stomach. Which of the following would most likely follow? a. Pernicious anemia b. Osmotic diarrhea c. Increased acid secretion d. Decreased gastrin secretion

a. Pernicious anemia

A 57-year-old male presents to his primary care provider for red face, hands, feet, ears, and headache and drowsiness. A blood smear reveals an increased number of erythrocytes, indicating: a. Polycythemia vera (PV) b. Leukemia c. Sideroblastic anemia d. Hemosiderosis

a. Polycythemia vera (PV)

A 25-year-old male nursing student recently learned how diet can alter the chances of developing cancer. He tries to minimize his risks of developing the disease by ordering his steak: a. Rare b. Medium c. Medium-well d. Well done

a. Rare

When a patient asks what types of cancers are associated with tobacco use, how should the nurse respond? a. Squamous and small cell adenocarcinomas b. Sarcoma and adenoma c. Melanoma and lymphoma d. Basal cell and lipoma

a. Squamous and small cell adenocarcinomas

A 60 year old female with a history of alcoholism complains of recent weight gain and right flank pain. physical examination reveals severe ascites. this condition is caused by decreased a. albumin and lack of cellular integrity b. capillary filtration pressure c. capillary permeability d. antidiuretic hormone secretion

a. albumin and lack of cellular integrity

Which of the following are complications of gastroesophageal reflux (select all that apply) a. aspiration pneumonia b. anemia c. weight loss d. delayed growth e. hepatomegaly

a. aspiration pneumonia b. anemia c. weight loss d. delayed growth

A 60 year old male with a 30 year history of smoking is diagnosed with a hormone-secreting lung tumor. Further testing indicates that the tumor secrets ADH. Which of the following assessment findings should the nurse expect (select all that apply) a. confusion b. weakness c. nausea d. muscle twitching e. weight loss

a. confusion b. weakness c. nausea d. muscle twitching

A 60 year old male is diagnosed with renal failure. while the nurse is reviewing lab results, which of the following lab values would be most consistent with this diagnosis a. elevated plasma creatinine level b. decreased plasma potassium level c. metabolic alkalosis d. increased urea clearance

a. elevated plasma creatinine level

Clinical manifestations of bile salt deficiencies are related to poor absorption of a. fats and fat-soluble vitamins b. water-soluble vitamins c. proteins d. minerals

a. fats and fat-soluble vitamins

a 35 year old female with Graves disease is admitted to a medical surgical unit. while the nurse is reviewing the lab tests, which results would the nurse expect to find a. high levels of circulating thyroid-stimulating antibodies b. ectopic secretion of thyroid stimulating hormone (TSH) c. low circulating levels of thyroid hormones d. increased circulation of iodine

a. high levels of circulating thyroid-stimulating antibodies

a 54 year old patient with pulmonary tuberculosis (lung infection) is evaluated for syndrome of inappropriate ADH secretion (SIADH). which of the following electrolyte imbalances would be expected in this patient a. hyponatremia b. hyperkalemia c. hypernatremia d. hypokalemia

a. hyponatremia

a patient has researched bradykinin on the internet. which information indicates the patient understands the functions of bradykinin. Bradykinin is involved in a. increasing vascular permeability b. vasoconstricting blood vessels c. stimulating the clotting system d. increasing degradation of prostaglandins

a. increasing vascular permeability

A 4 year old male is diagnosed with nephrotic syndrome. Which of the following assessment findings accompanies this condition a. proteinuria b. decreased blood urea nitrogen (BUN) c. hematuria d. sodium loss

a. proteinuria

A 60 year old male is diagnosed with cancer of the esophagus. which of the following factors most likely contributed to his disease a. reflux esophagitis b. intestinal parasites c. ingestion of salty foods d. frequent use of antacids

a. reflux esophagitis

The most common cause of pure water deficit is a. renal water loss b. hyperventilation c. sodium loss d. insufficient water intake

a. renal water loss

While planning care for a patient, which principle should the nurse use to guide care? The first line of defense against pathogens is the: a. skin and mucous membranes. b. inflammatory response. c. primary immune response. d. hypersensitivity response

a. skin and mucous membranes.

A 20-year-old male is in acute pain. An arterial blood gas reveals decreased carbon dioxide (CO2) levels. Which of the following does the nurse suspect is the most likely cause? a.Hyperventilation b.Hypoventilation c.Apnea d.Cyanosis

a.Hyperventilation

Pancreatic beta cells secrete insulin, which inhibits secretion of glucagon from neighboring alpha cells. This action is an example of which of the following signaling types? a.Paracrine b.Autocrine c.Neurohormonal d.Hormonal

a.Paracrine

Potassium adaptation

ability of the body to adapt to increased levels of potassium intake over time

ascites

abnormal accumulation of fluid in the abdomen

pleural effusion

abnormal accumulation of fluid in the pleural space

A 50-year-old male sustained a closed head injury as a result of a motor vehicle accident. CT scan revealed a collection of blood between the inner surface of the dura mater and the surface of the brain. Which type of injury will the nurse be caring for? a. subdural hematoma b. epidural hematoma c. contusion d. abrasion

ans: a a subdural hematoma occurs when blood is between the inner surface of the dura mater and the surface of the brain; it can result from blows, falls, or sudden acceleration/deceleration of the head. An epidural hematoma is a collection of blood between the inner surface of the skill and the dura; it is most often associated with a skull fracture. A contusion is bleeding into the skin or underlying tissues. An abrasion (scrape) results from removal of the superficial layers of the skin caused by friction between the skin and injuring object. REF: p. 94, table 4-8

A 52-year-old male suffered a myocardial infraction secindary to artherosclerosis and ischemia. Once blood flow is returned to the damaged heart, reperfusion injury occurs as a results of: a. oxidation stress b. vacuolation c. decreased intracellular calcium d. lipid acceptor proteins

ans: a reperfusion injury can result from oxidative stress, increased intracellular calcium, inflammation, or complement activation. Oxidative stress causes the formation of radicals that cause further membrane damage and mitochondrial calciu overload. vacuolation leads to cellular swelling but is not associated with reperfusion. An increase of intracellular calcium is a cause of reperfusion ijury. Lipid acceptor proteins bind with triglycerides to create fatty liver, and they do not affect the myocardium. REF: p. 81

A female student is driving to school when another driver nearly hits her. Her heart begins beating harder and faster as she becomes aroused and scared. Which of the following stages of the general adaptation syndrome is she experiencing? a. alarm stage b. stage of resistance c. adaptation d. exhaustion

ans: a the alarm stage, the initial reaction, is manifested by arousal of the body's defenses that prepare the body to fight or flee from threat. this stage involves the secretion of hormones and catecholamines to support physiologic/metabolic activity and boosts the immune system to thwart infection. the stage of resistance/adaptation is the second step as the body attempts to sustain the challenge. exhaustion marks the breakdown of compensatory mechanisms, when the response can no longer be sustained. REF: pp. 214-215

a 10-year-old female is arriving at a national spelling bee contest. her heart starts beating faster and harder, and she begins to sweat. which of the following is she experiencing? a. anticipatory response b. homeostasis c reactive response d. exhaustion stage

ans: a anticipation of experiencing these events produces a physiologic stress response. homeostasis is a steady-state. reactive response occurs following a stressful response, when the body can no longer sustain the response. REF: p. 215

a patient has been reading on the internet that light to moderate intake of alcohol is cardioprotective. when the patient asks the nurse what this means, the nurse should respond that the heart is protected by which of the following mechanisms? (select all that apply) a. increased levels of hihg-desity lipoprotein cholesterol b. prevention of clot formation c. reduction in platelet aggregation d. decrease in blood pressure e. increased collateral circulation f. decreased folate absorption

ans: a, b, c, d the suggested mechanisms for cardioprotection by low to moderate alcohol intake include increased levels of high-desity lipoprotein cholesterol (HDL-c), prevention of clot formation, reduction in platelet aggregation, and decrease in blood pressure. increased collateral circulation is not a benefit of alcohol. alcohol consumption can decrease folate absorption, which can lead to a nurtritional deficient. REF: p. 91

when a patient uses repression to deal with psychological stress, which of the following assessment findings should the nurse monitor for? (select all that apply) a. decreased monocyte counts b. increased eosinophil counts c. decreased serum glucose d. increased pulse rates e. increased medication reactions

ans: a, b, e repression is associated with lower monocyte counts, higher eosinophil counts, higher serum glucose, and more self-reported medication reactions in medical outpatients. it is not associated with increased pulse rates and glucose increases, rather than decreases. REF: p. 227

a 35-year-old female is diagnosed with multiple myeloma. Biopsy of the tumor reveals Russell bodies, and laboratory testing reveals kidney dysfunction. Which substance should the nurse monitor as it is accumulating in the patient's body? a. glycogen b. protein c. pigment d. melanin

ans: b Russell bodies occur due to excess aggregates of protein. excess glycogen would affect blood glucose. Increased pigment would not lead to kidney dysfucntion. melanin accumulates in the epithelial cells (keratinocytes) of the skin and retina. REF: p.99

a 20-year-old male presents to the emergency department with a jagged sharp-force injury that is longer thatn it is deep. Which type of wound will the nurse be caring for? a. stab wound b. incised wound c. puncture wound d. chopping wound

ans: b an incised wound is a cut that is longer than it is deep. A stab woundis penetratin sharp-force injury that is deeper than it is long. A puncture wound is without sharp edges and is made with an instrument like a nail. Heavy, edged instruments (axes, hatchets, propeller blades) produce wounds with a combination of sharp and blunt force characteristics. REF: p. 94, table 4-8

when planning care for the pregnant patient, the nurse will recall that the mammary glands enlarge as a consequence of: a. compensatory hyperplasia b. hormonal hyperplasia c. hormonal anaplasia d. compensatory anaplasia

ans: b an increase in the mammary glads during pregnancy is a result of hormonal changes. The number of mammary cells increases in response to increased hormone levels, not as a compensatory mechanism. Anaplasia is a reversal to less mature cells. REF: p. 76

A report comes indicating that muscular atrophy has occurred. A nurse recalls that muscular atrophy involves a decrease in muscle cell: a. number b. size c. vacuoles d. lipofuscin

ans: b atrophy is a decrease or shrinkage in cellular size. Hyperplasia is an increase in the number of cells. Vacuoles are membrane-bound vesicles within the cell that contain cellular debris and hydrolytic enzymes. Lipofuscin is the yellow-brown age pigment. REF: p. 74

stress-induced catecholamine release from the adrenal medulla may result in: a. decreased blood flow to the brain b. elevated blood pressure c. decreased glycogen synthesis d. decreased muscle contraction

ans: b catecholamine release results in elevated blood pressure as a result of vasoconstriction. catecholamine release results in increased blood flow to the brain and increased glucose production. catecholamine release results in increased muscle contraction. REF: p. 222, table 9-3

a 2-year-old swallowed watch batteries. following ingestion, kidney function was impaired, and the heart began to fail. which of the following was the most likely cause? a. karyorrhexis b. coagulative necrosis c. ammonia accumulation d. caseous necrosis

ans: b coagulative necrosis occurs primarily in the kidneys, heart, and adrenal glands and commonly results from hypoxia. Karyorrhexis means fragmentation of the nucleus into smaller particles or "nuclear dust." Ammonia accumulation is not associayed with this toxicity. caseous necrosis from results from tuberculosis pulmonary infection. REF: pp. 102-103

while reading a textbook, a student reads the term apoptosis. the student recalls that apoptosis is a condition in which cells program themselves to: a. atrophy b. die c. regenerate d. age

ans: b in apoptosis, cells are programmed to die. apoptosis is not associated with cell atrophy, regeneration, or aging. REF: p.104

when a patient is diagnosed with a hormone-secreting tumor of the adrenal cortex, which physiological respone would be expected? a. decreased blood pressure b. increased incidence of gastric ulcers c. increased lipogenesis of extremities d. decreased gastric secretion

ans: b increased release of cortisol leads to increased gastric secrretions, and therefoe an increased incidence of gastric ulcers. Hypertension is a result of increased cortisol. the increase of gastric secretions causes lipolysis, not lipgenesis. REF: p. 218-219

a family presents to their primary care provider reporting headache, nausea, weakness, tinnitus, and vomiting. which of the following would be the most likely explanation for these symptoms? a. lead exposure b. carbon monoxide poisoning c. ethanol exposure d. mercury poisoning

ans: b symptoms related to carbon monoxide poisoning include headache, giddiness, tinnitus (ringing in the ears), nausea, weakness, and vomiting. Although nausea and vomiting can occur with lead exposure, lead toxcity is primarily manifested by convulsions and delirium and, with peripheral nerve involvement, wrist, finger, and sometimes foot paralysis. Ethanol exposure has CNS effects and would not affect the whole family. Mercury poisoning is manifested by CNS effects and would not lead to nausea and vomiting. REF: p. 90

a nurse recalls that stress-induced stimulation of the adrenal cortex causes it to secrete: a. estrogen b. cortisol c. parathyroid hormone d. adrenocorticotropin hormone (ACTH)

ans: b the adreanl cortex secretes cortisol, not estrogen, parathyroid hormone, or ACTH. REF: p. 218

when a nurse is checking a urinalysis, the finding that would alert the nurse to cellular injury is the presence of: a. slight glucose b. excessive protein c. blood d. urea

ans: b the presence of protein in the urine in significant amts indicates cellular injury and altered cellular function. neither glucose nor blood is normally present in the urine, but its presence is not indictive of altered cellular function. urea is an expected substance in the kidney REF: p. 99

A patient has a severe kidney obstruction leading to removal of the affected kidney. which of the following would the nurse expect to occur? a. atrophy of the remaining kidney b. compensatory hypertorphy of the remaining kidney c. dysplasia in the remaining kidney d. renal failure

ans: b the remaining kidney would hypertrophy to compensate for the increased workload of the loss of the affected kidney. compensation for such a situation would not include atrophy or a change in cell structure of the remaining kidney. rena failure would be avoided. ref: p. 748

When teaching about the stress-age syndrome, what information should the nurse include? (select all that apply) a. decreased cortisol secretion b. decreased thyroxine c. Immunodepression d. increased catecholamine secretion e. hypercoagulation of the blood f. free-redical damage

ans: b, c, d, e, f the stress-age syndrome includes alteration in the excitability of structures of the limbic system and hypothalamus; rise of the blood concentration of catecholamines, ADH, ACTH, and cortisol; decrease in testosterone, thyroxine, and others; alteration of opioid peptides; immunodepression and patter of chronic inflammation; alterations in lipoproteins; hypercoagulation of the blood; and free-radical damge of cells. REF: p. 228, gariatric considerations

a group of prison inmates developed tuberculosis following exposure to an infected inmate. on examination, tissues were soft and granular (like clumoed cheese). which of the following is the most likely cause? a. coagulative necrosis b. liquefactive necrosis c. caseous necrosis d. autonecrosis

ans: c caseous necrosis results from tuberculosis pulmonary infection. coagulative necrosis occurs primarily in the kidneys, heart, and adrenal glands, and commonly results from hypoxia. Liquefactive necrosis results from ischemic injury to neurons and glial cells in the brain. autonecrosis is a process of cellular self-digestion and is not due to infection such as tuberculosis. REF: p. 103

A 55-year-old male is diagnosed with hepatocellular cancer secondary to hepatitis C. If the cancerous region of the liver is removed, the remaining cells would undergo: a. pathologic hyperplasia b. pathologic metaplasia c. compensatory hyperplasia d. compensatory aplaisa

ans: c compensatory hyperplasia is an adaptive, not pathologic, mechanism that enables certain organs to regenrate. Metaplasia is the reversible replacement of one mature cell type by another, sometimes less differentiated, cell type. Aplasia is not a compensatory mechanism. REF: p. 76

A nurse recalls that direct stimulation of the insulin-secreting cells of the pancreas by the autonomic nervous system is an example of which type of control? a. Negative feedback b. Positive feedback c. Neural d. Substrate-level dependent

ans: c direct stimulation of the insulin-secreting cells of the pancreas by the autonomic nervous system is a form of neural control. Direct stimulation of insulin-secreting cells is ot controlled by a feedback mechanism or by substrate-level dependency. ref: p. 439

The early dilation (swelling) of the cell's endoplasmic reticulum results in: a. increased aerobic metabolism b. failure of DNA c. reduced protein synthesis d. increased Na+ -K+ pump function

ans: c early dilation of the endoplasmic reticulum causes the ribosomes to detach from the rough endoplasmic reticulum, reducing protein synthesis. Aerobic metabolism is a normal process and would not lead to swelling. Cellular swelling will not alter cellular DNA. A reduction in the Na+ -K+ pump leads to an intracellular accumulation of sodium and calcium and diffusion of potassium out of the cell. Sodium and water can then enter the cell freely, and cellular swelling results. REF: p. 80

when assessing the effects of elevated b-endorphins in a patient, which of the following should the nurse monitor? a. peripheral vasoconstriction b. hyperglycemia c. pain inhibition d. decreased immune cell activity

ans: c elevated b-endorphins activate endorphin (opiate) receptors on peripheral sensory nerves, leading to pain relief or analgesia. none of the other options result from the effects of elevated b-endorphins. REF: P. 224, TABLE 9-4

which of the following hormones enhances myocardial contractility? a. oxytocin b. prolactin c. epinephrine d. somatotropin

ans: c epinephrine enhances myocardial contractility. oxytocin activates receptors in the uterus. prolactin does not activate adreenergic recpetors. somatotropin activates protein and carbohydrate metabolism REF: p. 221

Exhaustion occurs if stress continues and _____ is not successful a. flight or fight response b. alarm c. adaptation d. arousal

ans: c exhaustion occurs if adaptation is not successful. the alarm stage is the emergency reaction that prepares the body to fight or flee from threat. arousal occurs as the stress is recognized. REF: p. 215

A patient is diagnosed with urinary tract obstruction. While planning care, the nurse realizes that the patient is expected to have hydronephrosis and a decreased glomerular filtration rate caused by: a. decreased renal blood flow b. decreased peritubular capillary pressure c. dilation of the renal pelvis and calyces proximal to a blockage d. stimulation of antidiuretic hormone

ans: c hydronephrosis occurs due to dilaton of the renal pelvis and calyces proximal to a blockage. Hydronephrosis is not the result of a decrease in renal blood flow, or peritubular capillary pressure, or stimulation of the antidiuretic hormone ref: p. 747

liquefactive necrosis occurs in the brain because: a. debris is not digested by hydrolases b. of protein denaturation c. it is rich in hydrolytic enzymes and lipids d. ischemia results in chemical injury

ans: c liquefactive necrosis is due to enzymatic action and because cells of th ebrain are rich in enzymes. protein denaturation occurs primarily in the kidneys. liquefactive necrosis is due to enzymatic reaction and not to hypoxia or hydrolases. REF: p. 103

A 55-year-old male with a 30-year history of smoking is examined for respiratory disturbance. Examination of his airway (bronchial) reveals that stratified squamous epithelial cells have replaced the normal columnar ciliated cells. This type of cellular adaptation is called: a. anaplasia b. hyperplasia c. metaplasia d. dysplasia

ans: c metaplasia is the reversible replacement of one mature cell type by another, sometimes a less differentiated cell type. Anaplasia is loss of cellular differentiation. Hyperplasia is an increase in the number of cells resulting from an increased rate of cellular division. Dysplasia refers to abnormal changes in the size, shpae, and organization of mature cells. REF: p. 77

a 75-year-old male presents with chest pain on exertion. The chest pain is most likely due to hypoxic injury secondary to: a. malnutrition b. free redicals c. ischemia d. chemical toxicity

ans: c the cardiac cells are deprived of oxygen, leading to ischemia, a reduction in blood supply to tissues. The cells are deprived of oxygen; they are not malnourished. Free radicals are electrically uncharged atoms or groups of atoms that have unpaired electron. chemical toxicity is not a factor in the chest pain. REF: p. 78-79

Stress induces sympathetic stimulation of the adrenal medulla. This causes the secretion of catecholamines, which include: a. epinephrine and aldosterone b. norepinephrine and cortisol c. epinephrine and norepinephrine d. cortisol and aldosterone.

ans: c the catecholamines are epinephrine and norepinephrine. neither aldosterone nor cortisol is a catecholamine REF: p. 221

Confirmation of somatic death is based on: a. presence of algor mortis b. presence of livor mortis c. complete cessation of respiration and circulation d. change in skin color to pale yellow

ans: c the most notable manifestations are complete cessation of respiration and circulation. algor mortis is postmortem reduction of body temp. and is ot confirmation of somatic death. livor mortis is muscle stiffening and is not confirmation of somatic death. change in skin color to pale yellow does occur, but complete cessation of respiration and circulation confirms somatic death. REF: p. 109

a 55-year-old male has swelling of the feet. Which of the following aided in the development of swelling? a. increased ATP b. cloride movement out of the cell c. Na+ movement out of the cell d. decreased oncotic pressure

ans: c when sodium and water enter the cell freely, cellular swelling, as well as early dilation of the endoplasmic reticulum, results. Decreased ATP would lead to swelling. Chloride movement out of the cell would affect muscle contraction but does not lead to swelling. Increased oncotic pressure would not affect swelling. REF: p. 97

a 50-year-old male intravenous drug user is diagnosed with hepatitis c. examination of the liver reveals cell death secondary to: a. fat necrosis b. physiologic apoptosis c. infection-induced apoptosis d. pyknosis

ans: c with hepatitis c, the liver will demonstrate apoptosis that is a result of the viral infection. fat necrosis occurs with enzymatic action due to lipases. apoptosis is not a normal physiological process. pyknosis occurs when the nucleus shrinks and becomes a small, dense mass of genetic material. REF: p. 104

Which of the following is an example of the protein hormone? a. Thyroxine (T4) b. Aldosterone c. Testosterone d. Insulin

ans: d Protein hormones are also water-soluble hormones, and insulin is a part of this group. Thyroxine, aldosterone, and testosterone are all lipid-soluble hormones and not protein hormones. ref: p. 440, table 18-1

A 24-year-old female presents with excessive menstrual bleeding. The physician identified endometrial changes that are due to hormonal imbalances. These cellular changes would be referred to as: a. dysplasia b. pathologic dysplasia c. hyperplasia d. pathologic hyperplasia

ans: d because the changes are due to an imbalance, they would be considered pathologic hyperplasia, a term more desripitive than simple hyperplasia. The endometrial changes were not abnormal in size and shape; thus, it is not dysplasisa regardless of cause. REF: pp. 76-77

After teaching about coping, which information indicates a correct understanding? coping is best defined as the process of: a. adjusting to disease b. preventing psychological distress c. mediating anger d. managing stressful chanllenges

ans: d coping is the process of managing, not adjusting to, stressful challenges that tax the individual's resources. coping is not preventing psychological distress, nor does it mediate anger. REF: p. 226

a 50-year-odl female became infected with clostridium bacteria and died a week later. examination of her red blood cells revealed lysis of membranes. which of the following was the most likely cause of her death? a. fat necrosis b. wet gangrene c. gangrenous necrosis d. gas gangrene

ans: d gas gangrene is a pecial type of gangrene caused by infection of injured tissue by one of many speices of clostridium. fat necrosis is cellular dissolution caused by powerful enzymes, called lipases, that occur in the breast, pancreas, and other abdominal structures. wet gangrene develops when neutrophils invade the site, causing liquefactive necrosis. gangrenous necrosis is due to death of tissue and results from severe hypoxic injury. REF: p. 104

a 23-year-old male develops a black eye following a fight. when the aide asks the nure why this occurred. the nurse's best response is that the bruising is due to an accumulation of: a. transferrin b. bilirubin c. albumin d. hemosiderin

ans: d hemosiderin is responsible for the color chanes in a black eye. Trasferrin is a transpoet protein reponsible for iron transport. Bilirubin is the normal, yellow-to-green pigment of bile derived from the prophyrin structure of hemoglobin. Albumin is the protein in the serum, responsible for cellular integrity. REF: p.100

A common pathway of irreversible cell injury involves increased intracellular: a. sodium b. potassium c. magnesium d. calcium

ans: d increased intracellular calcium levels activate cell enzymes (caspases) tha promote cell death by apoptosis. Persistent ischemia is associated with irreversible injury and necrosis. Irreversible injury is associaed sructurally with severe swelling of the mitochondria, severe damage to plasma membranes, and swelling of lysosomes. Cellular injury is not associated with sodium, potassium, or magnesium levels. REF: p. 81

A patient has a heart attack that leads to progressive cell injury that causes cell death with severe cell swelling and breakdown of organelles. what term would the nurse use to define this process? a. adaptation b. calcification c. apoptosis d. necrosis

ans: d necrosis is the sum of cellular changes after local cell death. Cellular adaptation is a reversible, structural, or functional response to both normal or physiologic conditions and adverse or pathologic conditions. Calcification is an accumulation of calcium salts. Apoptosis is an active process of cellular self-destruction. REF: p. 78, table 4-1

a 15-year-old female presents to the ER following a physical assault. She has internal damage to the neck with deep bruising. X-ray reveals fractures of the hyoid bone and tracheal and cricoid cartilage. Which of the following most likely caused her injuries? a. chemical asphyxiation b. choking asphyxiation c. ligature strangulation d. manual strangulation

ans: d squeezing of the neck as with strangulation would fraction the hyoid bone. Chemical asphyxiation would lead to breathing problems but would not result in fracture. Choking asphyxiation would lead to swelling of tissues but would not reault in fracture. In ligature strangulation, the mark on the neck is horizontal without the inverted V pattern seen in hangings. It would not lead to fracture. REF: p. 96

A student arrives at school to find that he/she has an exam for which he/she is unprepared. Which physiological response would be expected? a. decreased lipolysis b. bronchoconstriction c. decreased cortisol release d. increased glucagon release

ans: d the student would experience increased glucagon release to supply the increased glucose requirements. increased lipolysis and bronchodilation would occur, not constriction. the student would experience increased cortisol release. REF: p. 219, table 9-2

After ingestion of lead, what organ systems should the nurse monitor because they are the most sensitive to the effects of lead (select all that apply) a. heart b. lungs c. liver d. kidneys e. brain f. hematopoietic

ans: d, e, f the organ systems primarily affected by lead ingestion include the nervous system (brain), the hematopoietic system (tissues that produce blood cells), and the kidneys. the heart, liver, and lungs are not affected. REF: p. 89

specific

attacks only certain organisms

A 54-year-old male complains that he has been vomiting blood. Tests reveal portal hypertension. Which of the following is the most likely cause of his condition? a. Thrombosis in the spleen b. Cirrhosis of the liver c. Left ventricular failure d. Renal stenosis

b. Cirrhosis of the liver

In disseminated intravascular coagulation (DIC), the nurse assesses for active bleeding after intravascular clotting because: a. Prothrombin is activated. b. Clotting factors are depleted. c. Inflammatory mediators are released. d. Tissue factor (TF) is inactivated.

b. Clotting factors are depleted.

When a nurse is reviewing lab results and notices that the erythrocytes contain an abnormally low concentration of hemoglobin, the nurse calls these erythrocytes: a. Hyperchromic b. Hypochromic c. Macrocytic d. Microcytic

b. Hypochromic

A 58-year-old female presents in the clinic presenting with fatigue, weight loss, and tingling in her fingers. Laboratory findings show low hemoglobin and hematocrit, a high mean corpuscular volume, and normal plasma iron. These assessment findings are consistent with which type of anemia? a. Hemolytic anemia b. Pernicious anemia c. Iron deficiency anemia d. Aplastic anemia

b. Pernicious anemia

A newborn is diagnosed with congenital intrinsic factor deficiency. Which of the following types of anemia will the nurse see documented on the chart? a. Iron deficiency anemia b. Pernicious anemia c. Sideroblastic anemia d. Hemolytic anemia

b. Pernicious anemia

The _____ represents the sum of all ventricular muscle cell depolarization. a. PR interval b. QRS complex c. QT interval d. P wave

b. QRS complex

After erythrocytes have circulated for about 120 days, they are removed by macrophages, which are mainly in the: a. Liver b. Spleen c. Appendix d. Bone marrow

b. Spleen

A staff member asks what leukocytosis means. how should the nurse respond? leukocytosis can be defined as a. a normal leukocyte count b. a high leukocyte count c. a low leukocyte count d. another term for leukopenia

b. a high leukocyte count

Which of the following gastrointestinal (GI) clinical manifestations is subjective (select all that apply) a. retching b. anorexia c. nausea d. vomiting e. diarrhea

b. anorexia c. nausea

Which statement indicates teaching was successful regarding the classic pathway of the complement system? THe classic pathway of the complement system is activated by a. histamine b. antigen-antibody complexes c. leukotrienes d. prostaglandins

b. antigen-antibody complexes

A 24 year old male who sustained a head injury and fractured femur develops a stress ulcer. a common clinical manifestation of this ulcer is a. bowel obstruction b. bleeding c. pulmonary embolism d. hepatomegaly

b. bleeding

A patient with end-stage renal disease has pruritus. When the patient asks what causes this, what is the nurse's best response? Pruritus, seen in patients with end-stage renal disease, is caused by high levels of a. potassium b. calcium c. sodium d. magnesium

b. calcium

A 27 year old male has a severe kidney obstruction leading to removal of the affected kidney. Which of the following would the nurse expect to occur a. atrophy of the remaining kidney b. compensatory hypertrophy of the remaining kidney c. dysplasia in the remaining kidney d. renal failure

b. compensatory hypertrophy of the remaining kidney

A 22 year old male underwent brain surgery to remove a tumor. following surgery, he experienced a peptic ulcer. his ulcer is referred to as a ____ ulcer a. infectious b. cushing c. ischemic d. curling

b. cushing

After teaching the staff about the clotting system, which statement indicates teaching was successful? The end product of the clotting system is a. plasmin b. fibrin c. collagen d. factor x

b. fibrin

A 55 year old male presents reporting urinary retention. Tests reveal that he has a lower urinary tract obstruction. Which of the following is of most concern to the nurse a. vesicoureteral reflux and pyelonephritis b. formation of renal calculi c. glomerulonephritis d. increased bladder compliance

b. formation of renal calculi

An increase in the rate of red blood cell breakdown causes which form of jaundice a. obstructive b. hemolytic c. hepatocellular d. metabolic

b. hemolytic

Chronic gastritis is classified according to the a. severity b. location of lesions c. patient's age d. signs and symptoms

b. location of lesions

A 52 year old female presents with continuous abdominal pain that intensifies after eating. she is diagnosed with chronic pancreatitis. contributing factors include (select all that apply) a. alcohol abuse b. peptic ulcer disease c. trauma d. smoking e. bulimia

b. peptic ulcer disease c. trauma d. smoking

When a patient asks how nonsteroidal anti-inflammatory drugs such as ibuprofen work, how should the nurse reply? a. histamine b. prostaglandins c. leukotrienes d. interferon

b. prostaglandins Nonsteroidal antiinflammatory drugs block prostaglandins by inhibiting the cyclooxygenase (COX) 1 and 2 pathway.

A 21 year old female was recently diagnosed with iron deficiency anemia. in addition to fatigue and weakness, which of the following clinical signs and symptoms would she most likely exhibit a. hyperactivity b. spoon-shaped nails c. gait problems d. petechiae

b. spoon-shaped nails

buffer

bind to excessive H+ or OH- w/o significant change to pH

ligands

bind with cellular receptors to cause an action or reaction

Hypochloremia

blood plasma chloride levels <97 mEq/L occurs in CF (kidneys)

Most phosphate is located in the ____

bone

b cells formed

bone marrow

cancer of bone

bone pain

pH major organs involved

bones,lungs,kidneys

Which of the following individuals should the nurse assess first for a vitamin B12 deficiency anemia? a. 3-year-old female who is a fussy eater b. 26-year-old female in the second trimester of her first pregnancy c. 47-year-old male who had a gastrectomy procedure (removal of the stomach) d. 64-year-old male with a history of duodenal ulcers and gastrointestinal bleeding

c. 47-year-old male who had a gastrectomy procedure (removal of the stomach)

A 60-year-old female with a recent history of head trauma and a long-term history of hypertension presents to the ER for changes in mental status. MRI reveals that she had a hemorrhagic stroke. What does the nurse suspect caused this type of stroke? a. Rheumatic heart disease b. Thrombi c. Aneurysm d. Hypotension

c. Aneurysm

A 50-year-old female develops skin cancer on her head and neck following years of sunbathing. Which of the following cancers is the most likely? a. Lymphoma b. Adenoma c. Basal cell carcinoma d. Leukemia

c. Basal cell carcinoma

A 27 year old male presents with fever, GI bleeding, hepatomegaly, and transient joint pain. He reports that as a child he received blood transfusions following a motor vehicle accident. He also indicates he was vaccinated against hepatitis B. Which of the following types of hepatitis does the clinician think he most likely has a. A b. B c. C d. D

c. C

A 5-year-old female is diagnosed with acute leukemia. The nurse will most likely treat this patient with: a. Bone marrow transplant b. Immunotherapy c. Chemotherapy d. Localized radiation therapy

c. Chemotherapy

What is the cause of the hyperpigmentation seen in people with Cushing syndrome? a. Abnormal levels of cortisol b. Permissive effects of aldosterone when cortisol levels are altered c. Elevated levels of ACTH d. Hypersensitivity of melanocytes with sun exposure

c. Elevated levels of ACTH

A primary care provider is attempting to diagnose cancer and is looking for a tumor marker. Which of the following could be a possible marker? a. Red blood cells b. Apoptotic cells c. Enzymes d. Neurotransmitters

c. Enzymes

While reviewing lab results, the nurse recalls the most abundant cells in the blood are: a. Leukocytes b. Lymphocytes c. Erythrocytes d. Thrombocytes

c. Erythrocytes

A 65-year-old male experienced loss of appetite, weight loss, lemon-yellow skin, liver enlargement, and a beefy red tongue shortly before his death. Autopsy suggested pernicious anemia, and the cause of death would most likely reveal: a. Brain hypoxia b. Liver hypoxia c. Heart failure d. Kidney failure

c. Heart failure

A 35-year-old female is diagnosed with vitamin B12 deficiency anemia (pernicious anemia). How should the nurse respond when the patient asks what causes pernicious anemia? A decrease in ______ is the most likely cause. a. Ferritin b. Gastric enzymes c. Intrinsic factor d. Erythropoietin

c. Intrinsic factor

The role of physical activity in the prevention of colon cancer is identified by which of the following? a. It increases fluid loss leading to thirst and increased fluid intake, hydrating the colon. b. It increases blood supply thereby increasing oxygen to the colon. c. It increases gut motility thereby decreasing the time the bowel is exposed to mutagens. d. It increases the secretion of hydrochloric acid thereby killing mutants.

c. It increases gut motility thereby decreasing the time the bowel is exposed to mutagens.

A nurse is preparing to teach about erythropoietin. Which information should the nurse include? Erythropoietin is produced in the: a. Liver b. Bone marrow c. Kidneys d. Spleen

c. Kidneys

Tissue damage in pancreatitis is caused by: a. Insulin toxicity b. Autoimmune destruction of the pancreas c. Leakage of pancreatic enzymes d. Hydrochloric acid reflux into the pancreatic duct

c. Leakage of pancreatic enzymes

The nurse is planning care for a patient with heart problems. Which information should the nurse remember? The _____ artery travels down the interventricular septum and delivers blood to portions of the left and right ventricle. a. Right coronary b. Circumflex c. Left anterior descending d. Cardiac

c. Left anterior descending

A nurse is teaching about the heart. Which information should the nurse include? The chamber of the heart that generates the highest pressure is the: a. Right atrium b. Left atrium c. Left ventricle d. Right ventricle

c. Left ventricle

A 40-year-old male presents with epigastric pain. Tests reveal acute pancreatitis. The most likely cause of his condition is: a. Pancreatic duct obstruction by a malignant tumor b. Surgical trauma to the pancreas c. Obstruction of the biliary tract by a gallstone d. Toxic injury to the pancreas from nonprescription medications

c. Obstruction of the biliary tract by a gallstone

The nurse is reviewing a normal electrocardiogram. The nurse assesses the PR interval because it represents a. Atrial depolarization b. Ventricular depolarization c. Onset of atrial activation to onset of ventricular activity d. "Electrical systole" of the ventricles

c. Onset of atrial activation to onset of ventricular activity

A nurse is discussing fibrinolysis. Which information should the nurse share? Fibrinolysis is mediated by: a. Heparin b. Fibrinogen c. Plasmin d. Albumin

c. Plasmin

A 38-year-old female complains of epigastric fullness following a meal, nausea, and epigastric pain. Tests reveal narrowing of the opening between the stomach and the duodenum. This condition is referred to as: a. Ileocecal obstruction b. Hiatal hernia c. Pyloric obstruction d. Hiatal obstruction

c. Pyloric obstruction

A 55 year old male died in a motor vehicle accident. autopsy reveled an enlarged liver caused by fatty infiltration, testicular atrophy, and mild jaundice secondary to cirrhosis. the most likely cause of his condition is a. bacterial infection b. viral infection c. alcoholism d. drug overdose

c. alcoholism

When a patient asks what the most common type of renal stones is composed of, how should the nurse respond? The most common type of renal stone is composed of a. magnesium b. struvite c. calcium d. phosphate

c. calcium

A patient asks a nurse what leukotrienes do. What is the nurse's best answer? Leukotrienes: a. cause smooth muscle relaxation b. activate chemotactic cytokines c. cause increased vascular permeability d. activate complement proteins

c. cause increased vascular permeability Leukotrienes (slow-reacting substances of anaphylaxis [SRS-A]) are sulfur-containing lipids that produce histamine-like effects: smooth muscle contraction and increased vascular permeability.

Complete obstruction of bile flow to the liver would be manifested by a. elevated hemoglobin and hematocrit b. lower-leg edema c. clay-colored stools d. hypotension

c. clay-colored stools

A 25 year old female is diagnosed with urinary tract obstruction. while planning care, the nurse realizes that the patient is expected to have hydronephrosis and a decreased glomerular filtration rate caused by a. decreased renal blood flow b. decreased peritubular capillary pressure c. dilation of the renal pelvis and calyces proximal to a blockage d. stimulation of antidiuretic hormone

c. dilation of the renal pelvis and calyces proximal to a blockage

When the nurse observes a diagnosis of nosocomial pneumonia, the patient generally acquires this pneumonia a. at day care centers b. on airplanes c. during hospitalization d. in the winter season

c. during hospitalization

Individuals with a recent diagnosis of emphysema should be assessed for which most common presenting factor a. productive cough b. cyanosis c. dyspnea d. cor pulmonale

c. dyspnea

When a nurse is asked which of the following inhibits the inflammatory response, what is the nurse's best answer? a. neutrophils b. mast cells c. eosinophils d. basophils

c. eosinophils The second function, regulation of mast cell derived inflammatory mediators, is a critical function of eosinophils and helps limit and control inflammation. neutrophils release chemicals that promote inflammation. Mast cells release chemicals that promote inflammation. Basophils, which are similar to mast cells, release chemicals that promote inflammation.

A 7 year old female is diagnosed with nephrotic syndrome. Which of the following should the nurse ask the parents if they or the child has noticed recently a. sunken fontanelles b. vesicular skin rash c. frothy urine d. jaundice

c. frothy urine

A person with type 1 diabetes experiences hunger, lightheadedness, tachycardia, pallor, headache, and confusion. The most probable cause of these symptoms is: a. hyperglycemia caused by incorrect insulin administration. b. fawn phenomenon from eating a snack before bedtime. c. hypoglycemia caused by increased exercise. d. Somogyi effect from insulin sensitivity.

c. hypoglycemia caused by increased exercise.

A nurse is reviewing the results of an ABG and finds reduced oxygenation of arterial blood. what term should the nurse use to describe this condition a. ischemia b. hypoxia c. hypoxemia d. hypocapnia

c. hypoxemia

Manifestations associated with hepatic encephalopathy from chronic liver disease are the result of a. hyperbilirubinemia and jaundice b. fluid and electrolyte imbalances c. impaired ammonia metabolism d. decreased cerebral blood flow

c. impaired ammonia metabolism

Anemia accompanies chronic renal failure because of a. blood loss via the urine b. renal insensitivity to vitamin D c. inadequate production of erythropoietin d. inadequate retention of serum iron

c. inadequate production of erythropoietin

A 42 year old male is involved in a motor vehicle accident during which he loses a lot of blood. the nurse realizes he is in acute renal failure caused by a. kidney stones b. immune complex deposition in the glomerulus c. inadequate renal blood flow d. obstruction of the proximal tubule

c. inadequate renal blood flow

A 70 year old male with chronic renal failure presents with edema. Which of the following is the most likely cause of this condition a. increased capillary oncotic pressure b. decreased interstitial oncotic pressure c. increased capillary hydrostatic pressure d. increased interstitial hydrostatic pressure

c. increased capillary hydrostatic pressure

Pancreatic insufficiency is manifested by deficient production of a. insulin b. amylase c. lipase d. bile

c. lipase

A 50 year old male is experiencing reflux of chyme from the stomach. he is diagnosed with gastroesophageal reflux. this condition is caused by a. fibrosis of the lower third of the esophagus b. sympathetic nerve stimulation c. loss of muscle tone at the lower esophageal sphincter d. reverse peristalsis of the stomach

c. loss of muscle tone at the lower esophageal sphincter

In the respiratory tract of smokers, stratified squamous epithelium (a normal cell type) often replaces ciliated columnar epithelium in response to chronic irritation and inflammation, meaning the protective functions of the cilia are lost. This is an example of a. hyperplasia b. atrophy c. metaplasia d. dysplasia

c. metaplasia

Which lab test helps the nurse monitor the primary phagocyte in the blood? a. macrophage b. eosinophils c. neutrophils d. basophils

c. neutrophils

The most common cause of hypoparathyroidism is: a. pituitary hyposecretion. b. parathyroid adenoma. c. parathyroid gland damage. d. autoimmune parathyroid disease

c. parathyroid gland damage.

a 35 year old female with Graves disease is admitted to a medical surgical unit. which of the following symptoms would the nurse expect to find before treatment a. weight gain, cold intolerance b. slow heart rate, rash c. skin hot and moist, rapid heart rate d. constipation, confusion

c. skin hot and moist, rapid heart rate

A nurse assesses the heart after acetylcholine because the effect of acetylcholine on the heart is to: a. Decrease the refractory period b. Increase calcium influx c. Increase the strength of myocardial contraction d. Decrease the heart rate

d. Decrease the heart rate

A 45-year-old male complains of heartburn after eating and difficulty swallowing. He probably has: a. Pyloric stenosis b. Gastric cancer c. Achalasia d. Hiatal hernia

d. Hiatal hernia

Which condition will cause a patient to secrete erythropoietin? a. Low blood pressure b. Hypercarbia c. Inflammation d. Hypoxia

d. Hypoxia

Which of the following patients would be at greatest risk for basal cell carcinoma? a. Dark complexion, light eyes, underweight b. Light complexion, dark eyes, overweight c. Medium complexion, light eyes, smoker d. Light complexion, light eyes, fair hair

d. Light complexion, light eyes, fair hair

A 25-year-old female has a heavy menses during which she loses a profuse amount of blood. Which of the following adaptations should the nurse expect? a. Movement of fluid into the cell b. Decreased cardiac output c. Decreased oxygen release from hemoglobin d. Peripheral vasoconstriction

d. Peripheral vasoconstriction

When a patient wants to know why vitamin B12 and folate deficiencies cause anemia, how should the nurse respond? a. Red blood cells are unable to differentiate into erythrocytes. b. Red blood cells have malformed hemoglobin molecules. c. Red blood cells have decreased O2-carrying capacity. d. Red blood cells have a shorter life span.

d. Red blood cells have a shorter life span.

A nurse is preparing to teach the staff about aplastic anemia. Which information should the nurse include? Aplastic anemia is caused by: a. Iron deficiency b. Excess levels of erythropoietin c. Hemolysis d. Stem cell deficiency

d. Stem cell deficiency

A 60-year-old female with a history of cirrhosis presents with dyspnea, impaired ventilation, and pleural pain. A diagnosis of pleural effusion is made, and a watery fluid is drained. When giving report, the nurse will refer to this fluid as: A. Exudative b. Purulent c. Infected d. Transudative

d. Transudative

which condition should cause the nurse to assess for high-output failure in a patient a. metabolic alkalosis b. hypothyroidism c. hypovolemia d. anemia

d. anemia

A nurse is explaining the complement system. Which information should the nurse include? The components of the complement and kinin system are: a. antibodies b. WBCs c. Growth factors d. plasma protiens

d. plasma protiens

A 56 year old male presents with flank pain and polyuria. Tests reveal that he has an enlarged prostate. which of the following types of renal failure should the nurse monitor for as it is the most likely to occur a. prerenal b. intrarenal c. extrarenal d. postrenal

d. postrenal

A 40 year old female is diagnosed with cervical cancer after a Pap smear. Which of the following cellular changes would the nurse most likely see on the report a. metaplasia b. atrophy c. hypertrophy d. dysplasia

d. dysplasia

a nurse is preparing to teach on the subject of opsonins. which information should the nurse include? opsonins are molecules that a. are composed of fatty acids b. regulate inflammation c. degranulate mast cells d. enhance phagocytosis

d. enhance phagocytosis

A 25 year old female presents with burning urination. She was diagnosed with a urinary tract infection. When the nurse checks the culture results, which of the following organisms is most likely infecting her urinary tract a. streptococcus b. candidia albicans c. chlamydia d. escherichia coli

d. escherichia coli

Which of the following clusters of symptoms would make a clinician suspect a child has developed glomerulonephritis a. pyuria, fever, and abdominal pain b. proteinuria, hypotension, and ascites c. dysuria, urinary frequency, and abdominal tenderness d. gross hematuria, flank pain, and hypertension

d. gross hematuria, flank pain, and hypertension

which of the following alterations would the nurse expect to find in a patient with untreated Cushing disease or syndrome a. bradycardia b. tachypnea c. hyperkalemia d. hypertension

d. hypertension

A nurse recalls that to increase a macrophage's capacity to kill, they must be activated by: a. neutrophils b. plasma cells c. mast cells d. interferon

d. interferon IFN-y is produced by lymphocytes; it activates macrophages, resulting in increased capacity to kill infectious agents (including viruses and bacteria).

Which of the following symptoms would help a heath care provider distinguish between ulcerative colitis and Crohn disease a. abdominal pain b. pattern of remission/exacerbations c. diarrhea d. malabsorption

d. malabsorption

A nursing student is scheduled to undergo a clinical exam of his assessment skills under the supervision of the instructor. As a result of this stressor, the student has high serum levels of glucocorticoid hormones such as cortisol. Which of the following statements best captures an aspect of the role of glucocorticoid hormones such as cortisol in the physiological response to stress? a. blood glucose levels are decreased in anticipation of energy expenditure b. attention, arousal, and respiratory rate are decreased in order to prepare for a response c. the immune system is enhanced in response to an impending threat d. osteoblast activity and protein synthesis are suppressed in order to refocus energy

d. osteoblast activity and protein synthesis are suppressed in order to refocus energy

A 25 year old male presents with chronic bronchitis of 5 months' duration. which of the following is the most significant concern for the nurse to monitor in this patient a. left heart failure b. pulmonary embolus c. immunosuppression d. recurrent pulmonary infections

d. recurrent pulmonary infections

Which patient should the nurse assess for both hyperkalemia and metabolic acidosis? A patient diagnosed with a. diabetes insipidus b. pulmonary disorders c. Cushing syndrome d. renal failure

d. renal failure

diabetes insipidus, diabetes mellitus (DM), and SIADH share which of the following assessment manifestations a. polyuria b. edema c. vomiting and abdominal cramping d. thirst

d. thirst

A patient has warmth and redness of the skin from inflammation. When the nurse is asked what process is responsible for these assessment findings, how should the nurse respond? a. increased vascular permeability b. phagocytosis c. vasoconstriction d. vasodilation

d. vasodilation

A nurse is teaching a patient about fluid and electrolytes. Which of the following indicates the teaching was successful regarding electrolytes? Electrolytes are: a.Small lipid-soluble molecules b.Large protein molecules c.Micronutrients used to produce ATP d.Electrically charged molecules

d.Electrically charged molecules

ionizing radiation

damages the cancer cell DNA

chemotherapy and radiation do what to the GI tract

decrease cell growth in GI tract

anemia

decrease in hemoglobin in the blood

thrombocytopenia

decrease in platelets

metabolic acidosis

decreased pH in blood and body tissues as a result of an upset in metabolism

histaminease

degrades kinin and histamine

potassium adaption

slow changes tolerated better

Erythropoietin

stimulates red blood cell production, successful in fighting anemia in people w/ cancer -made by the kidneys

tonicity

the ability of a solution to cause a cell to gain or lose water

Connective tissue functions

ØBinds tissues and organs together. ØSupports the tissues and organs (keeps them where they are supposed to be). ØStores excess nutrients.


Conjuntos de estudio relacionados

Real Property Characteristics, Legal Descriptions, and Property Use

View Set

DOD Cyber Awareness Challenge 2024

View Set

Chapter 27: The Cold War and The Fair Deal

View Set

AP Biology- Biochemistry (Chapter 3-8)

View Set

Ch. 2: Transplantations and Borderlands

View Set